ORTHOPEDIC MCQS ONLINE OB 20 TRAUMA 2B

ORTHOPEDIC MCQS ONLINE OB 20 TRAUMA  2B

  1. CT angiography and admit the patient for hourly neurovascular checks

  2. Interventional radiology consult for formal angiography and stenting Corrent answer: 2

This patient has a posterior knee dislocation with an ischemic limb that does not reverse following reduction. Emergent vascular exploration and reconstruction is indicated.

 

Knee dislocations are associated with popliteal artery injury in 18-45% of cases and range from intimal tears to complete transection. Amputation rates of 85% have been reported if revascularization is delayed greater than 6 to 8 hours.

Neurologic injury occurs in 15-40% of cases and is most common after posterolateral dislocation. The peroneal nerve is more commonly injured.

 

Rihn et al. reviewed the acutely dislocated knee. They recommend a vascular consult if pulses are weak, or ABI is compromised. They warn that in arterial injury, pulses, temperature and capillary refill can be normal. If the limb remains ischemic, surgical exploration and revascularization is indicated.

 

Medina et al. systematically reviewed neurovascular injury after knee dislocation in 862 patients. Vascular injury rate was 18%, and nerve injury rate was 25%. Repair was performed in 80% of vascular injuries, and amputation in 12%. The most vascular injury was seen in KDIIIL injuries (32%) and posterior dislocation (25%).

 

Figure A is an AP radiograph of a posterior knee dislocation. Figure B is a lateral showing the same injury.

 

Incorrect Answers:

Answer 1: The limb remains ischemic. Splinting and observation alone are inappropriate.

Answer 3: External fixation is appropriate for immobilization following reduction, especially for very obese patients (BMI>50) where casting/splinting is not possible, and especially to protect a vascular reconstruction. However, the MOST important next step is vascular exploration to attempt revascularization.

Answers 4 and 5: CT angiography and standard angiography will only serve to delay vascular exploration of the ischemic limb. On-table angiograms will be performed during the surgery, obviating the need for preoperative angiography outside the operating room (OR).

 

 

 

OrthoCash 2020

  1. A 51-year-old carpenter presents with a distal humerus nonunion (Figures A and B). After repair of the nonunion, he is lost to follow up for 3 months and then presents to your clinic pain-free and with evidence of fracture union. On examination the elbow arc of motion is 10-80 degrees. Following a brief failed trial of conservative management, you recommend:

     

     

     

     

     

     

     

    1. Open release of the posterior bundle of the MCL and excision of osteophytes

    2. Complete open release of the MCL and LCL complexes with radiation for heterotopic ossification prophylaxis

    3. Arthroscopic release of the anterior capsule with removal of hardware

    4. Removal of hardware and dome osteotomy

    5. Functional bracing and recommendation for job retraining

    Corrent answer: 1

     

    Limited flexion outside a functional range of motion (30-130 degrees) can be quite debilitating, and it is often addressed with open release of the posterior bundle of the MCL and posterior capsule and excision of osteophytes.

     

    A frequent complication of distal humerus fractures is post-traumatic stiffness. This patient has had multiple procedures about the distal humerus, and the question stem implies that he did not complete a structured range of motion protocol following the repair of his nonunion. Beyond a trial of conservative management, there are many techniques described for capsule and soft tissue releases as well as bony resections if heterotopic ossification is present. This patient's severely limited flexion would best be addressed with a release of the posterior bundle of the MCL, and with flexion <90 preoperatively, an ulnar nerve transposition may be beneficial.

     

    Sears et al. discuss the pathophysiology, sequelae, and treatment of posttraumatic arthritis and stiffness in the elbow. Numerous approaches - both open and arthroscopic - are described in the literature. Ultimately, the authors discuss the specific anatomic cause of the stiffness should be addressed, and in the case of restricted flexion, this is commonly the posterior bundle of the MCL and the posterior capsule.

     

    Moritomo, Tada, and Yoshida have published a 9 patient case series on aggressive early management of heterotopic ossification of the posterior capsule and posterior oblique bands of the MCL. In their patients, open resection of these tissues was performed, improving range of motion an average of 75 degrees and in no patients causing iatrogenic instability.

    Figures A and B are x-rays of the patient's distal humerus nonunion. Incorrect Answers:

    Answer 2: Complete release of the collateral complexes would likely destabilize

    the elbow

    Answer 3: Adhesions or ossification of the anterior capsule are more likely to limit extension as opposed to flexion

    Answer 4: An osteotomy to alter the range of motion to a more functional arc would be a decent salvage option for this patient, but is less optimal given that the osteotomy would be performed near or through the prior nonuinon site. A soft tissue release should be performed first.

    Answer 5: This patient is failing conservative management and functional bracing is unlikely to be of assistance

     

     

    OrthoCash 2020

     

  2. A 51-year-old female presents with an acute inability to extend her thumb, four months after she was treated with cast immobilization for a minimally-displaced distal radius fracture. What is the most appropriate treatment at this time?

    1. Occupational therapy for strengthening

    2. Extensor carpi radialis longus transfer to extensor pollicus longus

    3. Extensor pollicis brevis transfer to extensor pollicus longus

    4. Extensor indicis proprius transfer to extensor pollicus longus

    5. Primary repair of extensor pollicus longus Corrent answer: 4

    A rare complication of non-displaced or minimally displaced fractures of the distal radius treated with a cast is a delayed rupture of the extensor pollicis longus (EPL) tendon. The EPL is the primary extensor of the interphalangeal joint of the thumb and also assists with metacarpophalangeal extension.

    Extensor indicis proprius transfer to the EPL is the most widely used and reported treatment for this condition.

     

    Magnussen et al. reviewed results of EIP transfer following ruptures of the EPL, with 19/21 good results. None of the cases had any loss of independent index finger extension although index extensor strength reduced to half of that of the contralateral side.

     

    Hove et al. reported a similar satisfaction rate following treatment of 15 patients. In his series of 4,400 distal radius fractures treated over a 5 year period, the incidence of delayed tendon rupture following distal radius fracture was 0.3 percent.

     

     

     

    OrthoCash 2020

     

  3. A 25-year-old female is involved in a motor vehicle collision. She presents with the isolated injury seen in Figures A through D. Her leg is swollen but her skin is intact. She has no clinical signs of compartment syndrome. Which of the following treatment options will allow for maintenance of fracture alignment and minimize the risk of soft tissue complications?

     

     

     

     

     

     

     

     

     

     

     

    1. Closed reduction and long-leg non-weight bearing cast

    2. Immediate open reduction of the tibia through an anterolateral approach

    3. Percutaneous fixation of the articular surface and intramedullary nailing

    4. Definitive uniplanar external fixator

    5. Immediate open reduction of the tibia through an anteromedial approach Corrent answer: 3

    The patient presents with a closed distal third metaphyseal-diaphyseal distal tibia fracture with simple intra-articular extension. Immediate intramedullary nailing along with percutaneous fixation of the articular component provides appropriate restoration of length, rotation and alignment and minimizes the risk of wound complication.

     

    Displaced distal third tibia fractures may be associated with simple intraarticular extension. Operative treatment of intra-articular distal tibia fractures has historically been performed with open reduction and internal fixation. Early open reduction and plate fixation of pilon fractures has been associated with high rates of infection and wound complication. In select patterns with simple articular extension, percutaneous screw fixation and medullary nailing may provide appropriate reduction with minimal soft-tissue risk.

     

    Marcus et al. evaluated the outcomes of immediate intramedullary nailing and percutaneous fixation of simple intra-articular distal tibia fractures (AO/OTA 43 C1 and C2). The authors found excellent rates of union and alignment, however caution against broad application of this technique until more rigorous randomized studies can be performed.

     

    Sirkin et al. retrospectively evaluated the results of a staged approach to management of intra-articular distal tibia fractures with temporary bridging external fixation followed by open reduction and internal fixation (ORIF). The authors found decreased rates of wound complication and infection compared with prior studies.

     

    Figures A and B demonstrate a distal third tibial shaft fracture with simple intra-articular extension. The axial and coronal CT cuts in Figures C and D further clarify the articular injury. Illustrations A and B demonstrate a comminuted distal third tibial fracture with simple intra-articular extension. Illustrations C and D are fluoroscopic images of the same injury after intramedullary nailing and percutaneous fixation of the articular component.

     

    Incorrect Answers:

    Answer 1: Closed reduction and casting of this injury results in increased rates of malalignment

    Answers 2 and 5: Immediate open reduction of pilon fractures has historically been associated with high rates of infection and wound complications

    Answer 4: This fracture pattern would be difficult to control with a uniplanar fixator. A circular or multiplanar fixator would be a potential treatment option

     

     

     

     

     

     

     

     

     

     

     

     

    OrthoCash 2020

     

  4. A 24-year-old woman is thrown from her motorcycle and sustains the closed injury shown in Figures A through C. Open reduction and internal fixation is planned. What surgical technique will best allow visualization of the joint surface and allow early range of motion?

     

     

     

     

     

     

     

     

    1. Bryan-Morrey approach and parallel plating

    2. Triceps reflecting anconeus pedicle approach and parallel plating

    3. Triceps reflecting anconeus pedicle approach and orthogonal plating on the posteromedial and lateral surfaces

    4. Olecranon osteotomy and parallel plating

    5. Olecranon osteotomy and orthogonal plating on the posteromedial and lateral surfaces

    Corrent answer: 4

     

    Access to complex intra-articular fractures is best achieved by an olecranon osteotomy (OO). Fixation can be with parallel plating or orthogonal plating.

     

    Bicolumnar fixation of distal humerus fractures should follow the principles outlined by O'Driscoll: Distal fragments should be held by as many screws as possible; every screw in the distal fragments should pass through a plate; each screw should engage as many articular fragments as possible.

     

    Galano et al. review treatment for bicolumnar distal humerus fractures. They note that the olecranon osteotomy, Alonso-Llames triceps sparing and Campbell triceps splitting approaches expose 57%, 46% and 35% of the articular surface, respectively. The OO and paratricipital (triceps sparing)

    approaches allow for early ROM. Protected motion is required for the O'Driscoll TRAP and Bryan-Morrey approaches for tendon-to-bone healing.

     

    Coles et al. retrospectively reviewed the OO in fixation of 70 fractures. Osteotomy fixation was with an intramedullary screw and dorsal ulnar wiring, or with a plate. The rate of OO increased with fracture difficulty (from AO type C1-C3). There was 1 delayed union but no nonunions.

     

    Figures A and B show a AO/OTA type C2 intraarticular distal humerus fracture. Figure C is a coronal CT scan showing intraarticular comminution. Illustration A shows fixation of the fracture with bicolumnar plating through an olecranon osteotomy approach. Illustration B shows the various approaches to the distal humerus (left, Campbell triceps splitting; center left, O'Driscoll triceps reflecting anconeus pedicle; center right, Bryan-Morrey approach, leaving the triceps attached laterally to the fasciocutaneous flap, but elevating it off the ulna; right, olecranon osteotomy). Illustration C shows 3 methods of olecranon osteotomy (A and B, Intra-articular transverse; C-F, Extra-articular oblique; G, Intra-articular chevron).

     

    Incorrect Answers:

    Answer 1: The Bryan-Morrey approach provides excellent medial exposure, but inadequate lateral exposure. The triceps is lifted subperiosteally, and the anconeus is released subperiosteally. Triceps repair depends on tendon-bone healing.

    Answer 2: The O'Driscoll TRAP approach detaches triceps and anconeus from their distal insertions in a V-shaped flap that is reflected proximally. Repair to bone is via drill holes in the ulna. Early mobilization is not recommended.

    Answers 3 and 5: Orthogonal plating is performed on the POSTEROLATERAL and MEDIAL surfaces.

     

     

     

     

     

     

     

     

     

     

    OrthoCash 2020

     

  5. A 31-year-old male sustains an isolated handgun injury shown in Figure A. On physical examination, he has a 0.5 cm wound over the anteromedial aspect of his thigh, with no gross contamination of the skin edges. He is neurovascularly intact. The patient has received tetanus prophylaxis and antibiotics in the emergency department.

    What would next best step in treatment?

     

     

     

    1. External fixation and IV antibiotics for 6 weeks

    2. Irrigation and debridement of wound and external fixation for 6 weeks

    3. Reamed antegrade or retrograde intramedullary nail

    4. Irrigation and debridement of wound, reamed antegrade nail and IV antibiotics for 6 weeks

    5. Irrigation and debridement of wound, reamed retrograde nail and IV antibiotics for 6 weeks

    Corrent answer: 3

     

    This patient has sustained a low-velocity gunshot to the right femoral diaphysis. The next best step in treatment would be insertion of a reamed antegrade or retrograde intramedullary nail. During surgery, the gunshot wound could be closed primarily if it was considered clean, or allowed to heal by secondary intention if it was there was any concern.

     

    Low-velocity gunshot wounds are typically treated as closed fractures, since the risk of osteomyelitis is very low. These are typically bullets from handguns that travel <350 meters per second. Formal irrigation and debridement is usually considered only when there are nonviable skin edges or when the injury occurs from high-velocity bullets (>600 meters per second). The timing and direction (e.g. antegrade or retrograde) of nail insertion has not shown to

    affect outcomes.

     

    Cannada et al. reviewed 74 cases of femoral diaphyseal fractures caused by gunshots. All were treated with reamed, retrograde, statically locked intramedullary nailing, without formal irrigation and debridement in the operating room. Outcomes showed low incidence of shortening, angular deformity and infection rate.

     

    Dougherty et al. compared retrograde vs. antegrade intramedullary nailing for the treatment of gunshot diaphyseal femur fractures. No significant differences were found between groups with regards to operative time, blood loss, or radiographic union. They conclude that immediate retrograde or antegrade nailing is safe for the early treatment of gunshot femur fractures.

     

    Figure A shows an AP radiograph of the right femur. There is a gunshot fracture in the proximal femoral shaft with retained bullet fragments. The leg has been placed in temporary traction.

     

    Incorrect Answers:

    Answer 1: External fixation would be considered for treatment of femur fractures in damage control orthopaedics.

    Answer 2: IV antibiotics for 6 weeks would not be necessary with low velocity gunshot wounds with clean wounds.

    Answer 4 and 5: The direction of nail entry does not seem to affect outcomes with these fractures. Irrigation and debridement of wound, as well as IV antibiotics, wound not be necessary, aforementioned.

     

     

     

    OrthoCash 2020

     

  6. Figure A shows a radiograph of a 19-year-old male with an isolated shotgun injury. On physical examination, he has a 3 cm wound over the posterolateral aspect of his arm. In addition, he has weakness with wrist and finger extension, but no sensory deficits. The patient has received tetanus prophylaxis and antibiotics in the emergency department. What would be the next best steps in treatment?

     

     

     

    1. Urgent open reduction internal fixation and fibular osteoseptocutaneous flap

    2. Irrigation and debridement and external fixation

    3. Irrigation and debridement, repair of the radial nerve and retrograde intramedullary nail

    4. CT scan, urgent open reduction internal fixation and IV antibiotics for 6 weeks

    5. Closed reduction, CT scan, immobilization in a coaptation splint Corrent answer: 2

    This patient has sustained a shotgun blast to the midshaft humerus. The next best step would be irrigation and debridement, and external fixation of the fracture.

     

    Shotgun injuries are typically treated as open fractures, whereas low-energy gunshot wounds are treated as closed fractures. The initial operative treatment of shotgun wounds should include irrigation and debridement and stabilization with external fixation. The goal of treatment is to stabilize the severe soft-tissue injury and bone loss, as well as to aggressively debride devitalized tissue and gross contamination.

     

    Dougherty et al. reviewed gunshot fractures to the humerus. They report that

    peripheral nerve injuries are relatively common with these injuries, with a more common incidence in distal injuries than proximal.

     

    Joshi et al. looked at low velocity gunshot fractures to the humerus. They found that these injuries can be safely treated as closed fractures with local wound care, fracture brace and oral antibiotics. The time to union was similar in the non-operative and open treatment group.

     

    Berick et al. examined the indications for nerve exploration with humerus gunshot fractures. They recommend continued observation of isolated nerve palsies associated with gunshot fractures of the humerus. However, consider early nerve exploration of palsies when associated with a concomitant vascular injury.

     

    Figure A shows a high velocity shotgun fracture to the left humerus with retained buck fragments. Illustration A shows a similar injury stabilized with external fixation.

     

    Incorrect Answers:

    Answer 1: Due to the severity of soft tissue injury, external fixation should be considered for initial stabilization. Vascularized fibular grafts has been expanded to an osteoseptocutaneous flap by including a cutaneous flap on the lateral aspect of the lower leg. These are not typically used in the acute setting.

    Answer 3: Retrograde intramedullary nail would not be the ideal implant for this injury, acutely.

    Answer 4 and 5: There is no indication for a CT scan or prolonged antibiotics. The fracture and soft-tissue injury should be initially stabilized with external fixation. Coaptation splinting will interfere with soft tissue care.

     

     

     

     

     

    OrthoCash 2020

     

  7. A 34-year-old male sustains the closed injury seen in Figure A as a result of a high-speed motor vehicle collision. What is the most appropriate next step in treatment?

     

     

     

    1. Open reduction and internal fixation

    2. Spanning external fixation

    3. Percutaneous internal fixation

    4. Closed reduction and cast placement

    5. Ankle arthrodesis

     

    Corrent answer: 2

     

    The radiograph shows a comminuted pilon fracture, which is associated with high-energy trauma and significant soft tissue injury. The tested concept here is the importance of avoiding definitive reduction and fixation of this high-energy injury, which has been shown to be associated with an increased risk of wound complications and deep infections (as compared to staged treatment with usage of a spanning external fixator).

     

    Patterson et al. reviewed 23 consecutive patients with comminuted distal tibia fractures. They showed 0% infections or wound-healing problems in their patient population treated with a two-staged protocol. Their protocol involved fibula fixation with an intramedullary implant and application of a medial external fixator to to regain length and restore anatomic alignment. Reevaluation of the limb occurred ten to fourteen days later for definitive fixation.

     

    Sirkin et al. retrospectively reviewed 40 closed and 17 open pilon fractures (AO types 43A-C) that were treated with staged surgical management (avg. time from ext. fix. to formal reconstruction was 14 days (range 4 to 31) They reported 17% post-operative wound complication in the closed group and 11% post-operative wound complication in the open group (Gustilo Type I-III). They suggest the technique was successful in both closed and open pilon fractures.

     

     

     

    OrthoCash 2020

     

  8. A 72-year-old woman falls down stairs and sustains the injury shown in Figure A. Additional radiographs are performed in Figure B to help assess the fracture pattern. These additional radiographs represent which of the following?

     

     

     

     

     

     

    1. Oblique radiographs

    2. In-situ radiographs under anesthesia

    3. Traction radiographs

    4. Fracture flexion radiographs

    5. Fracture extension radiographs Corrent answer: 3

    Additional traction radiographs are performed in distal humerus fractures to better understand the fracture configuration.

     

    Traction radiographs help align the fracture fragments for better visualization. This is useful in comminuted fractures where distal fragments are telescoped or flexed on the proximal fragments, making CT scan visualization difficult.

    Proper visualization can help determine (1) the approach to the distal humerus, (2) whether to perform ORIF or total elbow arthroplasty (in select patients), (3) whether to add a 3rd plate (lateral column), (4) whether to add tricortical bone graft to augment distal fixation and restore trochlear width.

    Doornberg et al. examined whether 3D CT improved fracture characterization. They found that 3D CT improved interobserver and intraobserver reliability for fracture characterization and classification. It also improved intraobserver accuracy for fracture characteristics, but not interobserver accuracy. They concluded that 3D CT is helpful for preoperative planning.

     

    Galano et al. reviewed treatment strategies for bicolumnar distal humerus fractures. They state that 3D CT compensates for oblique scans from formatting in an incorrect plane, and allows for subtraction of the radius/ulna to properly demonstrate articular incongruity, degree of comminution and column involvement.

     

    Figures A and B show a distal humerus fracture without traction (A) and with traction (B) applied. Illustration A shows the same fracture after bicolumnar plating. Illustration B shows the different surgical approaches to the distal humerus (A, Campbell triceps splitting; B, O'Driscoll triceps reflecting anconeus pedicle [TRAP]; C, Bryan-Morrey triceps reflecting; D, olecranon osteotomy)

     

    Incorrect Answers:

    Answer 1: These radiographs aren't oblique images.

    Answer 2: Radiographs under anesthesia do not add further information unless traction is applied.

    Answers 4 and 5: Flexion and extension will further impact and displaced fragments and will not improve alignment.

     

     

     

     

     

     

     

     

    OrthoCash 2020

     

  9. Mirror therapy is indicated for which of the following clinical conditions?

    1. Impaired proprioception after lower extremity injury

    2. Two-stage flexor tendon reconstruction

    3. Plumbism

    4. Anterior cruciate ligament reconstruction

    5. Phantom pain

     

    Corrent answer: 5

     

    Mirror therapy is a useful technique to improve phantom pain in amputees. Phantom limb pain is a painful sensation that is perceived within a body part that no longer exists.

     

    Under mirror therapy, a patient is allowed to feel the imaginary movement of the removed body part behaving as normal body movement through a mirror. The mirror image of the normal body part helps reorganize and integrate the mismatch between proprioception and visual feedback of the amputated segment.

    Tilak et al. performed a randomized controlled trial of 26 patients with phantom limb pain, and found that mirror therapy as well as transcutaneous electrical nerve stimulation both significantly improved pain scores, but no difference was seen between the two groups. They recommend use of both modalities in treatment of phantom limb pain, as they are noninvasive and inexpensive.

     

    Chan et al. reviewed 22 patients utilizing mirror therapy for amputation related phantom limb pain, and found that patients reported a significant decrease in pain when utilizing this treatment modality as compared to covered mirror therapy or mental imagery therapy.

     

    Illustration A shows a patient with an upper extremity amputation using a mirror to perform movements during a mirror therapy session.

     

    Incorrect Answers:

    Answer 1: Mirror therapy is not indicated for proprioception training. Answer 2: Mirror therapy is not indicated for flexor tendon injuries. Answer 3: Mirror therapy is not indicated for lead poisoning.

    Answer 4: Mirror therapy is not indicated for ACL reconstruction.

     

     

     

     

     

     

    OrthoCash 2020

     

  10. A patient presents to the emergency department with the injury seen in Figure A. Which of the following is true about radial nerve palsies associated with isolated humeral shaft fractures after low velocity gunshot wounds?

     

     

     

    1. The initial treatment involves debridement, irrigation, nerve exploration, and osteosynthesis.

    2. The radial nerve palsy is often a result of neurotmesis.

    3. Initial treatment involves splinting and observation for return of neurologic function.

    4. Electrophysiologic testing for radial nerve palsies is indicated after 2-3 weeks without improvement.

    5. The radial nerve palsy will not resolve regardless of attempted interventions.

     

    Corrent answer: 3

     

    When a patient sustains an isolated humeral shaft fracture and radial nerve palsy from a GSW, the initial treatment involves splinting with observation.

     

    The majority of humeral shaft fractures are treated initially with a coaptation splint and then transitioned to a functional brace. Absolute surgical indications for operative management include: open fracture, brachial plexus injury, compartment syndrome, floating elbow, or vascular injury. A radial nerve palsy is not an indication for surgical management of an isolated humeral shaft fracture. Radial nerve injury from a low-velocity GSW is similar to that of blunt force trauma and thus, immediate exploration is not necessary. Instead, nerve function should be observed. Seventy percent of these nerve injuries will resolve spontaneously. Additionally, the patient should receive a short course of antibiotics as any low velocity GSW would be treated.

    Guo et al retrospectively reviewed the electrophysiologic data for 40 radial nerve palsies caused by GSWs and blunt trauma. After characterizing the palsies by level of injury, completeness of nerve injury, and other associated nerves injured, they found there to be no difference in any of these variables between GSW induced radial nerve palsies and blunt trauma induced palsies.

     

    Vaidya et al retrospectively reviewed the outcomes of 54 patients with humeral shaft fractures resulting from low velocity GSWs comparing operative and nonoperative treatments. They found that patients receiving non-operative management did well and that 70% of radial nerve palsies in the non-operative treatment group resolved on their own. They recommended non-operative treatment for the majority of isolated humeral shaft fractures resulting from civilian gunshot wounds.

     

    Figure A is a radiograph of a humeral shaft fracture after a GSW. Illustration A is a radiograph of a humeral shaft fracture with a coaptation in place.

    Illustration B is an example of a functional brace.

     

    Incorrect answers:

    Answer 1: This is the initial treatment for an open humeral shaft fracture associated with an radial nerve palsy.

    Answer 2: Radial nerve palsy from a low velocity GSW is usually axonotmesis. Neurotmesis the most common injury in open fractures.

    Answer 4:EMG testing is not indicated until 3-4 months of radial nerve palsy with no neural recovery.

    Answer 5: Seventy percent of radial nerve palsies will spontaneously resolve

     

     

     

     

     

    OrthoCash 2020

     

  11. A 76-year-old male community ambulatory presented to clinic complaining of pain in the left groin that has been persistent for the last 8 months. Radiographs obtained from clinic are seen in Figure A. You suspect a femoral neck nonunion and obtain a CT scan which confirmed it. Which of the following statements is true?

     

     

     

    1. Total hip arthroplasty will provide patient with the best long term outcomes

    2. Valgus malreduction is closely correlated with failure of fixation after reduction and cannulated screw fixation

    3. The patient's best outcome would be with an open reduction, bone grafting, and changing to an inverted triangle configuration of screws

    4. The most appropriate treatment is a valgus osteotomy to correct malreduction

    5. Providing this patient with a hemiarthroplasty increases rates of postoperative dislocation when comparing to total hip arthroplasty

    Corrent answer: 1

     

    A total hip arthroplasty (THA) after nonunion of a femoral neck fracture would provide the best long term outcomes in a 76-year-old male who is a community ambulator.

     

    After nonunion of a femoral neck fracture, hemiarthroplasty and THA are good salvage option for the physiologically older patients. When deciding between these two options, THA is better for active and cognitively intact patients. THA is also indicated in patients with radiographic evidence of degenerative disease about the acetabulum. Hemiarthroplasty is advocated for patients who are older and less active.

     

    Yang et al. retrospectively investigated the risk factors for nonunion in patients treated with cannulated screws. They reviewed 202 patients who had femoral neck fractures and were treated with internal fixation with cannulated screws. They identified that triangle configuration, displaced fracture, borderline or unacceptable reduction, and increased screw shaft subchondral purchase over the femoral neck were all risk factors for nonunion after internal fixation.

    Inverted triangle configuration was found to increase rate of union.

     

    Archibeck et al. retrospectively reviewed the outcomes of 102 THAs after failed internal fixation for a hip fracture (including both femoral neck and intertrochanteric). They concluded that the conversion of failed hip internal fixation has elevated risks compared to a primary THA, however, it may still be successful. The biggest concern for these patients postoperatively are periprosthetic fracture and dislocation.

     

    Figures and Illustrations:

    Figure A is an AP pelvic radiograph demonstrating a nonunion of a femoral neck fracture after suboptimal fixation with 3 cannulated screws in a triangle configuration.

    Illustration A is an AP radiograph of the left hip in this patient following conversion to THA.

    Illustration B is a radiograph demonstrating a valgus osteotomy. Illustration C shows an example of a femoral neck nonunion with varus malreduction.

     

    Incorrect Answers:

    Answer 2: Varus reduction is closely correlated with failure in this fixation method.

    Answer 3: The patient's best outcome would be with THA.

    Answer 4: This would be a good option in a younger patient, but given age and functional capacity, the best option is THA.

    Answer 5: Patients with hemiarthroplasty have lower rates of dislocation.

     

     

     

     

     

     

     

     

    OrthoCash 2020

  12. A 22-year-old male cyclist was struck by a car. He complains of right knee pain and swelling, as well as reduced sensation and weakness in his right foot. His leg compartments are soft and not tender. Distal pulses in the extremity are palpable. Radiographs of the knee, as shown in Figures A and B, were taken after a closed reduction maneuver was performed. What would be the next best step in management of this patient?

     

     

     

     

     

    1. Non-operative managment, but arrange for early follow-up in clinic

    2. Intra-compartmental pressure measurements

    3. Ankle-brachial index measurements

    4. Knee spanning external fixation

    5. Open reduction internal fixation Corrent answer: 3

    This patient presents with a Schatzker IV tibia plateau fracture with lower extremity neurologic deficits. The next best step would be to investigate for an acute vascular injury with ankle-brachial index measurements.

     

    Fracture-dislocations of the knee must be suspected with all Schatzker type IV injuries as this fracture pattern is usually associated with high energy trauma. Identifying this injury should prompt a thorough assessment of the neurovascular structures across the knee. After closed reduction and emergent immobilization of the knee, ankle brachial indices (ABI) must be immediately performed. If <0.9, further vascular testing is warranted, such as MR or CT angiography.

     

    Berkson et al. reviewed high energy tibia fractures. They state that Schatzker Type IV fractures typically requires more energy than corresponding lateral plateau fractures, due to denser bone on the medial side.

     

    Chang et al. described an anatomic sub-classification of Schatzker IV fractures. They describe Group 1* fractures as classic medial unicondylar fractures.

    Group 2* fractures are complicated variants characterized by medial condyle fractures with lateral plateau extension. Usually these have articular impaction of the centroposterior lateral plateau.

     

    Figures A and B show AP and lateral radiographs of the knee demonstrating a classic medial unicondylar Schatzker IV fracture. Note the anterior subluxation of the tibia in relation to the femur. Illustration A shows the multiple CT images of this fracture pattern.

     

    Incorrect Answers:

    Answer 1: A vascular injury should be investigated in patients presenting with high energy tibia plateau fractures or fracture patterns associated with vascular injuries (e.g. Schatzker type 4,5,6)

    Answer 2: This patients foot parasthesias is not due to increased leg compartment pressures. The subjective neurovascular findings is likely related to stretch or injury to the common peroneal nerve

    Answer 4: Knee spanning external fixation may be considered in situations of significant soft-tissue injury or documented vascular injury to protect the

    repair. The next step would be to investigate a vascular injury before applying an external fixator.

    Answer 5: This patient requires investigations for vascular injury, nerve injury and soft-tissue injury before planning operative fixation.

     

     

     

     

     

     

    OrthoCash 2020

     

  13. A 25-year-old woman sustains a fall on an outstretched hand. She complains of elbow pain. Examination reveals tenderness over the lateral elbow and pain on elbow motion. Injury radiographs and CT scans are shown in Figures A and B, respectively. What is the next best step?

     

     

     

     

     

     

    1. Splint until swelling subsides, then long-arm cast

    2. Excision of fracture fragments

    3. Closed reduction and percutaneous pinning

    4. Open reduction and internal fixation (ORIF)

    5. Radial head replacement Corrent answer: 4

    This patient has Mason II radial head fracture. ORIF will give the best results.

     

    Non-/minimally displaced radial head fractures without a block to rotation can be managed nonoperatively. Complete articular fractures with >=3 fragments do better with radial head replacement. Indications for ORIF include large articular surface fragments, > 2 mm of displacement, mechanical block to forearm rotation, or associated fractures or ligament injuries requiring surgery.

     

    Pike et al. retrospectively compared patients undergoing ORIF for isolated radial head fractures with radial fractures associated with other fractures/dislocations. They found no differences in pain/disability and complications or secondary capsular release between groups.

     

    Yoon et al. retrospectively compared isolated partial articular displaced (2-5mm) radial head fractures treated nonoperatively vs ORIF. They found no clinical benefit with ORIF compared to non-operative management. The ORIF

    group had more complications. Younger patient age and larger fracture displacement favored operative intervention. Younger patients fared worse.

     

    Figures A and B are radiographs and 3D reformatted CT images showing a displaced partial articular radial head fracture.

     

    Incorrect Answers:

    Answer 1: Nonoperative management is indicated for non-/minimally displaced fractures.

    Answer 2: Excision is an option for small partial articular fragments <25% of the radial head.

    Answer 3: There is no role for closed reduction and pinning in an adult. Answer 5: Radial head replacement is indicated for complete articular fractures with >= 3 fragments.

     

     

     

    OrthoCash 2020

     

  14. When placing an intramedullary nail for closed distal tibia shaft fractures, all of the following methods are described techniques to aid anatomic reduction EXCEPT:

    1. Plating of a concomitant fibula fracture

    2. Percutaneous placement of reduction foreceps at the fracture site

    3. Placing a Poller screw

    4. Placing a small-fragment plate at the fracture site

    5. Placing syndesmotic fixation Corrent answer: 5

      Of the choices above, syndesmotic fixation would not help anatomically reduce these fractures.

       

      Fractures of the distal tibia are challenging fractures to achieve an anatomic reduction. The medullary canals at either end of the tibia are capacious compared to the mid-diaphysis thus inhibiting an endosteal fit. These fractures have a tendency to become mal-aligned unless certain steps are taken to ensure restoration of the length, alignment, and rotation. Using a Poller or blocking screw can function to narrow the space available for the nail to guide the nail or correct deformity. The use of large pointed-reduction forceps or a small frag plate may also be used for provisional fixation. Additionally, plating concomitant fibula fractures may help indirectly restore length and reduce the tibia fracture. Concomitant syndesmotic injuries are not commonly seen and fixation for these is not required unless stress exam clearly demonstrates it.

      Casstevens et al. reviewed the surgical management of distal tibia fractures using either plates/screws or intramedullary nails. Current literature shows near equivalence of either technique regarding union and alignment. They recommend treating each fracture on a case-by-case basis with consideration of the soft tissues and fracture pattern when determining fixation choices.

       

      Tejwani et al. reviewed surgical techniques for managing metaphyseal and meta-diaphysial tibia fractures. Advances in tibial nail design have helped tremendously in negotiating these fractures. Though current supporting literature is based mostly on case series, the use of suprapatellar entry intramedullary nails may help avoid the common deformity of procurvatum and valgus seen with proximal tibia fractures.

       

      Egol et al. retrospectively reviewed distal tibial shaft fracture undergoing intramedullary nailing, with and without concomitant fibula fracture plate fixation. They found fractures with fibular fixation maintained better alignment beyond 12 week post-operatively than those without fixation. Given these findings they recommended fibular plating for all distal tibial shaft fractures.

       

      Illustration A shows a fibula fracture that was plated prior to tibia IMN placement. Illustrations B and C show the use of pointed reduction forceps placed percutaneously to reduce the fracture. Illustration D shows the use of a blocking screw (denoted by the arrow) to keep the nail lateralized and the fracture out of valgus. Illustration E shows the use of a plate to reduce the fracture prior to nail placement.

       

      Incorrect Answers:

      Answers 1, 2, 3, and 4: All of these have been described in the literature to help aid in reducing these fractures

       

       

       

       

       

       

       

       

       

       

       

       

       

       

       

      OrthoCash 2020

       

  15. A 70-year-old woman falls down a flight of stairs and sustains the injury shown in Figures A and B to her dominant upper extremity. She lives alone and has no other medical history. Which of the following surgical options is best to minimize complications for this fracture pattern?

     

     

     

     

     

    1. Open reduction and internal fixation with a long locking plate

    2. Uncemented reverse total shoulder arthroplasty with tuberosity repair

    3. Cemented reverse total shoulder arthroplasty without tuberosity repair

    4. Cemented reverse total shoulder arthroplasty with tuberosity repair

    5. Proximal humeral resection with endoprosthetic replacement Corrent answer: 4

    This patient has a complex proximal humerus fracture with metaphyseal comminution and poor bone stock. Cemented reverse total shoulder arthroplasty (rTSA) using a long stem prosthesis and tuberosity repair is indicated.

     

    Proximal humerus fragility fractures are hard to treat because of comminution and poor bone stock. AVN is common with glenohumeral fracture-dislocation. Hemiarthroplasty (and standard total shoulder arthroplasty, TSA) is unreliable because of dependence on tuberosity healing. rTSA is recommended for fractures in patients >70 years with severely comminuted fractures, high likelihood of head AVN, and poor tuberosity bone quality (osteoporosis or comminution).

     

    Bufquin et al. retrospectively reviewed the use of rTSA for treatment of 43 patients with 3- and 4-part proximal humerus fractures. They found satisfactory elevation (97°), ER in abduction (30°), constant scores (44) and modified Constant scores (66%). Complications included calcification (90%), tuberosity displacement (53%) and scapular notching (25%). They concluded that rTSA was a good procedure because it provides pain relief and easier functional recovery in spite of failed tuberosity healing.

     

    Anakwenze et al. systematically reviewed acute proximal humerus fractures. Frequency weighted range of motion was flexion 122°, abduction 97°, ER at neutral 18°. Tuberosity repair yielded higher ER compared to no repair.

    Scapular notching was the most common complication (32%). They concluded that rTSA patients tended to be elderly women with 4-part fractures, had good pain control but residual dysfunction.

     

    Jobin et al. reviewed rTSA for management of proximal humerus 3- and 4-part fractures. They note >50% of cases have tuberosity resorption. They recommend repairing the greater tuberosity to restore infraspinatus/teres minor function which improves external rotation strength. Greater tuberosity malunion is not a result of secondary displacement, but rather, from intraoperative malreduction. Lesser tuberosity repair should be performed if there is significant bone loss or intraoperative anterior instability.

    Figures A and B are AP radiograph and 3D reformatted CT scan showing comminuted proximal humerus fracture dislocation. Illustration A shows rTSA performed with distal cementation of a long stem prosthesis and cerclage fixation of the proximal shaft and the tuberosity fragments.

     

    Incorrect Answers:

    Answer 1: ORIF has potential complications of screw cut out, AVN, fracture collapse, tuberosity resorption and loss of fixation. The risk of AVN (and resultant screw cut out) is high because of comminuted fracture-dislocation. Answer 2: Cementing is indicated because metaphyseal fracture bone loss compromises stem fixation.

    Answer 3: Greater tuberosity repair improves external rotation strength. Lesser tuberosity repair adds to anterior stability.

    Answer 5: Proximal humeral replacement is only necessary for tumor or unreconstructable fractures. They have poorer outcomes because of loss of rotator cuff function.

     

     

     

     

     

     

    OrthoCash 2020

     

  16. A 26-year-old male underwent statically locked intramedullary nail fixation for a comminuted left femur fracture. An early post-

    operative computed tomography (CT) scanogram was taken to check rotational alignment, as shown in Figure A. What would be the next best step in the management of this patient?

     

     

     

     

    1. Observation and close follow-up

    2. Dynamization of the intramedullary nail

    3. Revision surgery, internally rotate distal fragment by 19 degrees

    4. Revision surgery, externally rotate distal fragment by 8 degrees

    5. Revision surgery, internally rotate proximal fragment by 11 degrees Corrent answer: 1

    The CT scanogram shows the operative left femur is 8 degrees externally rotated compared to the native right femur. No correction is required unless malalignment is >15 degrees and symptomatic. Therefore, the most appropriate next step would be to continue with postoperative observation and close follow-up.

     

    The primary purpose of CT scanogram is to measure the angle of rotation of the femoral neck relative to the femoral condyle. To do this, the right and left femurs must be scanned together using a 5mm helical slice scanner at the hip and knee. The first slice should reveal the alignment of the femoral neck, so as to allow for measurement of the femoral neck-to-horizontal (FNH) angle. The second slice should reveal the alignment of the posterior femoral condyles.

    This allows measurement of the posterior condyle-to-horizontal (PCH) angle. Finally, to calculate the rotational alignment (RA), the FNH angle and PCH angles are subtracted (e.g., RA = FNH - PCH). Normal RA is usually +5 to +20 degrees, which is also referred to as 5 to 20 degrees of femoral anteversion.

     

    Lindsey et al. reviewed femoral malrotation following intramedullary nail fixation. They showed the incidence of rotational malalignment was ~28%. Normal femoral neck anteversion (angle of the femoral neck relative to the transverse axis through the femoral condyles) is ~11-13°. However, they noted that some patients have up to 15° difference in rotation in native limbs.

    Therefore <15 degrees of rotational difference after fixation is considered acceptable.

     

    Gugala et al. examined the long-term functional implications for patients with iatrogenic femoral malrotation following femoral intramedullary nail fixation. They showed that patients can compensate for even significant femoral malrotation (up to 30 degrees) and tolerate it well. However, external femoral malrotation (more common) appears to be better compensated/tolerated than internal malrotation.

     

    Figure A shows that the left femoral neck is externally rotated (ER) by 15° to the horizontal (ER15). The right femoral neck is externally rotated (ER) by 4° to the horizontal (ER4). The left distal fragment is ER10. The right distal fragment is internally rotated (IR) by 9°. Thus, left femur has a total (ER15)-(ER10)= (+15)-(+10)=(+5), and right femur has (ER4)-(IR9)= (+4)-(-9)=

    (+13) to the horizontal. Therefore, the difference is 8 degrees.

     

    Incorrect Answers:

    Answer 2: Dynamization of the intramedullary nail is not appropriate as the left leg is shorter, and there is no mention of fracture gap distances or stability.

    Answers 3-5: Revision surgery would be indicated if rotational malalignment were >15 degrees and symptomatic.

     

     

     

    OrthoCash 2020

     

  17. Which of the following images shows an injury pattern most consistent with a lateral compression type 3 pelvic ring injury?

     

     

     

     

     

     

     

     

     

     

     

     

     

     

    1. Figure A

    2. Figure B

    3. Figure C

    4. Figure D

    5. Figure E

     

    Corrent answer: 3

     

    Figure C is an axial CT scan of a lateral compression type 3 (LC3) pelvic ring injury.

     

    Classically, LC3 injuries demonstrate an ipsilateral lateral compression and a contralateral APC (windswept pelvis) fracture pattern. The most common mechanism of injury in these cases is a rollover MVC or pedestrian vs. auto. LC1 injuries are characterized by an oblique or transverse ramus fracture and ipsilateral anterior sacral ala compression fracture, while LC2 injuries consist of a rami fracture and ipsilateral posterior ilium fracture dislocation (crescent fracture). While LC1 injuries can often initially be managed conservatively with protected weight-bearing and close observation, LC2 and LC3 pelvic ring injuries are almost universally operative.

    Pennal et al. discuss a radiologic technique for assessing the forces producing pelvic disruption and its use in logically classifying pelvic injury. Based on this radiologic assessment and along with some biomechanical studies, they propose a classification system involving three major forces producing injury that can also be helpful in the management of these patients.

     

    Young et al. performed a retrospective analysis of the plain radiographs of 142 cases of pelvic fractures and identified four patterns of force that presented with distinctive, recognizable radiographic appearances. They describe a classification system for pelvic fractures based on radiographic and clinical findings that correlates with associated injury to soft-tissue structures and enables the surgeon to begin corrective procedures rapidly.

     

    Incorrect Answers:

    Answer 1: This represents a lateral compression type 2 injury. Answer 2: This represents a lateral compression type 1 injury.

    Answer 4: This represents an anterior posterior compression type 2 injury. Answer 5: This represents an anterior posterior compression type 3 injury.

     

     

     

    OrthoCash 2020

     

  18. For a patient with an unstable pelvic fracture, the amount of blood tranfusions required in the first 24 hours has shown to be most predictive for what variable?

    1. Length of hospital stay

    2. Association with neurological deficit(s)

    3. Length of intensive care stay

    4. Cardiac collapse

    5. Mortality

     

    Corrent answer: 5

     

    Unstable pelvic fractures can be devastating injuries often resulting in significant morbidity and even death.

     

    According to the referenced study by Smith et al, fracture pattern and angiography/embolization were not predictive of mortality in patients with unstable pelvic injuries. The three factors they found to be predictive were: increased blood transfusions in the first 24 hours, age >60 years, and increased ISS or RTS scores. Deaths were most commonly from exsanguination (<24 hours) or multiorgan failure (>24 hours).

    Incorrect Answers: Choices 1-4 do not correlate with increased blood transfusions to the extent of Option 5.

     

     

     

    OrthoCash 2020

     

  19. A healthy 27 year-old-male is brought into the emergency department after a fall from height. He has a suspected left C8-T1 nerve injury. Which of the following findings would most suggest a root avulsion injury rather than a brachial plexus injury at this level?

    1. Reduced radial artery pulse

    2. Double break in the ipsilateral superior shoulder suspensory complex

    3. Elevated hemidiaphragm

    4. Musculocutaneous nerve deficit

    5. Drooping of the left eyelid Corrent answer: 5

    Drooping of the left eyelid is a presenting feature of Horner's syndrome. Horner's syndrome represents a disruption of the sympathetic chain via C8 and/or T1 root avulsion after trauma.

     

    Brachial plexus injuries are often classified as preganglionic vs. postganglionic injuries. Preganglionic injuries are typically avulsion injuries proximal to the dorsal root ganglion. Clinical features suggestive of lower root avulsion injury include a person falling from height clutching on object to save himself, Horner’s syndrome (drooping of the eyelid (ptosis), pupillary constriction (miosis) and anhidrosis), absence of a Tinel sign or tenderness to percussion in the neck, and a normal histamine test (C8-T1 sympathetic ganglion - intact triple response (redness, wheal, flare)).

     

    Caporrino et al. reviewed 102 patients to assess the best modality (e.g. physical examination, MRI and nerve conduction studies [NCSs]) for diagnosing and localizing brachial plexus injuries. They found the best diagnostic performance with physical examination (sensitivity = 97.8%; specificity = 30.8%) and NCSs (sensitivity = 98.9%; specificity = 23.1%). MRI had inferior performance for all measurements. They conclude that NCSs exhibited superior performance to MRI, and should be considered a more reliable supporting tool after detailed physical examination.

     

    Incorrect Answers:

    Answer 1, 2: The superior shoulder suspensory complex (SSSC) is located in close proximity to the brachial plexus. Disruption of the SSSC may affect the local anatomy and cause local brachial plexus or vascular injury.

    Answer 3: Elevated hemidiaphragm (Phrenic nerve C3,4,5) may be suggestive an upper brachial plexus root avulsion, e.g. C5, C6 injury.

    Answer 4: Musculocutaneous nerve deficiency (C5 deficiency) results in weakness to the biceps.

     

     

     

    OrthoCash 2020

     

  20. A 46-year-old competitive cyclist falls while racing and suffers an isolated fracture as seen in Figure A. He is positioned on a fracture table and a closed reduction maneuver is attempted, unsuccessfully.

    Which of the following treatment plans is most appropriate?

     

     

     

     

    1. Obtain a CT scan intraoperatively to reassess the quality of reduction

    2. Perform additional closed reduction maneuvers until the reduction is adequate for percutaneous fixation

    3. As long as the stepoff is less than 7mm in any plane of imaging, it is appropriate to proceed with percutaneous fixation

    4. Perform an open reduction and internal fixation using a sliding hip screw

    5. Perform an acute total hip arthroplasty through a direct anterior approach since the patient is on the fracture table

    Corrent answer: 4

     

    When standard closed reduction maneuvers using a traction table are unsuccessful, displaced femoral neck fracture in young adults (< 50 years old) should be open reduced prior to fixation.

     

    Displaced femoral neck fractures in young patients have created many treatment controversies that are ongoing in the literature due to devastating consequences of poor outcomes, including nonunion and osteonecrosis of the femoral head. Although poorly defined, the quality of reduction is associated with rates of both nonunion and osteonecrosis. A closed reduction that is malangulated (>10 degrees varus/valgus or anteversion/retroversion) or has

    significant displacement (5 mm or more in ANY view) is unacceptable, and an open reduction should be performed. Of course, this can be very difficult to assess in the operating room, where uncalibrated fluoroscopy with difficult to obtain tangential imaging is heavily relied upon to make this assessment.

    Accordingly, when the quality of closed reduction is questionable, the best treatment plan is to obtain a better reduction with direct visualization of the femoral neck prior to fixation.

     

    A systematic review of the literature by Pauyo, et al. cites numerous studies showing a higher incidence of osteonecrosis of the femoral head in patients with displaced femoral neck fractures treated with unsatisfactory reductions. Furthermore, performing multiple closed reduction attempts is also associated with a higher risk of osteonecrosis.

     

    Upadhyay et al. performed a randomized controlled trial of 102 patients with femoral neck fractures treated with closed or open reductions, which were randomized. The groups had similar rates of nonunions and osteonecrosis of the femoral head; however, subanalysis revealed a "poor" reduction was the highest predictor of poor outcome, whether the reduction was attempted open or closed. Interestingly, the quality of reduction was more important than the implant used or the timing of surgery (including surgeries performed > 48h after injury).

     

    Figure A shows a pre-operative AP x-ray of the patient's high-energy femoral neck fracture. Illustrations A and B are intraoperative fluoroscopic and postoperative CT scans of this same patient, highlighting that fluoroscopy may "hide" the degree of residual displacement.

     

    Incorrect Answers:

    1. While intriguing, this is not currently the standard of practice; additionally, the stem already states that the reduction is "unsuccessful"

    2. Repeated closed reduction maneuvers may further propagate comminution and damage the blood supply to the femoral head, in theory

    3. Any stepoff of 5 mm seen on x-ray is a marker of worse outcomes. Remember, tangential imaging of the femoral neck is difficult to obtain, and if 7 mm is seen, in actuality it may be a larger amount of displacement. Think of the femoral neck as a complex cylindrical tube of bone with asymmetric cortices (e.g. the calcar) - to obtain the perfect fluoroscopic image for measuring maximal displacement, a perfect perpendicular view to this displacement is required, which is very difficult to do before provisional fixation is placed.

      5. Total hip arthroplasty is regarded as a salvage procedure for treating a femoral neck fracture in a young adult patient

       

       

       

       

       

       

       

       

      OrthoCash 2020

       

  21. You are called to the neonatal intensive care unit to see a consult. The child was born 6 hours ago at 34 weeks of gestation. The pregnancy was complicated by oligohydramnios. The neonatologist is concerned about the appearance of his left forearm, shown in Figures A and B. Laboratory workup shows polycythemia. What is the diagnosis and next best step?

     

     

     

     

     

     

    1. Cellulitis; intravenous antibiotics

    2. Necrotizing fasciitis; debridement

    3. Necrotizing fasciitis; hyperbaric oxygen

    4. Neonatal compartment syndrome; needle manometry

    5. Neonatal compartment syndrome; fasciotomy Corrent answer: 5

    This child has neonatal forearm compartment syndrome. Fasciotomy is indicated.

    Neonatal compartment syndrome is rare. The diagnosis is often made retrospectively after complications have occurred. There is no obvious etiology although both extrinsic and intrinsic causes are thought to contribute. Extrinsic factors include mechanical compression from fetal posture, oligohydramnios, umbilical cord loops, amniotic band constriction, or direct birth trauma.

    Intrinsic factors include hypercoagulable state in the neonate leading to intraarterial/intravenous thrombosis. A skin lesion is pathognomonic. The lesions start at birth as bullous/ulcerative lesions with distal edema. Early treatment includes emergency fasciotomy.

     

    Ragland et al. reviewed neonatal forearm compartment syndrome in 24 patients. All presented with a sentinel forearm skin lesion (ranging form mild skin and subcutaneous lesions to distal tissue loss and fingertip gangrene).

    Only one patient had emergent fasciotomy - and only this child had a salvageable, functional limb. Missed diagnosis leads to Volkmann's contracture, muscle infarction, intrinsic paralysis and loss of hand sensibility.

     

    Allen et al. describe a case report of neonatal compartment syndrome initially thought to be amniotic band syndrome. They reiterate that it is often misdiagnosed. They emphasize that early diagnosis and fasciotomy may save the limb but unfortunately, in many cases, the timing and duration of initial insult and the extent of damage is unknown.

     

    Figures A and B show a swollen, erythematous forearm characteristic of neonatal forearm compartment syndrome. Illustration A shows fasciotomy on the same forearm. Illustration B shows late contracture. Illustration C shows another example of a skin lesion (an eschar). Illustration D shows late bony sequelae with shortening and premature closure of multiple metacarpals and phalanges and alteration in distal radial physis.

     

    Incorrect Answers:

    Answers 1, 2 and 3: The bullous appearance is more characteristic of compartment syndrome. The acuteness of the lesion and its presentation within a few hours of life make infection less likely. Necrotizing fasciitis in a neonate is not treated with hyperbaric oxygen.

    Answer 4: Needle manometry in a neonate is not performed. There are no standards for acceptable pressure gradients (delta value) in a neonate. A neonate’s diastolic pressure at birth is <40mmHg and a small increase in compartment pressure rapidly impairs muscle perfusion.

     

     

     

     

     

     

     

     

     

     

     

     

     

    OrthoCash 2020

     

  22. A 70-year-old male with longstanding diabetic neuropathy sustains a fall down a flight of stairs and sustains the injury shown in Figures A and B. In the operating room, direct reduction of the fracture is performed. The syndesmosis is assessed and found to be intact. The

    fibula is fixed with a small fragment locking plate and the medial malleolus is fixed with screws. What is the next best step?

     

     

     

     

     

    1. No syndesmotic fixation. Immediate touch-down weightbearing in CAM walker boot.

    2. No syndesmotic fixation. Non-weightbearing for 4 to 6 weeks, followed by progressive weightbearing in CAM walker boot.

    3. No syndesmotic fixation. Non-weightbearing for 8 to 12 weeks.

    4. Syndesmotic screws. Non-weightbearing for 4 to 8 weeks.

    5. Syndesmotic screws. Non-weightbearing for 8 to 12 weeks.

     

    Corrent answer: 5

     

    Ankle fractures in diabetics with neuropathy should be treated with enhanced fixation comprising stiff plates with syndesmotic screws, even in the absence of syndesmotic injury. Weightbearing should be delayed for 8 to 12 weeks after surgery, rather than 4-8 weeks (as for normal patients).

     

    Diabetics with ankle fractures are prone to complications. Nonoperatively treated cases have up to a 50% incidence of skin breakdown in a cast.

    Surgically treated patients have up to 40% complication rate. Supplemental fixation can include include multiple syndesmotic screws, spanning external fixation, tibio-talar Steinmann pins, more rigid fibular plates, supplemental intramedulary fibular pinning. Weightbearing after surgery should be delayed (8-12weeks).

     

    McCormack and Leith reviewed the complications in 26 diabetic ankle fractures. 19 patients had surgery. The incidence of complications was 42%. Two required amputation and died. None of the non-diabetic control group had complications. They concluded that in the older, diabetic patient with lower demands, especially if insulin-dependent, it may be preferable to accept a loss of reduction and malunion rather than risk the potentially devastating complications associated with operative intervention. If surgery is necessary, the results of this study provides a more accurate prognosis and allows for better discussion of surgical risks with the patient.

     

    Roseunbaum et al polled AOFAS members on the treatment of diabetic ankle fractures. (1) For nondisplaced bimalleolar fracture with diabetic neuropathy, respondents preferred casting and nonweightbearing (NWB) for 8-12 weeks.

    (2) For Weber B fracture dislocation without neuropathy, without syndesmotic injury, respondents preferred either 1/3 tubular or small fragment locking plates with syndesmotic screws and NWB for 8-12 weeks. (3) For bimalleolar fracture dislocation with neuropathy, without syndesmotic injury, respondents preferred small fragment locking plates with syndesmotic screws and NWB for 8-12 weeks.

     

    Wukich and Kline reviewed the management of ankle fractures in diabetics.

    They found that neuropathy is more prevalent in patients with ankle fractures than without. They stress that 1% reduction in HbA1C results in 30% reduction in complication rate. Fracture healing is slower, callus is smaller, with less stiffness, tensile strength and collagen content. Amputation rate is up to 5% for patients treated operatively or nonoperatively. Complications are higher in patients with vasculopathy, neuropathy, or Charcot arthropathy.

     

    Incorrect Answers:

    Answers 1, 2, 3: Supplemental syndesmotic fixation will increase construct rigidity.

    Answer 4: Extended NWB should be enforced after ORIF of diabetic ankle fractures (8-12wk).

     

     

     

     

     

     

    OrthoCash 2020

     

  23. In a statement put forth by AAOS, the role of the orthopaedic surgeon in the face of domestic and family abuse includes all of the following EXCEPT:

    1. Be aware that he or she may be the first physician to be caring for the victims

    2. Ensure that they maintain comprehensive and accurate medical records documenting the events and examinations

    3. Care should resume once the patient's social situation is evaluated and assessed

    4. Transfer an elderly victim who is in immediate danger to a hospital emergency department and notify the emergency department physician of the transfer and the reasons for your concern

    5. Advocate for appropriate legislation and public policy Corrent answer: 3

      Regardless of the social situation, the orthopaedic surgeon's primary role is to proceed and offer expeditious and appropriate care for the patient's injuries.

       

      The statement put forth by the AAOS implores the orthopaedic surgeon to be aware of the integral components to identify, document, and care for minors,

      elders, and/or partners who are victims of domestic abuse.

       

      Zilmer et al. in a JAAOS review and the basis for the AAOS statement emphasizes the ability to identify abuse, which includes, but is not limited to frequent/multiple injuries, temporal abnormalities in multiple injuries, frequent visits/utilization of the emergency department, and/or unusual injuries/fracture patterns not consistent with the clinical picture. Meticulous documentation is of paramount importance, in addition to communicating your concerns to the appropriate emergency department personnel.

       

      Incorrect answers:

      Answers 1, 2, 4 and 5: All are expected roles of the orthopaedic surgeon in the face of domestic and/or family abuse.

       

       

       

      OrthoCash 2020

       

  24. A 55-year-old woman slips on ice and sustains the injury shown in Figures A and B. She notes no other medical history, but has not seen a doctor in 2 decades. Routine preoperative testing reveals random blood sugar of 302mg/dL (normal, 70-110mg/dL) and glycated hemoglobin (HbA1c) of 10.7%. She undergoes operative treatment and postoperative radiographs are shown in Figures C and

    D. Postoperative management should include

     

     

     

     

     

     

     

     

    1. weight-bearing as tolerated in a cast on the first post-operative day

    2. weight-bearing as tolerated in a Controlled Ankle Movement (CAM) boot at 2 weeks, after sutures are removed

    3. protected weight-bearing in a cast at 2 weeks after syndesmotic screw removal to prevent stiffness

    4. protected weight-bearing in a CAM boot at 6 weeks

    5. protected weight-bearing in a CAM boot at 10 weeks Corrent answer: 5

    Operatively treated diabetic ankle fractures should undergo a prolonged period of non weight-bearing (up to 12 weeks) and protected weight-bearing (for another 6 weeks). Non-diabetic patients normally begin protected weightbearing at 4-8 weeks postoperatively.

     

    Diabetics with ankle fractures are prone to complications. Nonoperatively treated cases have up to a 50% incidence of skin breakdown in a cast.

    Surgically treated patients have up to 40% complication rate. Supplemental fixation can include include multiple syndesmotic screws, spanning external fixation, tibio-talar Steinmann pins, more rigid fibular plates, supplemental intramedulary fibular pinning. Weightbearing after surgery should be delayed (8-12weeks).

    Roseunbaum et al polled AOFAS members on the treatment of diabetic ankle fractures. (1) For nondisplaced bimalleolar fracture with diabetic neuropathy, respondents preferred casting and nonweightbearing (NWB) for 8-12 weeks.

    (2) For Weber B fracture dislocation without neuropathy, without syndesmotic injury, respondents preferred either 1/3 tubular or small fragment locking plates with syndesmotic screws and NWB for 8-12 weeks. (3) For bimalleolar fracture dislocation with neuropathy, without syndesmotic injury, respondents preferred small fragment locking plates with syndesmotic screws and NWB for 8-12 weeks.

     

    Wukich and Kline reviewed the management of ankle fractures in diabetics. They found that neuropathy is more prevalent in patients with ankle fractures than without. They stress that 1% reduction in HbA1C results in 30% reduction in complication rate. Fracture healing is slower, callus is smaller, with less stiffness, tensile strength and collagen content. Amputation rate is up to 5% for patients treated operatively or nonoperatively. Complications are higher in patients with vasculopathy, neuropathy, or Charcot arthropathy.

     

    Figures A and B show a bimalleolar ankle fracture dislocation with fibular comminution and syndesmotic disruption. Figures C and D show ORIF of the medial malleolus and plating of the fibula with multiple transtibial screws.

    Illustration A shows a CAM walker boot.

     

    Incorrect Answers:

    Answers 1-4: Early weight-bearing in diabetic patients after ankle fracture ORIF may lead to malunion/nonunion, wound breakdown, and infection.

    Prolonged non weight-bearing, followed by protected weight-bearing is prudent.

     

     

     

     

     

    OrthoCash 2020

     

  25. A 53-year-old patient underwent open reduction and fixation for the injury shown in Figures A through C. The operative report states the fracture was stabilized with anterolateral and posteromedial non-locking plates. Figure D is a lateral radiograph taken at the two-weeks followup appointment. What technical approach would have prevented this complication?

     

     

     

     

     

     

     

     

     

     

     

    1. Utility of a single anterolateral locking plate

    2. Utility of a single posteromedial locking plate

    3. Multiple lateral-to-medial rafting screws proximal to the anterolateral plate

    4. Supplementary fixation of the posterolateral fragment

    5. Supplementary fixation with a calcium cement Corrent answer: 4

    Figure A to C show a bicondylar proximal tibial fracture with significant posterolateral plateau comminution. Figure D shows the posterolateral

    fragment was not fixed properly, and the sagittal malalignment persisted. This complication would have been prevented if supplementary fixation of the posterolateral fragment would have been performed.

     

    The treatment of complex tibial plateau fractures remains an operative challenge. Traditionally, these fractures were evaluated and classified using the Schatzker or OTA classification using coronal imaging and, as a result, most surgeons direct their fixation methods toward medial and lateral plates without consideration of posterior fragment fixation. Posterolateral and posteromedial fragments are common with complex plateau fractures. Failure to appreciate their significance can result in fracture malreduction and postoperative complications.

     

    Sohn et al. retrospectively reviewed 278 complex tibial plateau fractures. Using CT scan images, the incidence of posterolateral fragments included 84 cases (44.2%). They found that these inverted conical shaped fragments usually occupy approx. one-third of the surface area of the lateral tibial plateau. They conclude that these fragments should be addressed with supplementary fixation methods, such as posterolateral buttress plates or raft screws.

     

    Figure A is a AP radiograph of the knee showing a bicondylar tibial plateau fracture. The lateral plateau is depressed and widened. There is an associated proximal tibial fracture suggestive of a high energy injury. Figure B is an axial CT scan image of the proximal tibia. PFCA = proximal femoral condyle angle. Figure C is a sagittal CT scan image of the knee with extensive comminution. Note the posterolateral fragment which demonstrates the inverted conical shape. Figure D shows a lateral radiograph of the knee. The posterior plateau has a double shadow, which represents malreduction of the posterolateral fragment.

     

    Incorrect Answers:

    Answer 1,2: Fixed angle locking plates are a good choice for osteoporotic bone or comminuted fractures in the proximal tibia. However, a single plate would not be the appropriate fixation technique for this bicondylar fracture pattern as it would not address the posterior tibial plateau fragments.

    Answer 3: Multiple lateral-to-medial rafting screws proximal to the anterolateral plate may be used. However, this would also not address the posterolateral fragment.

    Answer 5: Supplementary fixation with a calcium cement is utilized for articular depression and would not help the shear forces seen in this injury.

     

     

     

     

     

    OrthoCash 2020

     

  26. You are planning open reduction and internal fixation for a comminuted radial head fracture. To avoid impingement with the proximal ulna, you need to carefully place your fixation. What percent of the proximal radial head articulates with the proximal ulna?

    1. 0%

    2. 75%

    3. 50%

    1. 25%

    2. 100%

     

    Corrent answer: 2

     

    Only 75% of the radial head articulates with the ulna. The remaining 25% (approximately 90 degree arc) which does not articulate is considered the "safe zone" and is important for placement of fixation.

     

    Caputo et al looked at 24 cadevaric arms and found the average arc of the nonarticulating radial head was 113 degrees (range, 106 degrees to 120 degrees; standard deviation, 4 degrees). They found the nonarticulating portion of the radial head (or safe zone for prominent fixation) consistently encompassed a 90 degree angle localized by palpation of the radial styloid and Lister's tubercle.

     

    Smith et al in a similar cadaveric study identified a 110 degree safe zone based

    on cross-sectional dissections and landmarks. In this regard they were able to show this 110 degree arc of safe articulation in the proximal radius.

     

    A "safe zone" of approximately 110 degrees of radial head surface was first identified by cross-sectional anatomic dissections.

     

    Cohen et al reviewed the classification, diagnosis, and management of acute elbow dislocations and specifically discuss treatment of associated proximal radial fracture where fixation needs to be applied in the "safe zone."

     

    Illustration A depicts the radial head safe zone during forearm pronation, supination, and neutral positions.

     

     

     

     

     

     

    OrthoCash 2020

     

  27. A 34-year-old male arrives intubated with a closed head injury to the trauma bay after a motor vehicle accident. After initial hospital workup and resuscitation, he is transferred to the intensive care unit. In addition to multiple systemic injuries, he sustained the closed injury shown in Figure A. Intracompartmental pressure monitoring of the limb measure in a range from 28-30 mm Hg. Which of the following sustained blood pressure measurements would support the treatment of limb fasciotomy?

     

     

     

    1. 110/60 mmHg

    2. 115/55 mmHg

    3. 92/64 mmHg

    4. both a and b

    5. a, b and c Corrent answer: 2

    A reported indication to perform fasciotomy includes an ICP measurement that is elevated to 30 mm Hg below the diastolic blood pressure. This would be the case if this patient's blood pressure was consistently around 115/55 mmHg (dBp=55; ICP=30; delta p = dBp-ICP = 25 mmHg).

     

    Given the poor outcomes associated with missed compartment syndromes, it is important to obtain both clinical and objective data when determining if a patient needs fasciotomies. A clinical assessment is the diagnostic cornerstone of acute compartment syndrome. However, the intracompartmental pressure measurement has been advocated to help confirm the diagnosis in patients where there remains uncertainty after clinical exam - especially with intubated patients. An absolute compartment pressure >30 mm Hg or a difference in diastolic pressure and compartments pressure (delta p) <30 mm Hg may help to confirm the necessity for fasciotomy.

    McQueen et al. prospectively reviewed 116 patients with tibia fractures that had continuous monitoring of their anterior compartment for 24 hours. They found that using an absolute pressure of 30 mmHg would have resulted in 50 patients (43%) treated with unnecessary fasciotomies. They conclude using a differential pressure of 30 mmHg is a more reliable indicator of compartment syndrome.

     

    Olson et al. provide a review of compartment syndrome for the lower extremity. They discuss a variety of injuries and medical conditions that may initiate acute compartment syndrome, including fractures, bleeding disorders, and other trauma. Although the diagnosis is primarily a clinical one, they also recommend supplementation with compartment pressure measurements in equivocal cases.

    Figure A shows a closed comminuted tibial shaft fracture. Incorrect Answers:

    Answer 1: dBp=60; ICP=30; dBp-ICP = 30 mmHg. Therefore delta is not <30,

    which would not definitely prove to be compartment syndrome. Both the absolute and delta p measurements are not greater, and less than, the published parameters.

    Answer 3: dBp=64; ICP=30; dBp-ICP = 34 mmHg. Therefore delta is not <30

     

     

     

     

    OrthoCash 2020

     

  28. A 30 year-old male is involved in a motor vehicle collision and sustains a scapular fracture. In patients with scapular fractures, what other fracture is MOST commonly observed?

    1. Spine fracture

    2. Rib fracture

    3. Clavicle fracture

    4. Humerus fracture

    5. Pelvic fracture

     

    Corrent answer: 2

     

    Rib fractures are the most commonly observed fractures associated with scapular fractures.

     

    Scapular fractures are associated with high-energy trauma, with motor vehicle collisions being a common mechanism of injury. These fractures are also

    associated with increased Injury Severity Scores and therefore clinicians need to be cognizant of concomitant injuries. In addition to associated orthopaedic injuries, pulmonary injuries (contusions, hemothorax, and pneumothorax) and head injuries are associated with scapular fractures. A CT chest should be considered in patients with scapula fractures, to best visualize associated rib fractures and pulmonary injuries.

     

    Baldwin et al. conducted a retrospective case control analysis using the US National Trauma Database that included 9,453 scapular fractures. In their study, the most commonly associated fractures were rib fractures (52.9%), followed by fractures of the spine (29.2%), clavicle (25.2%), and pelvis (15.3%). Lung and head injuries occurred in 47.1% and 39.1% of the cases, respectively.

     

    Incorrect Answers:

    Answers 1, 3, 4, 5: Although these fractures do occur in association with scapular fractures, Baldwin et al demonstrated that they do not occur as frequently as rib fractures.

     

     

     

    OrthoCash 2020

     

  29. A 72-year-old woman falls onto her left hip after tripping over a curb during her daily 3-mile walk. An injury radiograph is shown in Figure A. What is the best long term solution?

     

     

     

    1. Cannulated screws

    2. Valgus intertrochanteric osteotomy

    3. Unipolar hemiarthroplasty

    4. Bipolar hemiarthroplasty

    5. Total hip arthroplasty Corrent answer: 5

    THA is the best long term solution for displaced femoral neck fractures (FNF) in active elderly patients.

     

    The aims of surgery for FNF in elderly patients are immediate pain relief, rapid mobilization, and low complications and revision. THA has best pain relief, fewer reoperations, best survivorship and is most cost-effective but has longer operative/anesthetic time, blood loss, higher infection rate, and potential instability compared with HA.

     

    Healy and Iorio examined the optimal treatment for elderly FNF. They compared internal fixation (120 patients) with arthroplasty (HA, 43 patients; THA, 23 patients). There was no different in reoperation or mortality rates between the 2 groups, but arthroplasty was more cost effective, had independent living, and longer interval to reoperation/death. THA had less pain, better function, and lower rates of reoperation than HA, and was most cost-effective. They concluded that THA was the best treatment.

     

    Yu et al. performed a meta-analysis of randomized controlled trials to determine whether THA or hemiarthroplasty (HA) was superior. They found that THA had lower risk of reoperation (RR = 0.53), higher risk of dislocation (RR = 1.99), and higher functional scores at 1 and 4 years. There was no difference in mortality, infection and complication rates.

    Figure A shows a displaced left femoral neck fracture. Incorrect Answers:

    Answer 1: Cannulated screws may be indicated for nondisplaced fractures

    (CRIF) or displaced fractures (ORIF) in young patients. But internal fixation is not predictable/ reproducible in all patients (especially elderly patients) because of variations in the condition of articular cartilage, bone stock, fracture angle, comminution, reduction, fixation, comorbidities, nonunion, osteonecrosis, reoperation, mortality, and postoperative rehabilitation.

    Answer 2: Valgus intertrochanteric osteotomy (or sliding hip screw) is indicated for displaced fractures with high Pauwel's angle in young patients where it is desirable to avoid arthroplasty.

    Answer 3: Unipolar HA is reserved for low demand elderly patients, and is

    associated with groin pain, hip stiffness, cartilage erosion from oversized / undersized femoral head and protrusio.

    Answer 4: Bipolar HA may be indicated for patients at risk for instability (neuromuscular/Parkinson’s disease, dementia). It is is associated with groin pain, acetabular cartilage deterioration, osteolysis from PE wear at the femoral head/mobile acetabular component, implant loosening, and early revision surgery.

     

     

     

    OrthoCash 2020

     

  30. A 41-year-old male underwent intramedullary nailing for a low-energy left femoral shaft fracture. At his follow-up appointment, he complains that his feet are pointing in opposite directions when walking. Using the imaging study shown in Figure A, which of the following represents this patient's left femur malalignment?

     

     

     

     

    1. Internal rotation malalignment of 44 degrees

    2. External rotation malalignment of 44 degrees

    3. Internal rotation malalignment of 21 degrees

    4. External rotation malalignment of 21 degrees

    5. Internal rotation malalignment of 63 degrees

    Corrent answer: 3

     

    Figure A shows axial CT scan slices of the pelvis and knee. On the operative left side, there is an internal rotation malalignment of 21° compared to the contralateral side (44°-23°=21°).

     

    Radiographic rotational malalignment after fixation of femoral shaft fractures may be measured by comparing the femoral anteversion of both femurs. This can be determined by measuring the angle between a line tangential to the dorsal bony contours of the femoral condyles and a line drawn through the axis of the femoral neck. Rotational differences of less than 10° are considered variations of normal.

     

    Jaarsma et al. reviewed rotational malalignment after intramedullary nailing of femoral fractures. They report that rotational measurements by CT are superior to clinical assessment. They note a high incidence of malrotation after IM nailing of fractures. This has shown to be in the range 15% to 30%.

     

    Figure A shows left femoral malrotation using CT-torsion measurements with axial cuts of the femoral neck and distal femoral condyles. Note the normal anteversion of the right femur (23 degrees; normal range 10-25).

     

    Incorrect Answers:

    Answer 1,2,4,5: To measure malrotation using axial CT cuts, the normal side is considered as neutral = 0 malrotation. For this patient, that correlates with 23 degrees of anteversion. If the femoral anteversion is increased to 44 degrees, this will mean a 21 degree increase in femoral anteversion from neutral (0 + [44-23] = +21). The opposite would occur if the femoral anteversion decreased = external rotation.

     

     

     

    OrthoCash 2020

     

  31. A fracture in the following location is most commonly associated with procurvatum and valgus malalignment?

    1. Humeral shaft

    2. Distal femur

    3. Proximal tibia

    4. Distal third tibia shaft with intact fibula

    5. Subtrochanteric proximal femur Corrent answer: 3

    Fractures of the proximal tibial shaft are associated with high rates of valgus and apex anterior (procurvatum) malalignment.

     

    Proximal third tibial shaft fractures are often difficult to reduce anatomically due to the tendency for both valgus and flexion deformity at the fracture site. Many different techniques have been devised to overcome the deforming forces. These include (1) Poller blocking screws posterior and lateral to the intramedullary nail (IMN), (2) utilizing a semiextended knee position during IMN of proximal tibia fractures (3) use of a suprapatellar approach for IMN, (4) usage of a slightly more lateral starting point during conventional IMN, and (5) application of unicortical plate.

     

    Ricci et al. describe the technique and results of using blocking screws and intramedullary nails to treat patients with fractures of the proximal third of the tibia. Post-operatively, all patients in their series had less than 5 degrees of angular deformity in the planes in which blocking screws were used to control alignment. At 6 months follow-up, 10/11 patients maintained this alignment.

     

    Illustration A shows intra-operative sagittal radiographs of the proximal tibia. Note the use of Poller blocking screws in the posterior and lateral aspects of the proximal tibia.

     

    Incorrect Answers:

    Answer 1: The most common malalignment with a humeral shaft fractures is varus, shortening and internal rotation.

    Answer 2: The most common malalignment with a distal femur fracture is varus, shortening and recurvatum.

    Answer 4: The most common malalignment with a distal third tibia shaft + intact fibula is varus and internal/external rotation.

    Answer 5: The most common malalignment with subtrochanteric proximal femur fracture is flexion, abduction and external rotation.

     

     

     

     

     

     

    OrthoCash 2020

  32. A 31-year-old male sustains a left elbow fracture. A laterally based incision that elevates the common extensor tendons and capsule anteriorly off the lateral column, would be the best approach for which of the following fracture patterns?

    1. Extra-articular distal humerus fracture with metaphyseal comminution

    2. Highly comminuted, proximal third radial shaft fracture

    3. Undisplaced, simple radial head fracture

    4. Complete coronal shear capitellum fracture

    5. Transolecranon anterior fracture-dislocation Corrent answer: 4

    A lateral extensile approach is preferred for exposure and surgical fixation of coronal shear capitellum fractures.

     

    A lateral extensile approach allows for visualization of capitellum and trochlea proximally, as well as radial head and neck pathology distally. These structures can be approached by developing the interval between the triceps muscle posteriorly, and the brachioradialis and extensor carpi radialis longus anteriorly. The dissection may be continued distally between the extensor carpi ulnaris and the anconeus muscles to access the radial head and neck.

     

    Ruchelsman et al. reviewed coronal shear capitellum fractures. They state that the forearm should be pronated to move the radial nerve away from the surgical field during the lateral extensile approach. In this approach, the common origin of the radial wrist extensors, with the anterior capsule, should be sharply elevated as a full-thickness sleeve from the lateral supracondylar ridge.

     

    McKee et al. identified a unique shear fracture pattern of the distal articular surface of the humerus in six patients that involved the capitellum and a portion of the trochlea. They termed this injury as the 'double-arc sign', based on a characteristic lateral radiographic abnormality seen with this fracture pattern. This injury pattern is also called the McKee modification (Type IV) of the Bryan and Morrey Classification of capitellum fractures.

     

    Illustration A shows the "double arc sign" as described originally by Dr McKee. Illustration B shows the lateral extensile approach to the elbow with an illustration (left) and cadaveric (right) labeled image.

     

    Incorrect Answers:

    Answer 1: Extra-articular distal humerus fractures with metaphyseal comminution would be best approached with a posterior incision and triceps

    splitting or sparing deep dissection.

    Answer 2: Highly comminuted, proximal third radial shaft fracture would be best approached with a volar or dorsal approach to the forearm (e.g., Henry or Thompson approach).

    Answer 3: Undisplaced, simple radial head fracture should be treated nonoperatively.

    Answer 5: Transolecranon anterior fracture-dislocations are best treated with a posterior approach to the elbow. Associated fractures can be reduced and fixed by working through the olecranon fracture.

     

     

     

     

     

     

     

     

     

    OrthoCash 2020

     

  33. Which of the following post-reduction forearm fractures patterns may be treated non-operatively in an otherwise healthy 22-year old male?

    1. Displaced diaphyseal fracture of the radius

    2. Non-displaced diaphyseal fracture of the radius, displaced diaphyseal fracture of the ulna

    3. Displaced diaphyseal fractures of both bones of the forearm, with less than

      10 degrees angulation after closed reduction

    4. An isolated mid-shaft ulna fracture translated 20%, with less than 5 degrees of angulation

    5. Gustilo grade II open fracture of the radius Corrent answer: 4

    In adults, minimally displaced fractures of the ulna may be treated nonoperatively.

     

    Even in the setting of minimal displacement, fractures involving the radial diaphysis, or both bones of the forearm, are at high risk of displacing further and progressing to malunion or nonunion. Given the potential for a resulting loss of forearm rotation, open reduction internal fixation is indicated for almost all adult diaphyseal radius and both bone fractures.

     

    Schulte et al. review the management of both bone forearm fractures in adults. They review biomechanics, fixation techniques, outcomes and complications.

    They note that the goals of fixation in simple patterns are 'cortical opposition, compression, and restoration of forearm geometry.'

     

    Anderson et al. treated 330 acute diaphyseal forearm fractures with compression plating from 1960 to 1970. At 4 months to 9 years follow up, they achieved a 97.9% union rate for the radius and 96.3% union rate for the ulna.

     

    Illustration A shows measurement of radial bow. A dotted line perpendicular to the line drawn from the radial tuberosity to the ulnar aspect of the distal radius can be used to measure radial bow when drawn at the point of maximum distance to the ulnar edge of the radius.

     

    Incorrect answers:

    Answers 1, 5. Any diaphyseal fracture of the radius in an adult warrants internal fixation, even if non-displaced.

    Answers 2, 3. Any diaphyseal fracture of both bones of the forearm should be treated with internal fixation in adults.

     

     

     

     

     

    OrthoCash 2020

     

  34. Which of the following treatments of an oligotrophic supracondylar femoral nonunion has been shown to have the best outcome?

    1. Retrograde femoral nailing with adjunct BMP-4

    2. Hybrid external fixation with adjunct BMP-4

    3. Usage of a percutaneous locking plate with adjunct BMP-3

    4. Open reduction and plating with autograft

    5. Open reduction and plating with adjunct calcium phosphate Corrent answer: 4

    Oligotrophic femoral supracondylar nonunions have been shown to be best treated with open reduction and plating (revision if previous surgery) and usage of autologous bone grafting.

     

    The first study by Bellabarba et al is a case series of 100% union rate (n=19) of supracondylar femoral nonunions treated with revision ORIF and autografting.

    The second study by Chapman et al is a case series of 100% union rate (n=18) of supracondylar nonunions treated with single or double plating and autograft.

     

    The referenced study by Rodriguez et al is an excellent review of the general principles and concepts of nonunions and their treatment.

     

     

     

    OrthoCash 2020

     

  35. When considering treatment options and their associated complications for a healthy adult with an isolated, completely displaced midshaft clavicle fracture, initial open reduction and internal fixation compared to nonoperative treatment with a sling leads to:

    1. Decreased nonunion rates and decreased healthcare costs

    2. Decreased nonunion rates and similar healthcare costs

    3. Decreased nonunion rates and increased healthcare costs

    4. Similar nonunion rates and decreased healthcare costs

    5. Similar nonunion rates and increased healthcare costs Corrent answer: 3

      In healthy adults with a completely displaced midshaft clavicle fracture, initial open reduction and internal fixation (ORIF) leads to decreased nonunion rates. However, the costs associated with this treatment and its possible complications are significantly greater than the cost of initial nonoperative treatment with a sling and delayed surgery if necessary.

       

      Midshaft clavicle fractures represent 80-85% of all clavicle fractures and often occur as result of direct trauma or fall on an outstretched arm. Treatment of this injury with primary ORIF versus nonoperative care with a sling is a controversial topic with respect to patient-reported functional outcomes. The current literature reports higher rates of nonunion in those managed nonoperatively. Economic evaluation of these treatments, using data from randomized controlled trials (RCTs), has demonstrated that ORIF is more costly compared to nonoperative treatment. The reported cost difference, accounts for both the costs of the initial treatment and costs incurred as a result of the treatment of complications, which includes delayed surgery rates for those initially managed non-operatively and re-operation rates for those initially managed with ORIF.

       

      Robinson et al. conducted a mullticenter, single blinded, RCT with 200 patients comparing primary ORIF versus nonoperative treatment. The rate of nonunion was significantly reduced with ORIF (relative risk = 0.07; p = 0.007). Although patient-reported functional outcomes appeared to be superior in those

      undergoing primary ORIF, this effect vanished when those with nonunions were removed from the analysis. Costs were significantly greater for those treated with primary ORIF (p < 0.0001).

       

      Rehn et al. performed a systematic review of RCTs comparing operative versus nonoperative treatment for displaced midshaft clavicle fractures. They found that ORIF leads to fewer nonunions but more minor complications compared nonoperative treatment. Additionally, they concluded that the effect of ORIF on functional outcomes remains controversial.

       

      Walton et al. utilized data from four RCTs to conduct a decision analysis with respect to costs (from the perspective of a single payer: adjusted 2013 Medicare rates) for ORIF versus nonoperative management. Reoperation and delayed surgery for those treated with ORIF and those treated nonoperatively, respectively, were defined as the end-points. The expected cost for ORIF was

      $14,763.21 compared with $3,112.65 for nonoperative treatment, yielding a cost savings of $11,650.56 for nonoperative treatment.

       

      Incorrect Answers:

      Answer 1: ORIF leads to increased, not decreased healthcare costs compared with nonoperative treatment.

      Answer 2: ORIF leads to increased, not similar healthcare costs compared with nonoperative treatment.

      Answer 4: ORIF leads to decreased, not similar nonunion rates and increased not decreased costs compared with nonoperative care.

      Answer 5: ORIF leads to decreased, not similar nonunion rates compared with nonoperative care.

       

       

       

      OrthoCash 2020

       

  36. A 32-year-old female sustained the injury seen in Figure A after a motor vehicle accident. On physical exam there was obvious deformity about the arm with a laceration that probed to bone over the lateral aspect of the arm. The patient was neurovascularly intact. She was treated with an intramedullary nail. Which of the following is true?

     

     

     

    1. Intramedullary nailing is associated with an increased range of motion postoperatively when compared to compression plating.

    2. There is a lower risk of iatrogenic radial nerve palsy with anterograde intramedullary nailing when compared to compression plating.

    3. There is a lower risk of nonunion with antegrade intramedullary nailing when compared to compression plating.

    4. The musculocutaneous nerve is at risk with lateral to medial distal locking screw placement

    5. There is a higher risk of shoulder impingement with antegrade nailing compared to compression plating.

    Corrent answer: 5

     

    When compared to compression plating, anterograde intramedullary nailing results show increased risk for shoulder impingement.

     

    Options for operative management of humeral shaft fractures mainly consist of intramedullary nail or plate and screw constructs. The main advantage to intramedullary nailing is when the soft tissue envelope makes a large incision undesireable or the fracture pattern dictates a relative stability construct - such as segmental or massively comminuted injuries. The disadvantages include trauma to the rotator cuff, post operative shoulder pain, indirect reduction

    leading to increased risk of malrotation, and increased reoperation for implant removal.

     

    Li et al. performed a randomized controlled trial with 45 patients that investigated the difference in post operative malrotation and functional outcomes when comparing intramedullary nails versus open reduction and internal fixation. They concluded that when comparing the two operative options, patients who underwent intramedullary nailing had a greater degree of malrotation, which was associated with decreased range of motion.

    Additionally, they found lower functional scores with patients who underwent intramedullary nailing.

     

    Kurup et al. performed a systematic review comparing outcomes between compression plating and intrameduallary nailing for operative treatment of humeral shaft fractures. With a total of 260 patients, they found no difference with blood loss, fracture union, iatrogenic radial nerve palsy, iatrogenic fracture comminution, elbow impingement, return to pre-injury occupation, and functional shoulder scores. They did show a statistically significant increase in shoulder impingement and reduction of range of motion when using an intramedullary nail.

     

    Figure A is a AP radiograph of a comminuted humeral shaft fracture. Illustration A is a radiograph of a humerus fixed with an intramedullary nail. Illustration B is a radiograph of a humerus fixed with a compression plate.

     

    Incorrect Answers:

    Answer 1,2,3: There is no difference between compression plating and intramedullary nailing in regards to risk of iatrogenic radial nerve palsy, and nonunion. Intramedullary nailing is associated with an increased risk of malrotation and subsequent decreased range of motion post-operatively.

    Answer 4: The musculocutaneous nerve is at risk with anterior to posterior distal locking screw placement. The radial nerve is at risk with lateral to medial screw placement.

     

     

     

     

     

    OrthoCash 2020

     

  37. A 32-year-old male is shot by a .38 special round during a drive-by shooting and is brought into the emergency room by emergency medical services. He lies in the trauma bay, unwilling to move and complaining of right hip pain. There is an entry wound in his left flank but no exit wound. Bowel sounds are absent and there is rebound tenderness in the right lower quadrant. Focused assessment with sonography for trauma is positive. A pelvic radiograph is shown in Figure A. What is the next best step?

     

     

     

    1. Diagnostic peritoneal lavage

    2. CT scan of the abdomen with extension to the hip joints to determine intra-

      / extra-articular location of the bullet

    3. Saline load test

    4. Arthroscopic irrigation and foreign body removal

    5. Laparotomy

     

    Corrent answer: 5

     

    This patient has (1) penetrating abdominal trauma (entry bullet wound, no exit wound), (2) signs of peritonism (which alone is indication for laparotomy) and

    (3) positive tests for hemoperitoneum on FAST. Laparotomy takes precedence over extraction of intra-articular bullet fragments.

     

    Gunshot wounds (GSW) to the hip are difficult injuries to treat. This is because of surrounding vasculature and the joint’s proximity to intrapelvic structures.

    Surgical treatment focuses on stabilization, assessment of neurovascular function, and fracture fixation. Recovery of the bullet is a secondary priority. Retained bullets may lead to infection, foreign body reaction, mechanical damage of the joint cartilage, proliferative synovitis, and plumbism. The trajectory of the bullet is also important: A bullet that enters directly into the hip joint is unlikely to lead to an infection, but an trans-abdominal trajectory represents a high risk for infection.

     

    Bartkiw et al. reviewed 2808 GSWs and found 1235 associated fractures including 42 fractures of the hip and pelvis. Ten orthopaedic operative

    procedures were performed in 7 patients. Associated nonorthopaedic injuries included 15 small/large bowel perforations (36%), 7 vessel lacerations (17%), and 2 urogenital injuries (5%) that required surgery. They recommend orthopedic intervention for intra-articular projectiles or bone fragments, and reconstruction of the hip and acetabulum.

     

    Najibi et al. reviewed 39 GSW to acetabulum. They found 32 simple and 7 associated fracture patterns. The most common simple and associated patterns were anterior column and both column, respectively. Bowel injuries were the most common associated injures (42%). Predictors of poor outcome include high-velocity missile, involvement of acetabular dome, abdominal injury, nerve injury, vascular injury, and male gender. Deep infection was associated with primary anastomosis of bowel injury and an associated fracture pattern.

    Figure A shows an intra-articular bullet in the right hip. Incorrect Answers:

    Answer 1: Diagnostic peritoneal lavage is not necessary as FAST is already

    positive.

    Answer 2: A CT scan can confirm FAST findings. But with a positive FAST and peritonism in the presence of penetrating trauma, laparotomy is indicated, even without a CT scan

    Answer 3: Saline load test of the hip is not indicated

    Answer 4: Removal of the bullet is not a priority at this point.

     

     

     

    OrthoCash 2020

     

  38. To be most effective at preventing post-operative sagittal plane deformity, poller blocking screws should be placed at which location labeled A-E in Figure A?

     

     

     

    1. A

    2. B

    3. C

    4. D

    5. E

     

    Corrent answer: 3

     

    Figure A shows an extra-articular proximal tibial fracture demonstrating the most common sagittal deformity (procurvatum) observed with these fractures. The most appropriate position for blocking screws would be medial-lateral screws placed on the concave side of the deformity and posterior to the nail.

    This corresponds to Label C.

     

    Poller blocking screws can be used during intramedullary nailing of proximal tibial fractures in an effort to prevent a procurvatum deformity. They are typically placed lateral and posterior relative to the nail trajectory in the proximal segment. Other modalities used to prevent this sagittal deformity includes a semiextended position of the knee for nail insertion, unicortical plating, and joystick manipulation of the proximal fracture segment.

     

    Stedtfeld et al. describes the mechanical model for blocking screws. They state

    that two or more blocking screws can be placed in each plane (AP and lateral) alongside the nail to create a narrow canal for the nail. This allows for multiple points of fixation and realignment of the bone as the nail is passed.

     

    Stinner et al. described the techniques for intramedullary nailing of proximal tibia fractures. They state in addition to the techniques described above, nail selection with distal Herzog bends may accentuate a sagittal plane deformity. This occurs because the the distal Herzog bend contacts the posterior cortex, which can create a wedge effect and translate the proximal segment anteriorly.

     

    Illustration A shows the post-operative radiograph of an extra-articular proximal tibial fracture in Figure A, which was treated with IM nailing. Note the two posterior Poller blocking screws used in the proximal fragment to prevent procurvatum of the fracture. The positioning of these screws corresponds to Label C in Figure A.

     

    Incorrect Answers:

    Answer 1: This screw placement would be too proximal in the fracture fragment to act as an artificial cortex. This blocking screw is placed slightly posterior to the midline, from medial to lateral, in the proximal fragment. Answers 2,4: These screws would either block the nail from passing down the canal, or significantly worsen the procurvatum deformity.

    Answer 5: Position E may be used for the placement of a distal Schanz pin in the posterior malleolus. This would be used if a femoral distractor or an external fixation frame is being used as an adjunct for fracture reduction and fixation.

     

     

     

     

     

    OrthoCash 2020

     

  39. You are treating a comminuted, apex-volar angulated distal radius fracture with an external fixator. Tightening the fixator with volar translation of the lunate will:

    1. Help restore appropriate volar tilt

    2. Maintain radial height and inclination

    3. Decrease the risk of non-union

    4. Increase risk of non-union

    5. Increase diastasis of intraarticular fracture fragments Corrent answer: 1

      Maintaining volar translation of the lunate after reduction of a comminuted distal radius fracture is recommended to help restore volar tilt.

       

      Closed reduction and maintenance of alignment of a comminuted distal radius fracture with an external fixator has been shown to produce good healing and good functional outcomes. Traction across the fracture is used to help maintain radial length and alignment and bring fracture fragments together with

      ligamentotaxis. Volar translation of the lunate is used to help restore volar tilt.

       

      Agee presents a review of distal radius fracture treatment with multiplanar ligamentotaxis using an external fixator. He notes that uniplanar ligamentotaxis, such as straight traction, typically does not restore volar tilt. He recommends also translating the carpus volarly, thus moving the lunate into a more volar position to achieve multiplanar ligamentotaxis and improve final alignment.

       

      Agee et al. presents a review of distal radius fractures treated in an external fixator. He notes that the common position of reduction involves holding the wrist flexed and in ulnar deviation. He points out that this position is not functional and leads to significant stiffness of the fingers. They recommend holding the wrist in a neutral to extended position and allowing early range of motion of the fingers. They noted better functional outcomes, and even had restoration of volar tilt in 55% of cases despite the extended wrist position.

       

      Incorrect answers:

      Answer 2: Radial height and inclination are maintained with traction and ulnar deviation of the wrist.

      Answers 3 and 4: Volar translation of the carpus has not been shown to increase or decrease the risk of non-union in distal radius fractures.

      Answer 5: Volar translation of the lunate has not been shown to increase displacement amongst intraarticualr fracture fragments in distal radius fractures.

       

       

       

      OrthoCash 2020

       

  40. 71-year-old woman with insulin-dependent diabetes mellitus and peripheral neuropathy has consented to undergo operative fixation of the injury shown in Figure A. The surgical plan is to utilize a small fragment plate and screws for fracture fixation. Which of the following treatment adjuncts will help to improve fracture stability/union in this patient?

     

     

     

    1. Open deltoid repair

    2. Lateral bridge plating

    3. Retrograde trans calcaneal-talartibial pins

    4. Denosumab administration

    5. Above knee immobilization Corrent answer: 3

    Retrograde trans calcaneal-talartibial pins is a fixation adjunct that has shown to increase fracture stability/union in the diabetic ankle fracture population.

     

    Unstable ankle fractures in diabetics are typically fixed with rigid plates and supplemental treatment modalities due to prolonged healing time, poor vascularity and high incidence of hardware failure in this population.

    Supplemental treatment options include longer and more rigid plating, multiple transtibial/trans-syndesmotic screws, addition of an external fixator frame, retrograde trans calcaneal-talartibial pins, and prolonged protected weightbearing protocols after surgery.

     

    Wukich et al. reviewed the management of ankle fractures in patients with diabetes mellitus. They state that diabetic patients have higher complication rates following both open and closed management of ankle fractures. They

    conclude that prolonged non-weight-bearing and subsequently protected weight-bearing are recommended following both operative and nonoperative management of ankle fractures in patients with diabetes.

     

    Jani et al. designed a protocol for treatment of diabetic patients with unstable ankle fractures. The treatment protocol included: 1) retrograde transcalcaneal-talar-tibial fixation using large Steinmann pins or screws in conjunction with standard techniques of open reduction and internal fixation; 2) casting and non-weightbearing status for 12 weeks; 3) removal of the intramedullary

    implants between 12 and 16 weeks; 4) application of a walker boot or short leg cast with partial weightbearing for an additional 12 weeks; and 5) transition to a custom-molded ankle-foot orthosis (AFO) or custom total-contact inserts in appropriate diabetic footwear.

     

    Perry et al. describe the outcomes of salvage fixation for failed neuropathic ankle fractures, using large-fragment fibular plating and multiple syndesmotic screws. The construct used was a laterally placed 4.5 DCP plate on the fibula with multiple 4.5-mm syndesmotic screws. They showed that this approach was a successful salvage procedure for neuropathic patients with ankle fractures that failed using traditional hardware techniques.

     

    Figure A is a AP and Lateral radiograph showing a Weber B type ankle fracture-dislocation. Illustration A shows treatment of unstable ankle fractures using ttrans calcaneal-talartibial pin fixation in a patients with diabetes mellitus.

     

    Incorrect Answers:

    Answer 1: Open deltoid repair would not be indicated in this population. Answer 2: Based on the fracture pattern and population, bridge plating alone would not provide adequate stability in this population. This fracture pattern requires anatomic reduction and rigid fixation.

    Answer 4: Denosumab administration would not increase stability of the fracture, or promote bone healing.

    Answer 5: Above knee post-operative immobilization would not improve the stability of the internal fixation device.

     

     

     

     

     

    OrthoCash 2020

     

  41. An 84-year-old female community ambulator with a history of hypertension undergoes a right hip hemiarthroplasty for a femoral neck fracture. When performed in the post-operative period, the timed up and go (TUG) test may be used to predict which patient outcome?

    1. Stair climbing ability

    2. Need for a walking aid

    3. Implant failure

    4. Balance impairment

    5. Independent performance of activities of daily living (ADL) Corrent answer: 2

    The timed up and go (TUG) test may be used as a clinical indicator of function and the need for a walking aid in patients treated with hip hemiarthroplasty for femoral neck fracture at 2-year follow-up.

     

    Hip fractures are a cause of significant functional decline for elderly patients. Many outcome tests have been developed to prediction function after hip

    fracture to manage patient expectations and to assist in rehabilitation planning. The TUG test objectively measures functional mobility and dynamic balance. The test is performed by timing the amount of seconds it requires for a patient to stand up from a chair, walk 10ft (3.05m), return to the chair, and sit.

     

    Laflamme et al performed a prospective study evaluating the utility of the TUG test to predict functional outcomes in patients undergoing hip hemiarthroplasty for femoral neck fracture. The TUG scores were significantly higher at 4-days and 3-weeks post-operatively in patients requiring a walking aid compared with patients walking independently at two-years. Patients who performed the test in >58s at 4-days post-operatively had an eightfold greater risk of requiring an assistive device.

     

    Springer et al prospectively analyzed the unipedal stance test (UPST) with eyes open and closed in healthy subjects to establish normative values for the test across age and gender groups. Performance on the test was found to be age-specific and not related to gender. The UPST is a method of quantifying static balance ability.

     

    Kristensen et al studied the relative and absolute inter tester reliability of TUG in patients with hip fractures. The authors found that the TUG has a high interobserver reliability and an improvement by 6.2 seconds for a patient with a baseline of 20s indicates a change in functional mobility.

    Video A shows the timed up and go test. Incorrect Answers:

    Answer 1. Stair climbing ability - The Lower Extremity Measure (LEM) includes

    stairs in the assessment.

    Answers 3, 5. Implant failure, Independent performance of activities of daily living (ADL) - Not predicted by the TUG test.

    Answer 4. Balance Impairment - Best assessed with the unipedal stance test (UPST).

     

     

     

    OrthoCash 2020

     

  42. A 41-year-old female sustains the injury shown in Figure A as a result of a high-speed motor vehicle collision. After a successful attempt at closed reduction in the emergency room using conscious sedation, repeat radiographs show a reduced hip joint. What is the next most appropriate step in treatment?

     

     

     

    1. Femoral skeletal traction

    2. CT scan of hip and pelvis

    3. Dynamic fluoroscopic examination under general anesthesia

    4. Hip spica dressing

    5. Touch down weight bearing mobilization Corrent answer: 2

    The radiograph shown in Figure A reveals a left hip dislocation, with some obscuring of detail secondary to the trauma backboard. CT scans should be obtained following a hip dislocation to evaluate for fractures or impacted areas of the femoral head or acetabulum, as well as noncongruent reductions and free intraarticular joint fragments.

     

    The referenced study by Brumback et al comments on the importance of postreduction CT scans and found that 23% of their posterior wall fractures had associated marginal impaction, with 94% of these discovered via preoperative CT scan.

     

     

     

    OrthoCash 2020

     

  43. A 30-year-old man sustained an acetabular fracture as the result of a gun-shot wound (GSW). If all other patient and treatment factors are the same, which fracture type (according to the Letournel classification) and injury pattern is LEAST likely to develop septic arthritis?

    1. T shaped with associated bowel injury

    2. Posterior column with posterior wall without associated bowel injury

    3. Transverse with posterior wall with associated bowel injury

    4. Anterior wall without associated bowel injury

    5. Associated both column with associated bowel injury Corrent answer: 4

    Associated acetabular fracture patterns (according to the Letournel classification) and the presence of an associated bowel injury are both independent risk factors for septic arthritis/deep infection in the setting of GSWs. Therefore an elementary fracture pattern (such as an anterior wall fracture) without the presence of an associated bowel injury is least likely to develop septic arthritis.

     

    Surgical indications for acetabular fractures due to GSWs include an intraarticular missile, a transabdominal GSW or an unstable fracture pattern. Bowel injuries are the most common associated injuries to occur with GSWs to the acetabulum. Due to the nature of these injuries, there is a high risk for major complications including septic arthritis/deep infections.

     

    Najibi et al. conducted a retrospective review of 38 patients with GSWs to the acetabulum. There was a 54% major complication rate. A multivariate logistic regression model identified that bowel injuries (P=0.04, HR=2.1) and associated fracture patterns (P=0.03, HR=2.3) were significantly associated with septic arthritis. With respect to the treatment of the bowel injury, primary anastomosis (versus colostomy) was also associated with the development of septic arthritis.

     

    Bartkiw et al. reported on a series of 42 patients with GSWs to the hip and pelvis. The most common associated injury was small/large bowel perforations (36%), followed by vessel laceration (17%) and urogenital injuries (5%). The authors concluded that orthopaedic intervention is indicated for intra-articular pathology and unstable fractures of the of hip and acetabulum.

    Illustration A shows Letournel Classification Fracture Types. Incorrect Answers:

    Answers 1,2,3&5 represent associated acetabular fracture patterns (with or

    without associated bowel injuries) according to the Letournel classification, which in the setting of GSWs are correlated with the development of septic arthritis/deep infections.

     

     

     

     

     

    OrthoCash 2020

     

  44. A 74-year-old female sustained a proximal humerus fracture after a ground level fall. Which of the following characteristics is the strongest indication for treating her proximal humerus fracture with an arthroplasty?

    1. Two part proximal humerus fracture

    2. Greater tuberosity fracture with 3 mm of posterior displacement

    3. 60 degree of initial varus angulation

    4. Valgus impacted fracture pattern

    5. Posteromedial metaphysis intact Corrent answer: 3

    Arthroplasty is indicated for a proximal humerus fracture in a geriatric patient that has 60 degrees of initial varus angulation.

     

    Most geriatric proximal humerus fractures are treated non-operatively, as the shoulder tolerates a substantial degree of malunion and many of these fractures are minimally displaced. However, surgical management is often recommended for displaced three and four part proximal humerus fractures. The optimal operative treatment for these fractures is somewhat controversial due to complications from both procedures. Hemiarthroplasty or reverse total shoulder arthroplasty is recommended in patients with highly comminuted fractures, a compromised rotator cuff, articular surface disruption, a short metaphyseal hinge, initial varus angulation > 20 degrees, and fracture dislocation. The most common complication from hemiarthroplasty includes nonunion and malunion of the tuberosities.

     

    Kontakis et al. performed a systematic review investigating the use of early hemiarthroplasty in proximal humerus fractures. The review included 16 studies, 808 patients, 810 hemiarthroplasties treating mostly four- part

    fractures or fracture dislocations. They found that though most patients never regained pre-injury level of function, patients also had no pain or mild pain after early hemiarthroplasty. Additionally, the most common complications were related to fixation and healing of the tuberosities (11.2%), followed by formation of heterotopic ossification at 8.8%.

     

    Solberg et al. retrospectively compared hemiarthroplasty with ORIF of the proximal humerus with a locked plate construct as treatment of three and four-part proximal humerus fractures. They concluded higher complications with the proximal locking plate including osteonecrosis of the humeral head, screw perforation in the humeral head, loss of fixation in patients with initial varus angulation of >20 degrees, and wound infection. Complications with hemiarthroplasty included nonunion of the tuberosities and wound infection.

    They also noted that despite the lower complication rates, it was found that hemiarthroplasty had worse outcome scores.

     

    Illustration A and B are radiographs of a 4 part proximal humerus fracture. Illustration C is the radiograph of a shoulder hemiarthroplasty performed after proximal humerus fracture.

     

    Incorrect answers:

    Answer 1: Classically, operative management of a two part proximal humerus fracture includes open reduction internal fixation or closed reduction and percutaneous pinning.

    Answer 2: Typically, greater tuberosity fractures with 3 mm of displacement may be treated non operatively. In young active patients, operative treatment may be performed, which is screw fixation or suture technique.

    Answer 4: Valgus impacted fractures may be treated with open reduction and internal fixation.

    Answer 5: Patients with an intact posteromedial hinge are thought to have maintained vascular supply to the humeral head and thus at low risk of avascular necrosis. A short metaphyseal hinge disrupting the blood supply would be an indication for arthroplasty in an older patient due high risk of avascular necrosis.

     

     

     

     

     

     

     

     

     

     

    OrthoCash 2020

     

  45. A 24-year-old female presents with right elbow pain after a mechanical fall from a bicycle. Hematoma aspiration and a lidocaine block improves her pain, and subsequent passive range of motion is 70-100 degrees of flexion/extension, 10 degrees of supination, and 10 degrees of pronation. What is the most appropriate management option?

     

     

     

     

    1. Radial head resection

    2. Radial head replacement

    3. Open reduction and internal fixation of the radial head

    4. Fragment excision

    5. Sling for 48 hours followed by early active and passive range of motion Corrent answer: 3

    This patient has sustained a displaced partial articular fracture of the radial head with a mechanical block to elbow motion (Mason type III). The most appropriate treatment is open reduction and internal fixation (ORIF).

     

    ORIF is indicated for displaced radial head fractures in order to restore normal radiocapitellar contact, retain valgus and axial stability of the elbow, and allow for early rehabilitation. If fracture fixation is unlikely to achieve these goals due to the pattern of injury, radial head arthroplasty or excision can be considered. In general, radial head arthroplasty is preferred for injuries with greater than three fracture fragments as well as those fracture that occur in conjunction with an elbow instability episode; resection is contraindicated in patients with elbow or distal radioulnar joint instability.

     

    Ruchelsman et. al. review the surgical management of radial head fractures. The authors suggest that surgical management is influenced by fragment number, fragment size, fragment comminution, fragment stability, displacement and block to motion, osteopenia, articular impaction, radiocapitellar malalignment, radial neck/metaphyseal comminution, and bone loss.

     

    Ikeda et. al. retrospectively reviewed outcomes of 28 patients with Mason type-III radial head fractures (many with concomitant injuries) treated with either radial head resection or open reduction and internal fixation (ORIF)

    through a lateral or posterolateral approach. The patients who underwent ORIF had significantly improved VAS pain scores, Broberg and Morrey functional score, and American Shoulder and Elbow Surgeons scores compared to those who underwent radial head resection. The authors concluded that ORIF can result in satisfactory joint mobility and function, and a generally better outcome when compared with radial head resection.

     

    Businger et. al. describe the "on-table" reconstruction technique for comminuted fractures of the radial head. In their study, they reported outcomes of seven patients with Mason type-III and type-IV radial head fractures treated with "on-table" radial head reconstruction using low-profile T-plates. At final follow-up (average 112 months), patients had a mean elbow range flexion arc of 0-141 degrees, 79 degrees of pronation, and 70 degrees of supination and excellent functional outcomes. All patients included in the study maintained a viable, functional radial head for up to 13 years post-operatively.

    Figure A is an AP radiograph demonstrated a three-part displaced fracture of the radial head splitting the center of the articular surface.

     

    Incorrect Answers:

    Answer 1: Radial head resection has been shown to yield worse outcomes than ORIF for Mason type III radial head fractures. Importantly, the distal radioulnar joint should be evaluated prior to radial head resection in order to ensure no axial radial instability (Essex-Lopresti injury)

    Answer 2: Radial head arthroplasty is indicated in fractures with >3 fragments or which occur in the setting of elbow instability. Arthroplasty may also be indicated if the fracture pattern suggests internal fixation is at risk of failure, which is not suggested in this case.

    Answer 4: Excision of the displaced lateral fragment in this case is inappropriate because it would remove 50% of the articular surface and leave a remaining unstable fracture fragment.

    Answer 5: Non-operative management is not indicated in patients with mechanical block to motion.

     

     

     

    OrthoCash 2020

     

  46. Patient outcomes can be improved and early post-injury management errors decreased by adhering to the Advanced Trauma Life Support (ATLS) protocol. This protocol includes criteria for inter-hospital transfer when the patient's needs exceed the resources available. Which of the following would it be acceptable to manage without interfacility transfer to a trauma center?

    1. A 29-year-old man with an open pelvic fracture

    2. A 42-year-old man with a major extremity crush injury

    3. A 31-year-old woman with an open forearm fracture

    4. A 36-year-old man with a spleen laceration after blunt trauma who is clinically stable and whose spleen laceration does not meet criteria for operative management

    5. An 18-year-old woman with multiple proximal long bone fractures Corrent answer: 3

    Patients with severe open fractures should be managed at a trauma center. A patient with an isolated open forearm fracture may not require transfer if her injury is isolated, relatively free of contamination and not otherwise complicated by other factors such as tissue ischemia or tissue loss.

     

    While most patients can receive all their definitive care at any hospital to which

    they present, early recognition of those that require transfer to a trauma center is essential to maximizing outcomes. Inherent in such identification is physician's' assessment of their own and their institutions' abilities and the limits thereof.

     

    Van Olden et al. prospectively studied severely injured patients consecutively presenting to two community hospitals in the Netherlands. Comparison was made between patients treated prior to and after introduction of ATLS training to physicians at the studied hospitals. There was no difference in overall mortality between the two groups. however, there was a significant reduction in mortality in the first 60 minutes after admission, from 24.2% pre-ATLS to 0.0% post-ATLS.

     

    Ali et al. studied trauma outcomes and mortality in patients presenting to the largest hospital in Trinidad and Tobago for the four years prior to and after introduction of ATLS training for physicians in the emergency department.

    Trauma mortality decreased to 33.5% from 67.5% after introduction of ATLS. Post-injury functional outcomes were also improved.

     

    Incorrect answers:

    Answer 1. Open pelvic fracture is an indication for transfer. Answer 2. Major crush is an indication for transfer.

    Answer 4. Expectant management of splenic and liver lacerations is predicated on the immediate availability of operative facility and appropriate staffing should the patient destabilize during non-operative management. Therefore, these patients must be transferred if expectant management is pursued.

    Answer 5. The presence of multiple proximal long bone fractures is an indication for transfer.

     

     

     

    OrthoCash 2020

     

  47. A 25-year-old male sustains the isolated injury shown in Figure

    A. Antegrade intramedullary nailing is planned for definite fixation within the next 12 hours. After obtaining lateral radiographs of the injury site, what would be the next best step in management of this patient?

     

     

     

    1. Convert to skeletal leg traction

    2. Radiograph/CT scan hip

    3. MRI scan hip

    4. Frog leg lateral hip radiograph

    5. Oncology consultation

     

    Corrent answer: 2

     

    This patient has sustained a femoral shaft fracture. The next best step in management would be to CT scan the hip to assess for an associated femoral neck fracture.

     

    Femoral neck fractures are seen less than 10% of the time with femoral shaft fractures, but they are frequently missed on initial evaluation. The neck fracture line is almost vertical and nondisplaced, or minimally displaced.

    Therefore fine (2-mm) cut CT scan through the femoral neck should be ordered in the preoperative workup of these patients.

     

    Tornetta et al. reported that they reduced the delay in diagnosis of concomitant femoral neck fractures by 91% by instituting a protocol that included: dedicated AP internal rotation plain radiograph, a fine (2-mm) cut CT scan through the femoral neck, an intraoperative fluoroscopic lateral

    radiograph prior to fixation, as well as postoperative AP and lateral radiographs of the hip in the operating room prior to awakening the patient.

     

    Figure A shows a trauma view lower extremity radiograph with an isolated left mid-shaft femur fracture.

     

    Incorrect Answers:

    Answer 1: Converting to skeletal leg traction would be necessary if a delay in treatment was expected.

    Answer 3: MRI scan of the left hip would not be necessary acutely for the identification of an associated fracture.

    Answer 4: Frog leg lateral hip radiograph would be much to painful to perform in the setting of fracture.

    Answer 5: The age, injury and fracture pattern are not consistent with a pathologic femur fracture.

     

     

     

    OrthoCash 2020

     

  48. ALL of the following factors are associated with increased risk of failure when treating an intertrochanteric hip fracture with a dynamic hip screw EXCEPT?

    1. Reverse oblique fracture pattern

    2. Tip apex distance of 32 mm

    3. Tip apex distance of 27 mm

    4. Use of a two hole side plate

    5. Lateral wall comminution Corrent answer: 4

    All of the factors listed are associated with increased risk of failure when treating an intertrochanteric hip fracture EXCEPT for the use of a 2 hole side plate DHS which has been shown to be an effective treatment for IT fractures.

     

    Dynamic hip screw (DHS) fixation of intertrochanteric hip fractures has been shown to produce good results when used appropriately. A common contraindication for use of a DHS is an unstable fracture pattern such as reverse oblique fractures, or those with associated lateral wall comminution. Tip-apex distance exceeding 25 mm has also been shown to increase the risk of screw cut out and loss of reduction.

     

    Baumgartner et al. present a review of 198 fractures to assess how predictive the tip-apex distance (TAD) is in predicting eventual screw cut-out. Average

    TAD for the successfully treated hips was 24 mm while the average TAD for hips that experienced cut-out was 38 mm (p=0.0001). There were no cases of cut-out when the TAD was less than 25 mm.

     

    Bolhofner et al. present a review of outcomes for 69 patients with intertrochanteric hip fractures treated with a 135° 2-hole DHS. They report "low blood loss" (range 10-300 cc, average 77 cc) and no cases of failure of fixation due to loss of fixation of the 2-hole side plate. 3 failures were noted: 2 due to screw cut-out, and one due to screw-plate dissociation.

     

    Illustration A depicts how to calculate the tip-apex distance. Illustration B demonstrates how a thin or comminuted lateral wall is at risk for failure with a sliding hip screw construct, and "d" is the measured lateral wall thickness.

     

    Incorrect answers:

    Answer 1: A reverse oblique fracture pattern is a contraindication to use of a DHS. This pattern may lead to distraction of the fracture rather than compression, and allow excessive medialization of the femoral shaft.

    Answers 2 and 3: Both of these TAD values exceed the recommended value of 25 mm.

    Answer 5: Lateral wall comminution is a contraindication to use of a DHS as it could allow excessive medialization of the femoral shaft as the screw slides laterally.

     

     

     

     

     

     

     

     

    OrthoCash 2020

     

  49. A 22-year-old male injures his ankle while playing ice hockey. He is seen in the emergency department, splinted, and sent to your office for evaluation. Radiographs obtained in your office are demonstrated in figures A, B, and C. Which of the following statements is most likely INCORRECT regarding this injury?

     

     

     

     

     

     

     

     

    1. If it were isolated, the amount of articular surface involvement of the posterior malleolus does not require operative fixation

    2. The posterior inferior tibiofibular ligament (PITFL) is disrupted

    3. Fibular length must be restored during reduction and fixation

    4. There are increased peak tibiotalar contact stresses if not anatomically reduced

    5. Rotational stiffness of the syndesmosis is greater with fixation of the posterior malleolus compared to placeing a trans-syndesmotic screw.

    Corrent answer: 2

     

    The patient sustained a displaced pronation-external rotation type 4 (PER-4) ankle fracture with a posterior malleolus fracture and tibiotalar subluxation. In rotational ankle fractures with posterior malleolus fragments, the PITFL itself is unlikely to be injured because the syndesmosis fails with an avulsion fracture of the posterior malleolus instead.

     

    The PER-4 ankle fracture is characterized by rupture of the anterior-inferior tibiofibular ligament (AITFL), a spiral or oblique Weber type C fibula fracture, disruption of the posterior-inferior tibiofibular ligament (PITFL) or the posterior malleolus, and failure of the deltoid ligament or fracture of the medial malleolus. Controversy remains regarding the exact indications for fixation of posterior malleolus fragments; however, either substantial involvement of the

    posterior plafond which destabilizes the tibiotalar joint or using the posterior malleolus to anatomically reduce the syndesmosis indirectly are commonly cited as reasons for fixation.

     

    Irwin et. al. reviewed fractures of the posterior malleolus, including the anatomy, biomechanics, classification, and surgical indications. The authors suggest nonsurgical management is reserved for isolated, non displaced posterior malleolus fractures and evaluated on CT, and operative fixation for those displaced fragments of 25-33% of the articular surface.

     

    Gardner et. al. evaluated the incidence of syndesmotic ligament disruption in PER-type ankle fractures associated with posterior malleolus fractures and also used a cadaveric model to assess post-surgical syndesmotic stability after posterior malleolus fixation compared with using a single 3.5 mm transsyndesmotic screw. The authors subsequently simulated PER-4 fractures in 10 cadaveric specimens and randomly fixed the posterior malleolus or the syndesmosis. Compared to intact specimens, they reported 70% of rotational stiffness restored after posterior malleolus fixation compared with 40% rotational stiffness after trans-syndesmotic fixation.

     

    Hartford et. al. evaluated 16 fresh cadaveric ankles to evaluate the effect of posterior malleolus fracture fragment size with an associated deltoid ligament disruption on tibiotalar contact area. They found that posterior malleolus fractures of 25%, 33%, and 50% of the articular surface created a corresponding decrease of 4%, 13%, and 22% respectively, of tibiotalar contact area. They found that 33% and 50% fragment sizes had statistically significant decreases in contact area which could predispose to degenerative changes in the tibiotalar joint. The authors recommended fixation of fragments greater than or equal to 33% of the tibial articular surface.

     

    Figure A is an AP radiograph of the ankle demonstrating a short oblique Weber C fibula fracture, cortical density overlap suggestive of the posterior malleolus fracture, and talar obliquity with tibiotalar overlap suggesting talar subluxation and deltoid ligament insufficiency.

     

    Incorrect answers:

    Answer 1. There is minimal involvement of the articular surface on the lateral image. The referenced literature suggests reduction and fixation for fragments involving at least 25% of the articular surface.

    Answer 3. Restoration of fibular length is critical to achieve an anatomic syndesmotic reduction.

    Answer 4. Loss of articular surface contact area can increase contact stresses within the joint and predispose to degenerative changes to the articular

    surface.

    Answer 5. The Gardner et. al. study demonstrated superior stiffness with fixation of the posterior malleolus fragment compared with syndesmotic screw alone.

     

     

     

     

     

     

    OrthoCash 2020

     

  50. An 82-year-old male sustains the injury shown in Figure A, and undergoes the procedure shown in Figure B. When discussing mortality risk with the patient and family after surgery, you note that the in-hospital and one-year mortality rate are which of the following?

     

     

     

     

     

    1. In hospital = 1%; 1 year = 17.5%

    2. In hospital = 2.5%, 1 year = 15%

    3. In hospital = 6%, 1 year = 22%

    4. In hospital = 10%, 1 year = 35%

    5. In hospital = 15% 1 year = 45%

     

    Corrent answer: 3

     

    Current in-hospital mortality rates are approximately 6%, while 1-year mortality rates depending on the study are found to be between 20-30%.

     

    Brauer et al. reviewed the incidence of hip fracture among U.S. elderly. They found that the overall incidence has decreased over the last two decades, but comorbidities among patients with hip fractures have increased. They also found that mortality rates have decreased during this time period.

     

    Jiang et al. reviewed nearly 4000 hip fracture patients, and found a 30.8% 1 year mortality rate, with increases in mortality with older age, male sex, preinjury residence in a nursing home, and patients with 10 or more comorbidities. They concluded that nearly 1 in 15 patients will die during hospitalization, and provide a risk-adjustment tool for risk stratification.

     

    Mundi et al. performed a meta analysis and assessed whether mortality and reoperation rates have improved in hip fracture patients over the past 31 years. They found that mean mortality rates for femoral neck fractures were very similar over time (~20%).

     

    Figure A shows a femoral neck fracture, while Figure B shows an uncemented hip hemiarthroplasty.

     

    Incorrect Answers:

    1,2,4,5: These mortality rates are incorrect.

     

     

     

    OrthoCash 2020

     

  51. During percutaneous iliosacral screw placement for an unstable pelvic ring injury, use of the lateral sacral fluoroscopic image is critical to help avoid iatrogenic injury to what structure?

    1. L4 nerve root

    2. L5 nerve root

    3. S1 nerve root

    4. Sacroiliac joint cartilage

    5. External iliac artery

    Corrent answer: 2

     

    Unstable anterior and posterior pelvic ring injuries are amenable to percutaneous treatment if reduction is able to be obtained in a closed manner and appropriate radiographic visualization is able to be achieved. In the 1996 reference by Routt et al, proper SI screw placement is described. Pelvic inlet, outlet, and lateral sacral images must be obtained to safely place a percutaneous iliosacral screw. The iliac cortical density seen adjacent to the SI joint is the anterior edge of the insertion safe zone, and is only able to be seen on the lateral image. Failure to place the screw behind this radiographic line would lead to an "in-out-in" screw (in the ilium, and then exiting anterior to the sacral ala, only to re-enter in the sacral body), which would cause direct injury to the L5 nerve root.

     

    In the 2000 reference by Routt et al, they state "a thorough knowledge of pelvic osseous anatomy, injury patterns, deformities, and their fluoroscopic correlations are mandatory for percutaneous pelvic fixation to be effective."

     

    Illustration A shows a representative lateral sacral radiograph, with the major anatomic landmarks labeled. Safe SI screw insertion in the S1 body should be underneath the sacral ala line to minimize risk of a "in-out-in" screw that would come out in the area of the ala and injure the L5 nerve root that sits directly on top of this structure. Dysmorphic pelvic rings will often have a more vertical sacral line, or one that starts more inferiorly.

     

     

     

     

     

     

    OrthoCash 2020

     

  52. Which of the following factors has been shown to have no impact on clinical outcomes in patients with the injury shown in Figure A and

    B?

     

     

     

     

     

    1. Presence of osteoporosis

    2. Preinjury level of function

    3. Smoking status

    4. Age

    5. Presence of diabetes Corrent answer: 4

    In patients with an unstable ankle fracture, age does not have an effect on outcomes.

     

    An increasing number of elderly patients retain an active lifestyle with high functional demands. Cast immobilization for highly unstable ankle fractures (especially the inherently unstable pronation-abduction fractures that are frequently seen in the elderly) leads to malunion or nonunion rates between 48% and 73%. Therefore, in the absence of severe systemic comorbidities, the principal indications for surgery should not differ from those in younger patients.

     

    Herscovici et al. evaluated 243 patients with high-energy foot and ankle injuries who underwent surgical fixation and found that when surgery was used in elderly individuals, the complications and outcomes are similar to those seen in younger patients.

     

    Pagliaro et al. analyzed outcomes after surgical treatment of unstable ankle fractures in 23 patients who were at least 65 years old. They found that open reduction and internal fixation of unstable ankle fractures in geriatric patients is an efficacious treatment regime that leads to results that are comparable to the general population.

     

    Figure A and B demonstrate AP and lateral radiographs of a trimalleolar ankle fracture.

     

    Incorrect Answers:

    Answer 1-3, 5: All of these factors have an impact on outcomes after unstable ankle fractures.

     

     

     

    OrthoCash 2020

     

  53. Which of the following is an FDA approved adjunctive treatment for an acute open tibia fracture being treated with an intramedullary nail?

    1. Calcitonin

    2. rhBMP-2

    3. rhBMP-7

    4. Teraparatide

    5. Bisphosphonates

     

    Corrent answer: 2

     

    rhBMP-2 has FDA approval for use when treating acute open tibia fractures with an intramedullary nail.

     

    Open tibial shaft fractures can present many treatment challenges. Although its use remains somewhat controversial, rhBMP-2 has been shown to have many positive effects when used to treat acute open tibia fractures. These benefits include accelerated early fracture healing, decreased rates of hardware failure, decreased need for subsequent bone grafting procedures, and decreased infection rates. rhBMP-2 does have FDA approval specifically for use in open tibia fractures being treated with an intramedullary nail.

     

    Alt et al. present a comparison of patients with Grade III open tibia fractures treated with un-reamed nails with or without rhBMP-2. They found significant decreases in need for secondary interventions such as bone grafting or nail exchange. Mean time to fracture healing was less in the rhBMP-2 group, but this difference was not statistically significant.

     

    Govender et al. present a prospective randomized study of 450 patients with open tibia fractures treated with an intramedullary nail with or without rhBMP-

    2. They found statistically significant decreases in need for secondary intervention, hardware failure, and infection as well as faster wound healing and faster time to fracture union.

     

    Wei et al. provide a meta analysis regarding use of rhBMP-2 in open tibia fractures. Due to decreased rates of secondary interventions they estimated a net savings of $6,000 per case when rh-BMP2 was used. They found no significant difference in rates of infection, postoperative pain, hardware failure, or fracture healing at 20 weeks.

     

    Incorrect answers:

    Answers 1, 3, 4, 5: These products do not have specific FDA approval as adjunct treatments for acute open tibia fractures. rhBMP-7 does have FDA approval for use in closed tibial non-unions.

     

     

     

    OrthoCash 2020

  54. A 76-year old patient underwent partial foot amputation through the talonavicular and calcaneocuboid joints. Besides Achilles tendon lengthening, what additional procedure(s) may be required to prevent the most common post-operative deformity?

    1. Posterior capsule release

    2. Anterior tibialis transfer to the talar neck

    3. Anterior tibialis transfer to cuboid

    4. Flexor hallucis longus transfer to calcaneus

    5. Peroneus brevis transfer to calcaneus Corrent answer: 2

    Achilles tendon lengthening AND anterior tibialis transfer to the talar neck would be required to prevent equinovarus deformity.

     

    Partial foot amputation through the talonavicular and calcaneocuboid joints is also known as the Chopart amputation. Chopart amputation alone is known to result in significant equinovarus deformity. This deformity results in excessive pressure on the anterior wound during gait, causing pain and wound complications. Transfer of the tibialis anterior tendon to talar neck will provide force, and muscle tone, that promotes ankle dorsiflexion. Lengthening of the Achilles tendon will also reduce the equinus moment force across the ankle joint.

     

    Dillion et al. examined the gait patterns of partial foot amputees. They found that amputations proximal to the metatarsal heads compromised the normal propulsive function of the foot and ankle. The ideal level of amputation to maintain normal propulsive function was distal to the metatarsal heads (i.e., disarticulating the metatarsophalangeal (MTP) joint).

    Illustration A is a lateral radiograph showing a Chopart amputation. Incorrect Answers:

    Answer 1: Achilles tendon lengthening and posterior capsule are insufficient to

    prevent the equinovarus deformity.

    Answer 3: The cuboid is removed with this amputation technique.

    Answer 4: Flexor hallucis longus transfer to the calcaneus would exacerbate the equinovarus deformity.

    Answer 5: Achilles tendon shortening would exacerbate the equinovarus deformity.

     

     

     

     

     

    OrthoCash 2020

     

  55. A 32-year-old male is involved in a motor vehicle collision and sustains the injury seen in Figure A. What is the most common urological injury associated with this injury pattern?

     

     

     

     

    1. Testicular torsion

    2. Posterior urethral tear

    3. Bladder denervation

    4. Testicular rupture

    5. Renal hematoma

     

    Corrent answer: 2

    The figure shows an anteroposterior pelvic ring injury. The most common urological injury with pelvic ring injuries remains the posterior urethral tear, followed by bladder rupture.

     

    Watnik et al notes lower urinary tract (bladder to end of urethra) injuries in up to 25% of patients with this injury. He reports that when contaminated urine communicates with the anterior arch, the possibility of infection exists, and early repair of bladder disruptions with simultaneous anterior arch plating minimizes this risk.

     

    Routt et al notes that even with simultaneous treatment of these injuries, complications are common (late stricture in 44%, impotence in 16%, delayed incontinence in 20% of females, anterior deep pelvic infection in 4%). Despite this, they report that early urological repairs are easily performed at the time of anterior pelvic open reduction and internal fixation.

     

     

     

    OrthoCash 2020

     

  56. A 26-year-old women, 31-weeks pregnant, presents to the emergency department with the injury shown in Figure A. She states the injury occurred while lifting a heavy vacuum five days ago. She suffers from chronic headaches and sleep disturbances. On inspection, there are multiple dorsal and volar bruising over her wrist and upper arm. She is neurologically intact. After closed reduction and immobilizing the arm, what would be the next best step in management of this patient?

     

     

     

    1. Diagnostic wrist arthroscopy

    2. Urgent MRI wrist

    3. Skeletal survey radiographs

    4. Request a consultation with social worker in the hospital

    5. Urgent open reduction internal fixation Corrent answer: 4

    This patient presents with classic features of domestic violence. The most appropriate next step would be consultation with a social worker at the hospital, assess for child and patient safety, and encouraging the patient to seek self-protection.

     

    Factors suggestive of domestic violence in the patient include pregnancy, delayed presentation after injury, inconsistent history, multiple bruises and complaints of chronic headache/sleep disturbances. Victims frequently miss days of work and as a result are at risk for losing their jobs. Victims are also more likely to engage in high-risk behavior with sex, drugs, alcohol, smoking, and eating.

     

    The AAOS published a document outlining the Orthopaedic Surgeon’s responsibilities in domestic and family violence. Musculoskeletal injuries that should raise a suspicion of a problem include (1) Multiple injuries/fractures; (2) Unusual patterns of injury/fracture; (3) Injuries/fractures of varying ages; (4)

    Injuries/fractures inconsistent with or disproportional to the history; (5) Multiple injuries treated in different hospital emergency departments or by different providers.

     

    Incorrect Answers:

    Answers 1,2,5: Work up of the injury would not be the next best step in the management of this patient. The responsibility of the orthopaedic surgeon would be to assess and assure the safety of the victim, appropriately treat victims and take steps to prevent further harm.

    Answer 3: Skeletal survey radiographs are not indicated in competent adults. The best steps would be to take photographs to document injuries, use body maps or drawings to document injuries, write legibly or use electronic medical records, utilize exact quotes of what the patient is describing, include the name/ names of the individual(s) who harmed the patient, fully describe the management of the injuries prior to your evaluation, describe the patient's demeanor and psychological state and involving other specialized health care works

     

     

     

    OrthoCash 2020

     

  57. A 45-year-old man undergoes open reduction and internal fixation for a comminuted intra-articular humerus fracture . An olecranon osteotomy is performed and subsequently fixed with an intramedullary cancellous screw. Which of the following options in the table shown in Figure A best describes the characteristics of this osteotomy?

     

     

     

     

    1. A

    2. B

    3. C

    4. D

    5. E

    Corrent answer: 4

     

    The chevron osteotomy is performed apex distal, 2cm from the palpable tip of the olecranon. The cancellous screw is driven slightly medially because of the proximal varus angulation of the ulna.

     

    An olecranon osteotomy provides extensile exposure of the distal humerus and elbow joint and exposes more of the articular surface (57%) than the triceps-splitting approach (35%) or the triceps-reflecting approach (46%). Both transverse and chevron osteotomies can be performed. To minimize articular surface disruption, it is recommended that the osteotomy be performed in the bare area of the olecranon. The bare area is described by Morrey as "transverse portion composed of fatty tissue divides the sigmoid notch into an anterior portion made up of the coronoid and the posterior olecranon." The osteotomy can be fixed with K-wires and a tension band construct or a single large cancellous screw. The proximal ulna has a slight metadiaphyseal varus angulation. The intramedullary screw should be long enough to reach the varus angle and should be directed in this direction.

     

    Wang et al. performed an anatomic study with 39 cadaver elbows. They found that on average, the bare area was 0.53cm wide, and located 2.1cm from the palpable tip of the olecranon. The length from the tip of the triceps insertion at the ulnar angle was 7.6cm, and the inner diameter of the medullary canal (at the ulnar angle) was 0.71cm (mediolateral) and 0.74cm (dorsovolar) and could easily accomodate a 7.0 or 7.3mm intramedullary screw. At 1-2cm proximal to the ulnar angle, the canal widened to dimensions larger than the 7.3mm cannulated screw. They recommend using this information when planning olecranon osteotomies and also in fixing simple olecranon fractures with an intramedullary screw.

     

    Elmadag et al. compared the olecranon osteotomy and triceps-lifting (Campbell) approach in 54 patients undergoing ORIF of distal humerus fractures. They found that functional outcomes and range of motion was better in the olecranon osteotomy group. Disadvantages of both approaches include: The olecranon osteotomy carries a risk of nonunion (<10%, higher with transverse osteotomy than chevron osteotomy). The triceps-lifting approach requires 3 weeks of postoperative immobilization for extensor healing.

     

    Illustration A shows the bare area of the ulna and the distance from the olecranon tip. Illustration B shows the Campbell triceps-lifting approach.

     

    Incorrect Answers:

    Answers 1, 2, 3, 5: The chevron osteotomy is performed apex distal, 2cm from

    the tip, and cancellous screw fixation will angle medially because of the proximal varus bend.

     

     

     

     

     

     

     

     

     

    OrthoCash 2020

     

  58. A 42-year-old man falls down a ravine while mountain biking, and is airlifted to a trauma center. Initial evaluation within one hour of injury reveals a temperature of 38.1 C, pulse rate of 119, blood pressure of 110/59, and bilateral femur and bilateral tibia shaft fractures. He has a GCS score of 15. The trauma surgeon follows standard precautions, and the patient has a negative CT Chest/Abdomen/Pelvis, after which he is sent to the floor to await surgical intervention the next day. Four hours later, a rapid response is called after his pulse is noted to be 140 and blood pressure is 88/50.

    Which statement best describes the patient's hemodynamic status from presentation onward?

    1. The patient has likely lost about 20% of his total blood volume.

    2. The patient presented in Class II hemorrhagic shock and now is in Class III hemorrhagic shock.

    3. This patient has a decompensation of his hemodynamic status and the radiologist likely missed a solid organ injury given his most recent parameters.

    4. The patient cannot go to the operating room for at least 24 hours.

    5. The patient's decline in blood pressure is likely due to cardiogenic shock Corrent answer: 2

    The patient presented with vital signs indicative of Class II hemorrhagic shock (tachycardic > 100 bpm, maintained blood pressure). When his rapid response was called, he was increasingly tachycardic and hypotensive, which define Class III hemorrhagic shock (tachycardic > 120 with decreased blood pressure).

     

    Long bone fractures - especially femurs - are an important source of bleeding in trauma patients. Each thigh can accommodate around 20% of total blood volume alone. With four long bone fractures and a triage presentation in Class II shock, the patient likely would have benefited from a more aggressive resuscitation and closer monitoring in the ICU. While early total care can place patients with pulmonary and head injuries at a higher risk of complications, patients with fractures as their only source of bleeding may be best treated with aggressive resuscitation and early stabilization of their long bones. Labs like lactate levels and base excess can help guide resuscitation.

     

    The ATLS handbook put out by the American College of Surgeons provides estimations of percent total blood volume loss for given vital sign changes and a patient's mental status. While these are only estimations, they can help clinicians understand the severity of a patient's hemorrhagic shock and guide resuscitation.

     

    Guly et al. compare estimated blood volume loss in trauma patients to their vital signs in the UK over a nearly two-decade span. They concluded that the ATLS guidelines may underestimate the actual blood loss in patients for their given cutoffs of vital sign abnormalities. Of course, it can be extremely difficult to estimate blood loss in an acute trauma patient; therefore, comparing ongoing sources of to a patient's resuscitation parameters may offer a better understanding of their hemodynamics than estimating fluids alone.

    Illustration A is the classification system used for hemorrhagic shock. Incorrect Answers:

    Answer 1: The patient presented in Class II shock - or compensated Class II

    shock - but now is in Class III shock which estimates 30-40% loss of total blood volume

    Answer 3: While it is true that his hemodynamic status is decompensating, this can occur from multiple long bone fractures alone (especially with bilateral femoral shaft fractures)

    Answer 4. Orthopaedic surgeons need to remain an active member in the early resuscitation and care for trauma patients. Ultimately, this patients bleeding is best treated with stabilizing the patient's long bones.

    Answer 5: Cardiogenic shock refers to a condition where the heart is the reason that inadequate tissue perfusion occurs. Most commonly, it is associated with myocardial infarcts that affect enough myocardium that the pump function is inadequate. Most patients in blunt traumas who present in shock have hemodynamic shock, which responds to fluids / blood products. The first-line treatment in cardiogenic shock is ionotropic support.

     

     

     

     

     

     

    OrthoCash 2020

     

  59. A 78-year-old male presents to the emergency room following a mechanical fall. Radiographs are shown in Figures A, B, and C. Past medical history is significant for early Alzheimer’s disease. Preoperative bloodwork demonstrates a creatinine clearance of 120 mL/min. What is the most appropriate treatment plan for the patient?

     

     

     

     

     

     

     

     

    1. Admit to orthopaedics, minimize centrally-active pain medications, urgently proceed to the operating room within 12 hours for operative fixation

    2. Admit to orthopaedics with geriatrician co-management, minimize centrally-acting pain medications, rapid medical optimization, and urgently proceed to the operating room within 48 hour for fracture fixation

    3. Admit to orthopaedics with neurology consultation for pre-operative evaluation of dementia, rapid medical optimization, and urgently proceed to the operating room within 48 hours for operative fixation

    4. Admit to orthopaedics, perform routine echocardiogram, anesthesia preoperative assessment, cardiology consultation given her relative cardiac risk index (RCRI), and urgent operative fixation

    5. Admit to orthopaedics, rehydration and nephrology consult for impaired renal function, medical clearance, and urgently proceed to the operating room within 48 hours for operative fixation.

    Corrent answer: 2

     

    The patient is a geriatric male with dementia who presents with an intertrochanteric hip fracture. The most appropriate next steps in treatment include admission with geriatrician co-management, minimization of centrally-acting pain medications, rapid medical optimization, and operative fixation within 48 hours.

     

    Hospital management of elderly patients with hip fractures should include medical evaluation and avoidance of delirium-inducing medications. Centrally-acting medications, such as meperidine, can induce delirium and should not be used, especially in the presence of less delirium-inducing alternatives. These drugs should be used cautiously due to age related changes in drug metabolism and renal elimination.

    Friedman et. al. report on the experience of co-management of hip fracture patients by both orthopaedic surgeons and geriatricians. They reported that geriatricians helped to address the risk of adverse outcomes and manage those individuals at higher risk for delirium, infection, and functional decline. Their

    co-management model resulted in shorter hospital length of stay, lower readmission rates, short time to surgery, low complication rates, and low mortality.

     

    Kates et. al. retrospectively reviewed the financial results of an organized hip fracture program for adults greater than 60 years old. The program involved orthopaedic and geriatric co-management of patient care with emphasis placed on rapid geriatric assessment and optimization, with surgery typically achieved within 24 hours of admission. The authors identified a 33.3% cost decrease relative to national averages for the episode of care.

     

    Potter reviewed the perioperative considerations necessary for orthopaedic patients >65 years of age. She reports that patients with dementia are more likely to develop in-hospital delirium as well as present challenges in communication and decision-making. Older patients taking multiple drugs present a unique challenge as they are at higher risk for adverse events and drug interactions including sedation and cognitive impairment.

     

    Figures A, B, and C demonstrate trauma series of the hip with an unstable intertrochanteric fracture.

     

    Incorrect Answers:

    1. Operative fixation without prior medical evaluation and optimization is inappropriate.

    1. Preoperative evaluation of dementia is likely unnecessary as it will potentially delay operative fixation and will not alter the patient's level of dementia.

    2. Echocardiogram should not be part of routine preoperative workup unless the patient's medical comorbidities and cardiac risk suggest its utility.

    3. This patient has a normal-appearing creatinine clearance. Importantly, elderly patients have decreased muscle mass and therefore decreased creatinine production, which may cause renal function to appear normal when it is, in fact, not.

     

     

     

    OrthoCash 2020

  60. A 68-year-old female sustains a closed ankle fracture and is treated with open reduction and internal fixation. Her postoperative radiographs are shown in Figure A. Widening of the tibia-fibular clear space with external rotation stress would be a result of injury of which structure?

     

     

     

     

    1. Anterior ankle joint capsule

    2. Syndesmosis

    3. Anterior talofibular ligament

    4. Posterior tibial tendon

    5. Calcaneofibular ligament

     

    Corrent answer: 2

     

    The radiograph shows a trimalleolar ankle fracture, which can be associated with syndesmosis injuries. The syndesmosis acts to maintain the relationship of the fibula in the incisura fibularis of the distal tibia as well as the congruity of the ankle joint. Failure to detect these injuries can lead to lateral talar shift and negative outcomes. So, if there was an isolated fibula fracture, the stress examination would test the deep fibers of the deltoid ligament complex.

    However, in this case, with fixation of the fibula, widening of the ankle joint would require an injury to the syndesmosis, as this structure would prevent it after restoration of the lateral column of the ankle (fibula).

     

    Beumer et al describe that stress radiographs may be performed by external rotation stress on the hindfoot or by providing a lateral "pull" on the distal

    fibula after fixation (Cotton test).

     

    Park et al showed that "ankle stress radiographs taken in dorsiflexion-external rotation were most predictive of deep deltoid ligament disruption after distal fibular fracture. Under this stress condition, a medial clear space of > or =5 mm was the most reliable predictor of deep deltoid ligament status."

     

     

     

    OrthoCash 2020

     

  61. What is the most prevalent adverse event associated with allogeneic blood transfusion?

    1. Clerical error leading to transfusion reaction

    2. Anaphylactic reaction

    3. HIV transmission

    4. Hepatitis C transmission

    5. Bacterial contamination leading to sepsis/shock Corrent answer: 1

    Clerical error leading to acute hemolysis and even death occurs in 1:12,000 to 1:50,000 transfusions. Bacterial contamination leading to sepsis/shock occurs in 1:1 million transfusions. HIV transmission is approximately 1:500,000 transfusions and hepatitis C is 1:103,000 transfusions. Anaphylactic reactions occur in 1:150,000 transfusions.

     

     

     

    OrthoCash 2020

     

  62. A 30-year-old male falls off the roof and sustains the injury seen in Figure A. Multiple attempts at a closed reduction are made, but are unsuccessful. Entrapment of which of the following structures is the most likely etiology?

     

     

     

    1. Peroneal tendons

    2. Posterior tibial tendon

    3. Extensor retinaculum

    4. Anterior tibial tendon

    5. Flexor hallucis longus Corrent answer: 2

    The radiograph shows a lateral subtalar dislocation with an associated talonavicular dislocation as well. Dislocations of the talonavicular are often seen with subtalar dislocations and one needs a high index of suspicion in order not to miss this associated injury.

     

    Subtalar dislocations are associated with high energy, open(25%), and irreducible (33%) fractures. Lateral subtalar dislocations are thought to result from forceful eversion of a plantar-flexed foot, with the anterior process of the calcaneus acting as a fulcrum around which the anterolateral corner of the talus pivots. The reduction maneuver involves initial traction and foot hyperpronation, followed by supination for lateral dislocations.

     

    Saltzman and Marsh present a review article about acute hindfoot dislocations and discuss their management and long term follow up. Lateral dislocations

    that are irreducible are blocked most commonly by the posterior tibialis tendon. The tendons of the FHL and FDL less commonly block reduction. These dislocations often require emergent open reductions, tendon relocation, and stabilization. Medial dislocations account for 65%, and reduction is often blocked by the extensor digitorum brevis. They are thought to be more common due to the strong buttress of the lateral malleolus.

     

     

     

    OrthoCash 2020

     

  63. A 22-year-old male sustains a right shoulder injury after being thrown from his motorcycle. After nine months of conservative treatment, he continues to complain of pain. A current radiograph is shown in Figure A. What is the most appropriate treatment?

     

     

     

     

    1. Addition of a bone stimulator

    2. Figure of eight brace

    3. Closed reduction and percutaneous pinning

    4. Open reduction and intramedullary nailing

    5. Open reduction and compression plating Corrent answer: 5

    Figure A shows a clavicular nonunion. Observation is the wrong answer because the patient is symptomatic (if the patient is asymptomatic an atrophic nonunion of the clavicle can be observed unless neurovascular symptoms are present). Intramedullary fixation is difficult because the pin has to pass through thin atrophic ends of bone close to neurovascular structures.

    Percutaneous pinning may cause distraction and migration of K-wires is common.

     

    In the referenced study by Boyer et al, the authors note success in treating atrophic nonunions of the clavicle with compression and lag-screw fixation (absolute stability), in conjunction with cancellous autograft.

     

    The referenced study by Bradbury et al reports on 32 clavicle nonunion

    procedures. They note 31 of 32 went on to union with use of a 3.5mm recon plate or small fragment DCP plate, also in conjunction with cancellous autograft.

     

     

     

    OrthoCash 2020

     

  64. Reamed femoral intramedullary nailing is associated with a higher rate of which of the following, as compared to nonreamed nailing for distal femoral shaft fractures?

    1. Malalignment

    2. Pulmonary complications

    3. Need for transfusion

    4. Iatrogenic fracture

    5. Union

     

    Corrent answer: 5

     

    Reamed intramedullary femoral nailing is associated with a higher rate of union than nonreamed femoral nailing.

     

    The reference by the Canadian group randomized 224 patients to reamed vs. unreamed femoral nails and found that the relative risk of nonunion was 4.5x greater without reaming, and nonunion was also greater with the use of a small-diameter nail.

     

    The referenced article by Tornetta et al randomized 81 patients to reamed or unreamed nails and found more intraoperative technical complications in the group without reaming. There was no statistical difference in OR time, transfusion requirement or pulmonary complications between the groups. This study showed the overall union rate was similar but when they selected out distal femur fractures, the reamed group healed faster.

     

    The reference by Brumback et al is a review of reamed v. nonreamed nailing, with discussions of reaming techniques and the importance of proper reamer technology and usage.

     

     

     

    OrthoCash 2020

     

  65. When performing a Kocher approach to the radial head for open reduction internal fixation the forearm is held in pronation. What structure is this maneuver attempting to protect?

    1. median nerve

    2. brachial artery

    3. anterior interosseous nerve

    4. radial nerve

    5. posterior interosseous nerve Corrent answer: 5

    Dilberti et al quantified the dimensions of a surgical safe zone (with respect to the posterior interosseous nerve) when using the posterolateral approach to the radial head between the anconeus and the extensor carpi ulnaris. They found that the safe zone increased with pronation and decreased with supination.

     

     

     

    OrthoCash 2020

     

  66. During placement of an external fixator for a distal radius fracture, the most commonly injured nerve is a branch of which of the following nerves?

    1. Ulnar

    2. Median

    3. Superficial radial

    4. Lateral antebrachial cutaneous

    5. Medial antebrachial cutaneous Corrent answer: 3

    Pin track infections and sensory injuries are among the most common complications of external fixation for distal radius fractures. The proximal pins of most distal radius external fixators are placed in the “bare area” of the distal radius, about four finger-breadths above the radial styloid. This corresponds to the area where the dorsal sensory branch of the radial nerve penetrates the fascia dorsal to the brachioradialis tendon to become a subcutaneous structure. Injury to the superficial radial nerve may produce painful dysesthesias and neuromas.

     

     

     

    OrthoCash 2020

     

  67. In patients with displaced radial neck fractures treated with open reduction and internal fixation with a plate and screws, the plate must be limited to what surface of the radius to avoid impingement on the proximal ulna?

    1. 2 cm distal to the articular surface of the radial head

    2. 1 cm distal to the articular surface of the radial head

    3. Within a 90-degree arc or safe zone

    4. Within a 120-degree arc or safe zone

    5. Within a 180-degree arc or safe zone Corrent answer: 3

    The radial head is covered by cartilage on 360 degrees of its circumference. However, with the normal range of forearm rotation of 160 to 180 degrees, there is a consistent area that is nonarticulating. This area is found by palpation of the radial styloid and Lister’s tubercle. The hardware should be kept within a 90-degree arc on the radial head subtended by these two structures.

     

     

     

    OrthoCash 2020

     

  68. A patient is treated with volar plating for a distal radius fracture. The CT scan shown in Figure 15 is obtained after union of the fracture because the patient reports ongoing symptoms. The prominent hardware is most likely injuring what tendon?

     

     

     

     

    1. Extensor pollicis brevis (EPB)

    2. Extensor carpi radialis brevis (ECRB)

    3. Extensor digitorum communis (EDC)

    4. Extensor carpi ulnaris (ECU)

    5. Extensor carpi radialis longus (ECRL) Corrent answer: 3

    Extensor tendon injuries have been reported after volar plating of distal radius fractures. The CT scan shows prominent dorsal hardware a few millimeters ulnar to Lister’s tubercle. The second compartment, the ECRL and ECRB, is radial to Lister’s tubercle. The ECU runs along the distal ulna. The contents of the fourth dorsal compartment run just ulnar to Lister’s tubercle. The EDC tendon is likely irritated in this patient. The EPB runs along the radial border of the radius and is well away from prominent hardware.

     

     

     

    OrthoCash 2020

     

  69. A 30-year-old patient underwent open reduction internal fixation of a talar neck fracture 8 weeks ago. His current radiographs demonstrate a subchondral radiolucency of the dome of the talus.

    What is the next most appropriate course of action?

     

    1. Injection of bone cement into the talus to prevent further avascular necrosis

    2. Ankle fusion

    3. Subtalar fusion

    4. Ankle arthroscopy to address this osteochondral lesion

    5. Continued observation as the vascularity to the talus is intact Corrent answer: 5

    A subchondral radiolucency of the talar dome after a talar neck fracture is known as the "Hawkins sign" and is a well-described radiographic indication of viability of the talar body. Rockwood and Green state "by the 6th-8th week, if the patient has been non-weight-bearing, diffuse atrophy is evident by radiographs. An AP radiograph of the ankle reveals the presence or absence of subchondral atrophy in the dome of the talus. Subchondral atrophy excludes the diagnosis of avascular necrosis." Tezval et al in a retrospective review showed that a subchondral lucency seen on the AP radiograph was a good indicator of talus vascularity following fracture. They state it is unlikely that AVN will develop at a later stage after injury if a Hawkins sign was present.

    Illustration A shows the characteristic appearance of a Hawkins sign and subchondral sclerosis.

     

     

     

     

     

    OrthoCash 2020

     

  70. Myodesis of which muscle group is most important for optimal outcome after transfemoral amputation?

    1. Abductors

    2. Adductors

    3. Hip flexors

    4. Hip extensors

    5. Hip external rotators Corrent answer: 2

    Adductor myodesis is critical for optimal outcome after transfemoral amputation.

     

    Pinzur et al. emphasize that when the adductors are not anchored to bone, the hip abductors are able to act unopposed, producing a dynamic flexion-abduction deformity. This deformity prepositions the femur in an orientation that is not conducive to efficient walking. The retracted adductor muscles lead to a poorly cushioning soft-tissue envelope, further complicating prosthetic fitting. Preservation of a functional adductor magnus helps to maintain the muscle balance between the adductors and abductors by allowing the adductor magnus to maintain its power and retain the mechanical advantage for positioning the femur. Preservation is best accomplished with a myodesis.

     

     

     

    OrthoCash 2020

  71. A 75-year-old woman began a walking program 2 months after undergoing right total knee arthroplasty. She had to stop the program after 4 weeks because of hindfoot pain and ankle swelling.

    Radiographs are shown in Figures 42a and 42b. What is the most likely diagnosis?

     

     

     

     

     

     

     

    1. Plantar fasciitis

    2. Osteochondral lesion of the talus

    3. Heel spur

    4. Insufficiency fracture of the calcaneus

    5. Chondrocalcinosis of the ankle joint Corrent answer: 4

    It is often tempting to assign a diagnosis of plantar fasciitis in patients with hindfoot pain. In this patient, the radiographs confirm a diagnosis of a calcaneal insufficiency fracture. The dense condensation of bone on the lateral view confirms the diagnosis. There is no radiographic evidence of a heel spur, osteochondral lesions, or chondrocalinosis.

     

     

     

    OrthoCash 2020

     

  72. All of the following are advantages of supine over lateral positioning during intramedullary nailing of subtrochanteric femur fractures EXCEPT:

    1. Can be protective to an injured spine

    2. Facilitates access to other injured sites in the polytrauma patient

    3. Provides easier fluoroscopic imaging

    4. Allows for easy reduction of the distal fragment to the flexed proximal fragment

    5. Easier to assess rotation Corrent answer: 4

      Based on the references provided, the advantages of the lateral position include: facilitates the retraction of the vastus lateralis, allows hip flexion to aid reduction, improves access to the proximal segment (easier to get starting point). Disadvantages of the lateral position include: intraoperative imaging may be more difficult, rotation is more difficult to judge, and lateral positioning may not be practical in the polytraumatized patient.

       

      Advantages of the supine position include: may help protect a potentially unstable spine, facilitates access to sites other than the injured femur, shorter setup time, rotational and angulatory deformities may be more easily appreciated. Disadvantages of the supine position include: starting point localization may be more difficult.

       

       

       

      OrthoCash 2020

       

  73. Following a fall from a height of 5 feet, a patient reports pain along the lateral border of the foot. The CT scan shown in Figure 54 indicates what pathology?

     

     

     

    1. Impaction injury of the cuboid

    2. Retracted os peroneum

    3. Fifth metatarsal avulsion fracture

    4. Avulsion injury of the bifurcate (Y) ligament

    5. Lisfranc injury

     

    Corrent answer: 4

     

    The CT scan reveals an avulsion of the dorsal beak of the anterior process of the calcaneus. This common fracture is an avulsion of the origin of the bifurcate ligament, which runs from the anterior calcaneal process to both the cuboid and the lateral aspect of the navicular. An inversion mechanism is common, and the fracture is often missed in evaluation for a suspected ankle sprain. MRI may be useful in the diagnosis of these occult injuries, and suspicion should be present when tenderness exists over the superior portion of the anterior process of the calcaneus.

     

     

     

    OrthoCash 2020

     

  74. A 70-year-old woman with known osteoporosis sustains a distal radius fracture of her dominant arm with some metaphyseal comminution. Adequate maintenance of reduction by non-operative treatment is unsuccesful. Which plating option provides the most appropriate treatment of this fracture?

    1. semitubular

    2. dynamic compression

    3. limited-contact dynamic compression

    4. peri-articular locked

    5. pelvic reconstruction

     

    Corrent answer: 4

    Egol et. al. studied locked and conventional plates. They concluded that locked plates may be increasingly indicated for indirect fracture reduction, diaphyseal/metaphyseal fractures in osteoporotic bone, bridging severely comminuted fractures, and the plating of fractures where anatomical constraints prevent plating on the tension side of the bone. Conventional plates remain the fixation method of choice for periarticular fractures that demand perfect anatomical reduction, and certain types of non-unions that require increased stability for union.

     

     

     

    OrthoCash 2020

     

  75. In addition to lengthening the Achilles, transfer of which tendon is most important for functional ambulation after performing a Chopart amputation of the foot?

    1. Peroneus brevis

    2. Peroneus longus

    3. Tibialis anterior

    4. Tibialis posterior

    5. Flexor hallucis longus Corrent answer: 3

    The partial foot amputation through the talonavicular and calcaneocuboid joints is also known as the Chopart amputation. The Chopart amputation may result in significant equinovarus deformity with anterior weight bearing through the scar line, predisposing to skin breakdown over time. Therefore, lengthening of the Achilles tendon and transfer of the tibialis anterior to the talar neck should also be performed in conjunction with this disarticulation.

     

    The tibialis anterior transfer results in dorsiflexion and distributes the weightbearing portion more centrally and the lengthening of the Achilles tendon is necessary to accommodate this posteriorly. Transfer of the tibialis anterior or posterior tibialis to the calcaneus would exacerbate the equinovarus deformity. Shortening of the Achilles tendon would also exacerbate the anterior loading of the scar.

     

     

     

    OrthoCash 2020

     

  76. The risk for osteonecrosis after a proximal humerus fracture is greatest for what fracture pattern?

    1. Two-part fracture of the surgical neck

    2. Three-part fracture

    3. Four-part fracture

    4. Four-part valgus impacted

    5. Four-part fracture-dislocation

     

    Corrent answer: 5

     

    The risk of osteonecrosis for a proximal humerus fracture is greatest for four-part fractures with associated dislocation. Isolated surgical neck and isolated greater tuberosity fractures have a very low incidence of osteonecrosis as the blood supply to the humeral head is relatively preserved. Four-part fractures have a higher incidence of osteonecrosis than three-part fractures. Four-part fractures with associated dislocation have the highest risk for osteonecrosis.

     

     

     

    OrthoCash 2020

     

  77. Which one of the following lower extremity amputations requires a soft-tissue balancing procedure to prevent deformity following amputation?

    1. Gritti-Stokes

    2. Transtibial

    3. Transmetatarsal

    4. Knee disarticulation

    5. Lisfranc

     

    Corrent answer: 5

     

    The Lisfranc level amputation removes the attachment of the peroneus brevis (base of the fifth metatarsal) and the peroneus longus (base of the first metatarsal), creating a varus deformity due to unopposed overpull by the tibialis anterior and posterior muscles. An anterior tibialis tendon transfer may be necessary. Other possible tissue balancing choices include posterior tibialis transfer and lengthening of the gastrocsoleus complex. Another option is to leave the base of the fifth metatarsal attached to preserve the eversion pull of the peroneus brevis. All the other amputations do not require soft-tissue balancing procedures to prevent deformities. However, adherence to the prescribed surgical techniques for reattachment of major muscle groups is important for optimizing limb strength and function.

     

     

     

    OrthoCash 2020

     

  78. When patients undergoing chronic hemodialysis for end stage renal disease sustain a displaced femoral neck fracture, treatment with open reduction and internal fixation will result in

    1. a lower mortality rate than patients treated with hemiarthroplasty.

    2. a higher mortality rate than patients treated with hemiarthroplasty.

    3. a lower complication rate than patients treated with hemiarthroplasty.

    4. a higher complication rate than patients treated with hemiarthroplasty.

    5. significant fracture shortening but an acceptable union rate of 80%.

     

    Corrent answer: 4

     

    Hemiarthroplasty is the treatment of choice for displaced femoral neck fractures in patients who are undergoing dialysis for end stage renal disease. A recent study reported outcomes of patients on chronic hemodialysis for end stage renal disease and who were surgically treated for a hip fracture. The series included 13 femoral neck fractures that were treated with open reduction and internal fixation and eight femoral neck fractures treated with hemiarthroplasty. The overall mortality rate was 45% at a mean duration of follow-up of 23 months, but there was no difference in mortality between the groups. Successful union was achieved in only two of the 13 hips treated with open reduction and internal fixation. There were 16 complications in the group treated by open reduction and internal fixation, compared with only one complication in the hemiarthroplasty group. Regression analysis of age, sex, and total hemodialysis duration in relation to mortality risk showed that only age had a significant effect on mortality (P = 0.019).

     

     

     

    OrthoCash 2020

     

  79. During an ilioinguinal approach for fixation of the anterior pelvic ring, brisk bleeding is encountered as the dissection is extended along the superior pubic ramus approximately 5 cm from the midline. What structure has most likely been injured?

    1. A branch of the femoral artery

    2. An anastomosis between the external iliac artery and obturator artery

    3. An anastomosis between the external iliac and femoral artery

    4. Internal iliac artery

    5. External iliac artery Corrent answer: 2

    The corona mortis, or "crown of death," is a common anatomic variant that consists of an anastomosis between the obturator and the external iliac or inferior epigastric arteries or veins. Its reported incidence is over 80%. It is located behind the superior pubic ramus at a variable distance from the symphysis pubis (3 cm to 9 cm). It is at risk during surgical approaches to the

    anterior pelvic ring. If accidentally cut, the vessel can retract making control of hemorrhage difficult.

     

     

     

    OrthoCash 2020

     

  80. A 51-year-old man has an open book pelvic fracture with a 6-cm anterior pubic diastasis. Which of the following studies should be obtained to rule out a urethral injury?

    1. Cystogram

    2. Urinalysis

    3. CT of the abdomen

    4. Diagnostic peritoneal lavage

    5. Retrograde urethrogram

     

    Corrent answer: 5

     

    About 15% of patients with pelvic ring injuries have a genito-urinary injury. A cystogram evaluates the bladder but with a bladder tear and too much contrast leaking around the perineum, the urethra cannot be evaluated. A urinalysis with more than 25 RBCs/hpf merits a cystogram in a woman. In a patient with an indwelling urinary catheter, CT of the abdomen will not assess the urethra. A diagnostic peritoneal lavage is used to look for blood in the abdomen. The retrograde urethrogram is the best test to assess injury to the urethra.

     

     

     

    OrthoCash 2020

     

  81. Locked lateral plate fixation of the proximal tibia is most appropriate for what Schatzker type fracture?

    1. I - lateral plateau split

    2. II - lateral plateau split/depression

    3. III - lateral plateau depression

    4. IV - medial plateau fracture

    5. V - bicondylar tibial plateau fracture Corrent answer: 5

    Locked lateral plate fixation of the proximal tibia has proven effective for bicondylar fractures (Schatzker types V and VI). The laterally based plate provides support for the lateral condyle fracture and locked proximal screws support the medial fragment from varus collapse. A medial antiglide plate can be used in conjunction with the locked plate. Lateral locked plate fixation has not been shown to have advantages for pure lateral fractures (Schatzker types

    I, II, and III) that require buttress plating or support of depressed articular components. Pure medial fractures (Schatzker type IV) are best treated with medial fixation.

     

     

     

    OrthoCash 2020

     

  82. A patient is seen in the emergency department after a motor vehicle accident. He reports right hip pain and chest pain. Initial hypotension has responded to a fluid bolus. Radiographs reveal a posterior hip dislocation with a small posterior acetabular wall fracture. You are called at home and informed of the findings. What is the next most appropriate step in management?

    1. Obtain a CT scan to assess the injury.

    2. Obtain an MRI scan to assess for osteonecrosis.

    3. Reduce the hip and evaluate hip stability.

    4. Perform open reduction of the hip in the operating room.

    5. Ask the emergency room physician to transfer the patient to a higher level trauma center.

    Corrent answer: 3

     

    An immediate reduction of the hip is required. Transfer to a trauma center may be indicated to treat a possible chest injury and the acetabular fracture.

    Reduction of the hip dislocation should be considered emergent and should be performed prior to transfer. Additional diagnostic studies prior to hip reduction are not necessary. Most hip dislocations can be reduced closed and this is the preferred management.

     

     

     

    OrthoCash 2020

     

  83. Figures 12a and 12b show the radiographs of a 48-year-old man who sustained an isolated injury in a motor vehicle collision.

    Intramedullary stabilization is elected and performed. Methods to aid in preservation of reduction include all of the following EXCEPT

     

     

     

    1. oblique proximal interlocking screws.

    2. countersinking the nail.

    3. blocking screws.

    4. a proximal starting point in line with the lateral eminence.

    5. a unicortical plate.

     

    Corrent answer: 2

     

    Methods of intramedullary stabilization of proximal quarter tibia fractures have, in the past, been complicated by the seeming incompatibility of osseous anatomy, implants, and technique. Nail design, trajectory, and starting point must be considered in an effort to limit undesirable malreduction in both sagittal and coronal planes. Blocking screws placed prior to nail insertion serve to narrow the effective diameter of the intramedullary canal thereby facilitating reduction. The judicious use of a unicortical plate inserted with minimally invasive techniques may serve a similar purpose. The retention of devices using either technique may enhance construct stability, preserving reduction as does the use of proximal oblique (rather than transverse) screws.

    Countersinking the nail diminishes desirable "aperture fixation" within the proximal fragment and is considered undesirable.

     

     

    OrthoCash 2020

     

  84. The use of antiglide plates for fixation of lateral malleolar fractures is most commonly associated with injuries to the

    1. peroneus brevis tendon.

    2. lateral plantar nerve.

    3. superficial peroneal nerve.

    4. sural nerve.

    5. distal tibial fibular syndesmosis.

     

    Corrent answer: 1

     

    Posterior antiglide plating of Weber type B fibular fractures provides a biomechanically stronger construct when compared to lateral plating. However, high rates (up to 43%) of hardware removal have been reported because of peroneal tendon symptoms. The peroneus brevis tendon is most commonly involved and is often injured by a prominent screw in the most distal hole of the plate.

     

     

     

    OrthoCash 2020

     

  85. During a lateral approach to repair a distal one third humeral shaft fracture, the radial nerve is found within the fracture site. The nerve is noted to be grossly in continuity and is carefully extracted. Radial nerve function is absent clinically in the postoperative period. Six weeks after surgery, a nerve velocity conduction study and electromyogram are obtained. What finding is most consistent with a first-degree injury (neurapraxia)?

    1. Normal sensory conduction in the lateral antebrachial cutaneous nerve

    2. Normal amplitude and conduction velocity in the radial sensory nerve of the forearm

    3. Normal insertion activity in the flexor carpi ulnaris muscle

    4. Fibrillations in the extensor digitorum communis muscle

    5. Strong motor unit action potentials in the triceps muscle Corrent answer: 2

    A neurapraxic, or first-degree, nerve injury is a lesion in which there is a focal conduction block within the nerve. There is no loss of continuity of the nerve fiber or axon sheath, and Wallerian degeneration is not observed. The end-organs served by the injured nerve (muscle end-plates or sensory endings) do not develop the atrophic changes typical of a more severe nerve injury. While conduction across the focus of the nerve injury is inhibited, conduction is observed within the nerve both proximal and distal to the lesion. The lateral

    antebrachial cutaneous nerve is not part of the radial nerve, and the flexor carpi ulnaris muscle is not innervated by the radial nerve. The triceps is innervated by the radial nerve proximal to the area of the nerve injury, so action potentials in the triceps are not helpful in assessing distal radial nerve function. Fibrillations in the extensor digitorum communis, which is innervated by the posterior interosseous nerve (which is a branch of the distal portion of the radial nerves), are indicative of more severe nerve injury (grade 2 or 3).

     

     

     

    OrthoCash 2020

     

  86. Patient outcome after open reduction and internal fixation of tibial plateau fractures shows that patients older than 50 years of age when compared to younger patients have

    1. equal results for all fracture types.

    2. better outcomes in high-energy fractures.

    3. better outcomes in low-energy fractures.

    4. a higher wound complication rate.

    5. worse outcomes in low-energy fractures.

     

    Corrent answer: 5

     

    Several studies have shown worse functional results in patients older than 40 or 50 years of age compared to younger patients after open reduction and internal fixation of tibial plateau fractures. Two studies showed that older patients with less severe fractures performed less favorably than younger patients with more severe injuries. Only 35% of patients older than 50 years were satisfied with their results independent of fracture type.

     

     

     

    OrthoCash 2020

     

  87. Which of the following is considered a disadvantage of knee disarticulation when compared with transfemoral amputation?

    1. Increased energy expenditure

    2. Worse end-bearing surface

    3. Asymmetry of the prosthetic knee joint

    4. More pain with weight bearing

    5. More stump complications in pediatric patients Corrent answer: 3

    The decision to perform a transfemoral amputation versus a knee disarticulation remains controversial because there are advantages and

    disadvantages to each procedure. One potential advantage of the knee disarticulation is that energy expenditure during walking may be less than that for a transfemoral amputee. Another advantage is that the knee disarticulation provides for an end-bearing residual limb. The knee disarticulation also includes complete preservation of the thigh muscles, leading to better muscular balance, and the bulbous shape of the residual limb leads to increased stability of the prosthesis. Disadvantages of knee disarticulation include limitations in knee-joint prosthetic components and concerns regarding the cosmesis of the prosthesis, as well as the asymmetry of the knee joint with respect to sitting and gait. Because knee disarticulations are relatively uncommon compared to transfemoral amputations, many prosthetists may have less familiarity with the fabrication and fit of this prosthesis. One of the more important issues with respect to knee disarticulation versus transfemoral amputation is the degree of trauma involving the soft tissue around the knee, as increased tissue trauma may lead to painful residual limbs and eventually the need for revision amputation at a higher level. If the residual limb of the knee disarticulation is unable to accommodate the distal-end weight bearing of the socket, then patients with a knee disarticulation may experience decreased levels of function compared with patients undergoing transfemoral amputation.

     

     

     

    OrthoCash 2020

     

  88. The success of internal fixation of anatomically reduced transcervical femoral neck fractures in patients can be improved by

    1. using titanium rather than stainless steel screws.

    2. starting screws distal to the level of the lesser trochanter so that they are oriented in a more valgus direction.

    3. positioning screws so that the shafts abut the cortex of the femoral neck posteriorly and inferiorly.

    4. using a dynamic hip screw device instead of multiple parallel screws.

    5. using an alignment guide to ensure that all screws are parallel.

     

    Corrent answer: 3

     

    The femoral neck region is devoid of bone in patients with advanced osteoporosis. In these patients, screws placed in the central region of the femoral neck achieve purchase at only two points, the lateral cortex and the subchondral bone of the femoral head. Loss of reduction is common postoperatively. The fracture displaces until the shaft of the screws contacts the cortical portion of the femoral neck. In an analysis of screw position relative to the femoral neck cortex, Lindequist and Tornkvist reported an 89% union rate when screws were positioned to contact the femoral neck cortex posteriorly and inferiorly. When screw contact was present at only one of these

    sites, the union rate fell to 59%, and when none of the screws were in this position the union rate was zero. It is recommended that screws should not start distal to the level of the lesser trochanter to avoid the risk of subsequent subtrochanteric fracture due to the stress riser. Whereas a dynamic hip screw may be considered for a basicervical femoral neck fracture, its use is not recommended in transcervical femoral neck fractures. While parallel screws theoretically allow compression of the fracture site, the accuracy of parallel screw placement has not been shown to effect union.

     

     

     

    OrthoCash 2020

     

  89. A 25-year-old motorcyclist has a knee dislocation that is reduced by the trauma surgeon in the emergency department. Radiographs show no fracture and a reduced knee joint. Which of the following is the most appropriate initial examination step for evaluation of a potential arterial injury?

    1. Pulse oximeter measurement at the great toe

    2. Angiography

    3. Measurement of the ankle-brachial index (ABI)

    4. Doppler ultrasound

    5. Assessment of capillary refill in the nail beds Corrent answer: 3

    A high index of suspicion should exist for an arterial injury after any knee dislocation. Due to collateral circulation around the knee, pulses may still be present, as well as normal capillary refill. Though angiography is the gold standard for assessment of both major and minor (intimal) injury to the arterial system, it is invasive and not always readily available. Assessment of the ABI can be done without specialized equipment and personnel. When the ABI (systolic BP distal to injury/systolic BP of uninjured upper extremity) is less than 0.9, consideration of invasive testing or surgical exploration is recommended.

     

     

     

    OrthoCash 2020

     

  90. A patient has a vertically and rotationally unstable hemipelvis following a motor vehicle accident. An indication for application of an anterior resuscitative pelvic external fixator is made. Two options with regard to pin insertion location are considered as seen in Figure 20.

    When compared to pins in position A, the pins in position B may be more advantageous because

     

     

     

    1. there is less risk to regional neurologic structures.

    2. of superior bone purchase.

    3. they more definitively stabilize the posterior fracture.

    4. they may be more easily applied in the emergency department setting.

    5. there is less risk of intra-articular hip joint penetration.

     

    Corrent answer: 2

     

    Pelvic external fixation can be used for the acute resuscitation of patients with pelvic fractures and for definitive treatment of certain injury patterns. Typically frames are constructed with anterosuperior half-pin placement within the iliac crest. Intracortical placement of these pins may be difficult and erroneous placement may render purchase inadequate. Recently, external fixation of the pelvic ring with half-pin placement into the dense supra-acetabular bone in the region of the anterior inferior iliac spine has gained popularity. Kim and associates, in a biomechanical model, demonstrated that anterior-inferior pin placement was biomechanically superior to conventional anterior-superior pin placement in rotationally and vertically unstable fracture patterns.

    Haidukewych and associates performed a cadaveric study that demonstrated the lateral femoral cutaneous nerve is at risk within a mean distance of 10 mm from the inferior half-pin site but the femoral nerve and femoral artery are not at risk. The average superior extent of the hip capsule was 16 mm above the joint. They suggested that these pins be inserted at least 2 cm above the hip to avoid potential hip capsule penetration. Poelstra and Kahler described a case during which the lower pins were inserted without the benefit of imaging using only palpable landmarks. However, this technique is better reserved for nonresusitative purposes permitting the use of multiplanar fluoroscopic imaging. Image guidance better ensures proper pin placement within the pelvic cortices, minimizing penetration of the hip joint and sciatic notch. No anterior external fixator, regardless of design or region of application, offers sufficient

    posterior stability to serve as definitive treatment for vertically unstable pelvic fracture variants.

     

     

     

    OrthoCash 2020

     

  91. Compared with surgically treated patients, patients with extra-articular distal third humeral shaft fractures that are treated nonsurgically with functional bracing can be expected to show which of the following findings?

    1. Similar loss of elbow motion

    2. Greater loss of elbow extension

    3. Higher rate of varus malalignment

    4. Higher rate of functionally limiting malalignment

    5. Significantly better shoulder motion Corrent answer: 1

    In a retrospective review of patients with extra-articular distal humeral shaft fractures treated surgically versus nonsurgically, the authors found that the amount of motion loss was not different between the treatment groups. Of 21 patients in the nonsurgical group, one lost 20 degrees of extension, one lost 30 degrees of extension, and one patient lost 15 degrees of flexion. Of the 19 patients in the surgical group, two patients lost 5 degrees of extension, and one each lost 10, 15, and 20 degrees of extension, respectively. One patient lost 5 degrees of flexion and one lost 15 degrees of flexion. The average loss of motion in the surgical group was 3 degrees, compared with 6 degrees in the nonsurgical group, but this difference was not significant. One hundred percent of the nonsurgically treated fractures healed. Both groups of patients regained shoulder motion within 10 degrees of normal. In the nonsurgically treated group, 10 healed with less than 10 degrees of malalignment, 6 healed with 11 to 20 degrees of malalignment, and three healed with greater than 30 degrees of malalignment, but the authors did not report any functional problems due to these deformities.

     

     

     

    OrthoCash 2020

     

  92. What is the most common complication associated with open reduction and internal fixation using a 90/90 plate configuration and olecranon osteotomy for an OTA type C2 distal humerus fracture?

    1. Nonunion of the lateral column

    2. Nonunion of the medial column

    3. Nonunion of the olecranon osteotomy

    4. Pain related to the plates

    5. Pain related to the olecranon fixation Corrent answer: 5

    The most common complications associated with open reduction and internal fixation of distal humerus fractures are those associated with repair of an associated olecranon osteotomy. Complications associated with olecranon osteotomy fixation include failure of fixation (5%) and the need for secondary removal of painful hardware (70%). Nonunion of a distal humerus fracture treated with 90/90 plating is uncommon and results from inadequate fixation, excessive soft-tissue stripping, or use of inadequate plate fixation such as one third tubular plates. Heterotopic ossification is seen in approximately 4% of cases, infection 4%, and ulnar nerve palsy 7%. Although a relatively minor complication, the need for removal of painful hardware from the olecranon osteotomy is by far the most common complication seen in these cases.

     

     

     

    OrthoCash 2020

     

  93. In a polytraumatized patient with a high lactate level, large base deficit, and pulmonary injury, what is the best initial treatment for a femoral shaft fracture?

    1. Unreamed femoral nail

    2. Reamed femoral nail

    3. Plating of the femur

    4. Skeletal traction

    5. External fixation

     

    Corrent answer: 5

     

    To prevent the "secondary hit" phenomenon in the polytraumatized patient, the use of Damage Control Orthopaedics has been accepted as the best initial treatment option until the patient has been successfully resuscitated. The use of reamed or unreamed intramedullary nails in the acute setting is to be avoided because of the possibility of increasing the patient's morbidity. Plating is time consuming and leads to blood loss which should be avoided in the severely injured patient. Skeletal traction and supine positioning are detrimental in the trauma patient. Expedient external fixation, resuscitation, and later definitive treatment with an intramedullary nail is considered to be the best choice for the polytraumatized patient with a femoral shaft fracture.

     

     

     

    OrthoCash 2020

  94. When evaluating a CT scan of a calcaneus fracture, the constant fragment shown in Figure 35 represents the

     

     

     

     

    1. sustentaculum tali.

    2. tuberosity.

    3. anterior process.

    4. lateral wall.

    5. posterior facet.

     

    Corrent answer: 1

     

    The constant fragment refers to the sustentaculum tali fragment that stays attached to the talus through ligamentous attachments. It is the nondisplaced fragment in relation to the rest of the calcaneus. During surgical reduction, the lateral articular fragment(s) and the tuberosity are reduced and fixed to this constant fragment.

     

     

     

    OrthoCash 2020

     

  95. An otherwise healthy 25-year-old man with an isolated closed mid-diaphyseal femoral fracture undergoes intramedullary nailing. Compared with nonreamed nailing, reamed femoral nailing is associated with a higher rate of

    1. union.

    2. symptomatic pulmonary complications.

    3. infection.

    4. transfusion requirements.

    5. required secondary procedures.

    Corrent answer: 1

     

    Bhandari and associates, in a meta-analysis, concluded that sufficient evidence exists to suggest that reamed intramedullary nailing of lower extremity long bone fractures significantly reduces rates of nonunion and implant failure in comparison with nonreamed nailing. Tornetta and Tiburzi, in a prospective randomized study, determined that reamed canal preparation led to faster healing of distal fractures treated with statically locked intramedullary nails.

    Blood loss was greater in the reamed group, but this did not translate into increased transfusion requirements. In this series, there was no advantage to nail insertion without reaming. In a prospective randomized multicenter study, the overall incidence of acute respiratory distress syndrome (ARDS) was found to be low with primary stabilization of femoral shaft fractures with intramedullary nailing. There was no difference in the incidence of ARDS between the reamed and unreamed groups. In a retrospective study performed by Handolin and associates, intramedullary nailing of long bone fractures in patients with multiple injuries and with a coexisting pulmonary contusion did not impair pulmonary function or outcome. No study has convincingly demonstrated an increased trend toward infection with reamed femoral intramedullary nailing.

     

     

     

    OrthoCash 2020

     

  96. A 35-year-old right-handed man sustained a posterior elbow dislocation without fracture 6 months ago. He now reports progressive loss of elbow motion. He denies pain and has no numbness in the median or ulnar nerve distributions. Examination reveals an extension loss of 80 degrees and flexion of 110 degrees, a negative Tinel's sign over the ulnar nerve, and full pronation and supination. Radiographs reveal a mature calcification anterior and lateral in the elbow joint.

    The articular surfaces appear normal as seen in Figures 39a and 39b. He requests treatment to improve elbow range of motion. What is the most appropriate management?

     

     

     

     

     

     

    1. Arthroscopic resection of heterotopic ossification

    2. Open resection and capsulectomy through an anterior approach

    3. Open resection and capsulectomy through a lateral approach

    4. Indomethacin and low-dose radiation therapy

    5. Manipulation under anesthesia Corrent answer: 3

    The patient sustained an apparent simple elbow dislocation that is complicated by the development of anterior heterotopic ossification and loss of elbow motion. The 30-degree range of motion of the elbow limits function even though he has no pain. Open resection and capsulectomy through a lateral approach offers exposure both anteriorly as well as posteriorly. Heterotopic ossification is defined as the inappropriate formation of mature lamellar bone in nonosseous locations. It may present as soft-tissue swelling with progressive loss of elbow motion. Radiographs reveal an initial fluffy ill-defined density that matures over 3 to 6 months, developing sharp margins. Usually by

    the time changes appear on the radiograph, prophylaxis with nonsteroidal anti-inflammatory drugs or low-dose external beam radiation of 700cGy is not effective. Laboratory tests and bone scans provide no prognostic value and are no longer recommended in the evaluation of elbow heterotopic ossification. In this patient, range of motion has been limited for a number of months and although the heterotopic ossification is anterior, the posterior capsule will most likely need to be released. The limited motion and tight capsule increase the risk of complications during arthroscopy. While arthroscopic techniques are continuing to improve, arthroscopic resection is best left to highly experienced elbow arthroscopic surgeons. The open anterior approach will allow good exposure, but the dissection is tedious and does not allow access to the posterior joint without a separate incision. Anterior heterotopic ossification is usually approached from the lateral or medial sides, and both in extensive cases. These approaches allow access to both the anterior and posterior compartments of the joint. Manipulation of the elbow joint in this situation is contraindicated and could cause joint injury or fracture.

     

     

     

    OrthoCash 2020

     

  97. Plate fixation of olecranon fractures is recommended over tension band wire fixation when

    1. the fracture is proximal and only involves the olecranon tip.

    2. the fracture is transverse.

    3. the fracture is comminuted.

    4. the fracture is widely displaced.

    5. there is associated osteoporosis.

     

    Corrent answer: 3

     

    Tension band wire fixation of olecranon fractures is recommended for fracture patterns that are proximal to the coronoid process and are relatively transverse to withstand compressive forces. When comminution is present, a neutralization technique such as plating is preferred over a compressive technique such as tension band wire fixation. Such neutralization plating, if performed correctly, does not have the risk of narrowing the sigmoid notch as tension band wire fixation would. Fractures of the tip of the olecranon, transverse fractures, fractures associated with osteoporosis, and displaced fractures are all relative indications for tension band wire fixation.

     

     

     

    OrthoCash 2020

  98. A patient sustains an injury to the pelvis as a result of high-energy trauma. A radiographic and clinical assessment is performed. A lateral view of the pelvis will best identify

    1. pubic symphysis diastasis.

    2. sagittal plane pelvic rotation.

    3. sacroiliac diastasis.

    4. spinopelvic dissociation.

    5. a vertical transforaminal sacral fracture.

     

    Corrent answer: 4

     

    Radiographic assessment of the pelvis is an essential part of the initial evaluation of trauma patients. In addition to an AP radiograph, 40-degree caudal (inlet) and 40-degree cephalad (outlet) views are obtained. The caudal/inlet view is useful for detecting the integrity of the pelvic ring because it shows the sacroiliac joints and the sacrum, which can help detect either sacroiliac dislocations or sacral fractures and their displacement in the anterior-posterior plane. The outlet view detects superior or inferior displacement and sagittal plane flexion or rotation of the pelvis. A lateral radiograph of the sacrum identifies transverse fractures of the sacrum and coccyx and the degree of their displacement. A lateral view should be obtained in instances of bilateral sacral fractures and U-shaped sacral fractures in which there are bilateral vertical fractures through the sacral foramina that connect with a transverse fracture line between the second or third sacral segments. Some patients with these fractures have sacral fracture-dislocations with spinopelvic dissociation and a high incidence of neurologic injuries. The lateral view discloses the extent of displacement of the upper sacral segment (usually S2 forward on S3) and the resultant kyphotic deformity.

     

     

     

    OrthoCash 2020

     

  99. The essential lesion in recurrent or persistent instability following simple dislocation of the elbow typically involves which of the following structures?

    1. Medial collateral ligament

    2. Lateral ulnar collateral ligament

    3. Coracohumeral ligament

    4. Anterior joint capsule

    5. Posterior joint capsule Corrent answer: 2

    The lateral ulnar collateral ligament is the essential lesion in recurrent or persistent instability following simple dislocations of the elbow. Simple elbow dislocations are usually stable and may be managed by a short period of immobilization followed by early range of motion. Treatment of dislocations resulting in persistent instability frequently involves focusing on the lateral ulnar collateral ligament. The medial collateral ligament is repaired only if treatment of associated fractures and lateral collateral ligament injury does not restore stability.

     

     

     

    OrthoCash 2020

     

  100. Disadvantages of anterior-inferior plate fixation for acute clavicular fractures relative to superior plating include

    1. more prominent implants leading to higher rate of reoperation for implant removal.

    2. a higher nonunion rate.

    3. inferior fixation due to shorter screw lengths laterally.

    4. an increased need to detach the deltoid origin.

    5. an increased risk for injury to subclavian structures.

     

    Corrent answer: 4

     

    Anterior-inferior plate fixation of midshaft clavicular fractures has evolved to be an alternative plate location compared to superior plating. The advantages of anterior-interior plating are reduced prominence of the hardware compared to the subcutaneous superior plates; the potential for placement of longer screws as the clavicle is wider front to back than top to bottom, especially laterally; and a potential for decreased risk to the subclavian structures. A relative disadvantage of anterior-inferior plating is a need to detach a small portion of the deltoid origin. Union rates for anterior-inferior plating are similar to those with superior plating.

     

     

     

    OrthoCash 2020

     

  101. A 37-year-old man who fell while running upstairs is seen in the emergency department. He reports pain and is unable to bear weight on the affected lower extremity. Examination reveals considerable soft-tissue swelling limited to the hindfoot region. A lateral radiograph is shown in Figure 46. He has no concomitant injuries and no past medical history. There is no sensory impairment or signs or symptoms of compartment syndrome. Preferential management should consist of

     

     

     

    1. a splint, reassessment as an outpatient in 5 to 7 days, and delayed fixation as soft-tissue status allows.

    2. admission, elevation, and application of a pneumatic compression foot device followed by delayed fixation via an extensile lateral approach when soft-tissue swelling resolves.

    3. immediate reduction and fixation via an extensile approach.

    4. immediate fixation with a minimally invasive technique.

    5. immediate cast application in ankle plantar flexion.

     

    Corrent answer: 4

     

    Avulsion tongue-type fractures of the calcaneal tuberosity demand urgent intervention with prompt reduction and secure fixation to prevent retrocalcaneal soft-tissue necrosis. Rapid full-thickness necrosis may ensue if treatment is delayed and the significance of this injury underappreciated.

    Proper soft-tissue handling and strategically positioned percutaneous or small incisions combined with compression screw fixation (engaging the opposite cortex) remains the most prudent tactic. An extensile lateral approach should be avoided in the face of significant soft-tissue swelling.

     

     

     

    OrthoCash 2020

     

  102. The implant shown in Figures 47a and 47b is introduced submuscularly employing a minimally invasive technique. A percutaneous method of screw insertion is used distally. What nerve is most at risk?

     

     

     

    1. Sural

    2. Saphenous

    3. Superficial peroneal

    4. Deep peroneal

    5. Posterior tibial

     

    Corrent answer: 3

     

    Minimally invasive methods used for stabilizing complex periarticular fractures continue to evolve. Encouraging results suggest a diminished threat to the soft tissues and enhanced preservation of osseous blood supply. Contemporary locking implants combined with indirect reduction lead to desirable biomechanical and biologic environments for osseous and soft-tissue healing. Deangelis and associates, in a cadaveric tibial study, demonstrated the superficial peroneal nerve to be at significant risk during percutaneous screw placement in very distal targeted holes (within laterally applied tibial locking plates). Use of a larger incision and cautious dissection to the plate in this region were encouraged to minimize risk to this structure.

     

     

     

    OrthoCash 2020

  103. A 32-year-old man sustains an elbow dislocation while playing flag football. Postreduction radiographs are shown in Figures 48a through 48c. Which of the following treatment options is most appropriate?

     

     

     

     

     

     

     

     

    1. Cast treatment for 3 weeks, followed by mobilization

    2. Radial head excision, followed by early mobilization

    3. Closed reduction and pinning

    4. Open reduction and internal fixation with repair of the lateral collateral ligament complex

    5. Radial head arthroplasty with repair of the lateral collateral ligament complex

    Corrent answer: 5

     

    The radiographs reveal a markedly comminuted radial head fracture that occurred in conjunction with an elbow dislocation. One fragment of the head is located in the soft tissues on the medial side of the elbow. In the setting of an elbow dislocation with a comminuted radial head fracture, radial head arthroplasty is the preferred option. With the introduction of modular implants, matching the size of the implant has become considerably easier. Repair of the lateral collateral ligament is important to allow for an early motion rehabilitation protocol. Sometimes the radial head can be reconstructed, but the fixation is technically difficult and often interferes with an early motion rehabilitation protocol. As such, the elbow can become very stiff. Whereas radial head excision can be an effective treatment in an isolated radial head fracture, this treatment option is generally avoided in the setting of an elbow

    dislocation. Pinning or casting treatment regimens in this case scenario are best avoided.

     

     

     

    OrthoCash 2020

     

  104. A rocker-soled shoe, toe filler, and extended carbon fiber foot plate are appropriate postamputation devices for which of the following foot amputations?

    1. Transmetatarsal

    2. Lisfranc

    3. Boyd

    4. Pirigoff

    5. Syme

     

    Corrent answer: 1

     

    Transmetatarsal amputation has enough length and necessary muscle attachments to be functional in a shoe. All the other amputations are more proximal and require a prosthetic device that incorporates the foot and controls the ankle. Studies have shown that a full-length shoe with a total contact insert and a rigid rocker bottom sole was the most effective footwear combination in reducing peak plantar pressure on the distal residuum of patients with diabetes and a transmetatarsal amputation.

     

     

     

    OrthoCash 2020

     

  105. A 25-year-old man involved in a motor vehicle accident sustains the injury shown in Figures 53a through 53d. Soft-tissue swelling has resolved. There are no neurovascular deficits and the soft-tissue envelope is uncompromised. The preferred management should include

     

     

     

     

     

     

     

     

     

     

     

    1. hybrid frame external fixation.

    2. dual plate fixation with two surgical approaches.

    3. dual plate fixation with a single surgical approach.

    4. a single lateral locking plate.

    5. transarticular external fixation.

     

    Corrent answer: 2

     

    This injury pattern demonstrates components of split and depression laterally and combined sagittal and coronally oriented fractures medially. A lateral locking plate alone will not adequately address either the lateral or medial fracture patterns. This implant is best reserved for bicondylar fractures (although fractured and separated, each condyle is largely intact) with metaphyseal comminution. The injury seen in this patient is most predictably managed with two plates inserted via two approaches. In addition to a construct with a raft of screws laterally, a second plate posteromedially is required to neutralize shear forces on the posteromedial fragment. A two-

    incision technique (lateral and posteromedial) allows for implant insertion while respecting the soft-tissue envelope. A single incision approach to introduce medial and lateral plates is fraught with wound healing complications. Hybrid external fixation is most often reserved for fairly simple bicondylar fractures with metaphyseal comminution and coexistent soft-tissue envelope concerns.

    Transarticular external fixation is often used as an initial first stage method of osseous and soft-tissue stabilization. It is then typically followed by definitive articular and metaphyseal reconstruction when soft tissues allow (usually 1 to 2 weeks later).

     

     

     

    OrthoCash 2020

     

  106. Which of the following patients is more likely to have an overall poorer outcome following a lower extremity amputation?

    1. A male patient with a below-the-knee amputation

    2. A male patient with a through-the-knee amputation performed in a zone of injury

    3. A male patient treated with a myoplasty at the time of wound closure

    4. A female patient with an above-the-knee amputation

    5. A female patient using a less technologically sophisticated prosthesis Corrent answer: 2

    Many factors influence the outcome of lower extremity amputations with worse outcomes noted in patients with lower socioeconomic status, preexisting medical conditions, and low self-efficacy. Patients with amputations performed in a zone of injury, especially if this was a through-the-knee amputation, have a significantly poorer outcome than those with either above-the-knee or below-the-knee amputations. Patients with above-the-knee amputations have similar outcomes to those with below-the-knee amputations, although those with below-the-knee amputations have faster self-selected walking speeds. Gender and sophistication of the prosthesis have no bearing on outcome. The type of muscle anchoring technique used also plays no role in outcome.

     

     

     

    OrthoCash 2020

     

  107. Which statement is true with respect to acetabular fracture surgery as the time between injury and surgery increases?

    1. decreased chance of anatomic fracture reduction

    2. decreased risk of heterotopic ossification

    3. decreased rate of neurologic injury

    4. decreased rate of infection

    5. decreased rate of multi-organ failure Corrent answer: 1

    Madhu et al showed time to surgery was a significant predictor of radiological and functional outcome for both elementary and associated displaced fractures of the acetabulum. Both anatomic reduction and functional outcome significantly worsened as time to surgery increased. It was found anatomic reduction was more likely when surgery was within 15 days for elementary fracture and 5 days for associated.

     

    Incorrect answers:

    2: No data exists showing a decrease in heterotopic ossification as time to surgery increases.

    3: Neurologic injury is more associated with the initial injury.

    4,5: Multi-organ failure was not commented on, but infection showed a trend towards being more likely with longer time to surgery.

     

     

     

    OrthoCash 2020

     

  108. What is the best option for treatment of the fracture shown in Figure 55?

     

     

     

     

    1. A 135-degree sliding hip screw device

    2. A 150-degree sliding hip screw device

    3. A cephalomedullary intramedullary nail

    4. External fixation using hydroxyapatite-coated pins

    5. Hip arthroplasty

    Corrent answer: 3

     

    According to Haidukewych and associates, treatment of reverse obliquity trochanteric femoral fractures with 95-degree fixed angle plates or with cephalomedullary intramedullary nails provides results superior to sliding hip screw devices. Moroni and associates demonstrated that external fixation with hydroxyapatite-coated pins provides satisfactory results for complex trochanteric fractures. Hip arthroplasty is not indicated for reverse obliquity fractures because this procedure still requires healing of the greater trochanteric fracture fragment to the shaft.

     

     

     

    OrthoCash 2020

     

  109. Fixation of a distal supracondylar femoral fracture with metaphyseal comminution is elected. When compared to conventional open plating, percutaneous submuscular reduction and fixation with a precontoured locking plate is associated with a higher incidence of

    1. nonunion.

    2. compromise to medullary perfusion.

    3. compromise to periosteal perfusion.

    4. malreduction.

    5. violation of the perforator vessels.

     

    Corrent answer: 4

     

    Biological methods of fracture reduction and fixation continue to evolve. Minimally invasive methods offer a reduction in surgical trauma, enhanced union, and a minimization of complications associated with conventional open plating. Complex periarticular fractures of the lower extremity that cannot be adequately managed with intramedullary techniques of stabilization are particularly amenable to these techniques. Alignment is achieved with indirect reduction and fixation performed with the application of extraperiosteal plates. Farouk and associates performed injection dye studies suggesting preservation of perforators and diminished compromise to both endosteal and periosteal perfusion when compared to open plating techniques. This trend is not without potentional complications, particularly with regard to reduction. Krettek and associates, in two studies, demonstrated diminished rates of infection and nonunion but acknowledged a tendency toward malalignment, particularly rotational.

     

     

     

    OrthoCash 2020

  110. A patient has a tibial shaft fracture and is suspected of having a compartment syndrome involving the deep posterior compartment.

    Associated signs and symptoms would include paresthesias over the

     

    1. dorsum of the foot and pain with passive toe flexion.

    2. dorsum of the foot and pain with passive toe extension.

    3. plantar foot and pain with passive toe flexion.

    4. plantar foot and pain with passive toe extension.

    5. entire foot and pain with active toe extension.

     

    Corrent answer: 4

     

    A compartment syndrome of the deep posterior compartment causes symptoms related to structures running through that compartment. The deep posterior compartment includes the tibialis posterior, flexor hallucis longus, flexor digitorum longus, popliteus muscle, as well as the posterior tibial artery and the tibial nerve. Elevated pressures in this compartment would cause paresthesias in the distribution of the tibial nerve (plantar aspect of the foot) and would cause associated pain with passive stretch of the muscles in the compartment (great toe extension).

     

     

     

    OrthoCash 2020

     

  111. Which of the following tibial plateau fracture patterns is most likely to benefit from a temporary spanning external fixator?

    1. Schatzker I with minimal displacement

    2. Schatzker II with significant depression

    3. Schatzker III with an associated meniscal tear

    4. Schatzker IV with minimal displacement

    5. Schatzker VI with shortening Corrent answer: 5

    Spanning external fixation for proximal tibial plateau fractures is indicated for fracture patterns that are axially unstable and for those that have associated unstable knee subluxation/dislocation. Schatzker types I, II, and III are unicondylar fractures where the medial side remains intact. These are axially stable fracture patterns that do not benefit significantly from temporary spanning external fixation. Schatzker type IV fractures are isolated medial plateau fractures where the lateral side remains intact and are also axially stable and do not benefit significantly from spanning external fixation.

    However, Schatzker type IV fractures are often associated with knee subluxation/dislocation. In these fractures, the medial side is usually widely displaced and the femur is moving with the medial side leaving the lateral tibial

    plateau subluxated laterally. These fractures would benefit from spanning external fixation. Schatzker type VI fractures are usually high-energy fractures. They are bicondylar and axially unstable, and often benefit from temporary spanning external fixation. Once the soft-tissue envelope has recovered, definitive fixation can follow with removal of the spanning external fixator.

     

     

     

    OrthoCash 2020

     

  112. A 24-year-old man involved in a motorcycle accident sustained multiple injuries including a closed tibia fracture, liver laceration, and blunt chest trauma. He has a blood pressure of 80/50 mm Hg, a pulse of 130, and respirations of 20. He is intubated and being resuscitated. The calf is very swollen with compartment pressures: anterior 25 mm Hg, lateral 24 mm Hg, deep posterior 21 mm Hg, and superficial posterior 21 mm Hg. What is the most appropriate treatment?

    1. Observation

    2. Splinting and limb elevation above the heart

    3. Splinting and no limb elevation

    4. Immediate fasciotomy

    5. Continuous pressure monitoring with an indwelling catheter Corrent answer: 4

    A compartment syndrome is best diagnosed with a "deltaP" (diastolic pressure minus compartment pressure) of less than or equal to 30 mm Hg. This patient is hypotensive and the "deltaPs" are all less than 30 mm Hg. Emergent fasciotomy is the preferred treatment.

     

     

     

    OrthoCash 2020

     

  113. A patient sustained a fracture of the left acetabulum. A single axial CT scan from a two-dimensional study is shown in Figure 61. This fracture pattern is best classified as

     

     

     

    1. both column.

    2. anterior column.

    3. posterior column.

    4. transverse.

    5. anterior column-posterior hemitransverse.

     

    Corrent answer: 4

     

    A transverse fracture divides the innominate bone into two portions. The fracture plane is horizontal (or semihorizontal) through the acetabulum at a variable proximal distal level. The superior segment retains a portion of the acetabular roof and the lower ischiopubic segment, the intact obturator foramen. A sagittally oriented fracture line is typically seen on axial view assessment.

     

     

     

    OrthoCash 2020

     

  114. An 18-year-old girl sustains a closed tibial fracture when she is pinned between the bumpers of two cars. You suspect that she has compartment syndrome and you prepare her for fasciotomies in the operating room. Which of the following muscles is not released by a standard medial-sided incision?

    1. Tibialis posterior

    2. Extensor hallucis longus

    3. Soleus

    4. Flexor digitorum longus

    5. Popliteus

     

    Corrent answer: 2

    The tibialis posterior, flexor digitorum longus, and popliteus are all located in the deep posterior compartment. The soleus is in the superficial posterior compartment. All are easily released through a medial surgical approach. The extensor hallucis longus is located in the anterior compartment and therefore cannot be released through a medial-sided incision.

     

     

     

    OrthoCash 2020

     

  115. A 73-year-old man sustains the fracture shown in Figure 62. Which of the following factors or combination of factors puts this patient at highest risk for nonunion if nonsurgical management is used?

     

     

     

     

    1. Advanced age and shortening

    2. Fracture displacement and rotation

    3. Mid diaphyseal fracture

    4. Male gender

    5. Fracture comminution, fracture displacement, and advanced age Corrent answer: 5

    Most textbooks and early publications list the incidence of complications of nonsurgical treatment of clavicle fractures as very low. However, recent studies on this topic have found an entirely different picture. The studies show that patients reported shoulder weakness and fatigability, upper extremity dysesthesia, and shoulder asymmetry with an incidence of 31%. Indications for surgery in the past have included open fractures, associated neurovascular injury, and widely displaced fractures tenting the skin. Fractures with more than 2 cm of shortening and comminuted fractures with significant displacement have been associated with poor outcomes. Nonunion after nonsurgical management was found to be more common in the study by

    Robinson and associates in displaced comminuted fractures, in patients with advanced age and female gender.

     

     

     

    OrthoCash 2020

     

  116. A 66-year-old woman who previously underwent hemiarthroplasty 2 years ago for a fracture continues to have severe pain and loss of motion despite undergoing physical therapy. A radiograph is shown in Figure 2. What is the most likely reason that this patient has failed to improve her motion?

     

     

     

     

    1. She was noncompliant in physical therapy.

    2. The original surgery should have included resurfacing the glenoid.

    3. The humeral head was too large.

    4. The humeral component was placed too proud.

    5. The tuberosities are malpositioned.

     

    Corrent answer: 5

     

    The radiograph shows tuberosity malposition. The effect of improper prosthetic placement has also been associated with poor outcomes. However, the malposition of the tuberosity seen on the radiograph clearly explains loss of motion in this patient. It has been demonstrated that the functional results after hemiarthroplasty for three- and four-part proximal humeral fractures appear to be directly associated with tuberosity osteosynthesis. The most significant factor associated with poor and unsatisfactory postoperative functional results was malposition and/or migration of the tuberosities. Factors associated with a failure of tuberosity osteosynthesis in a recent study were poor initial position of the prosthesis, poor position of the greater tuberosity,

    and women older than age 75 years (most likely with osteopenic bone). Greater tuberosity displacement has been identified by Tanner and Cofield as being the most common complication after prosthetic arthroplasty for proximal humeral fractures. Furthermore, Bigliani and associates examined the causes of failure after prosthetic replacement for proximal humeral fractures and found that although almost all failed cases had multiple causes, the most common single identifiable reason was greater tuberosity displacement.

     

     

     

    OrthoCash 2020

     

  117. A 40-year-old woman underwent an arthroscopic acromioplasty and mini-open rotator cuff repair 4 weeks ago. At follow-up examination, the incision is painful, erythematous, and draining fluid. The patient is febrile and has an elevated WBC count. What infectious organism should be under high suspicion of causing this outcome?

    1. Escherichia coli

    2. Streptococcus viridans

    3. Oxalophagus oxalicus

    4. Proprionobacter acnes

    5. Enterococcus faecalis

     

    Corrent answer: 4

     

    Proprionobacter acnes has been a leading cause of indolent shoulder infections. During shoulder arthroscopy, the arthroscopic fluid may actually dilute the shoulder preparation and lead to a higher rate of infection during subsequent mini-open rotator cuff repair surgery. The remaining bacteria listed are rarely associated with shoulder infections after arthroscopy.

     

     

     

    OrthoCash 2020

     

  118. A 30-year-old right hand-dominant woman is seen in the trauma unit after a high-speed motor vehicle accident. She sustained a right shoulder anterior dislocation that is gently reduced under sedation. A CT scan is shown in Figure 3. If left untreated, the patient is at greatest risk for

     

     

     

    1. axillary neuropathy.

    2. recurrent instability.

    3. shoulder girdle weakness.

    4. luxatio erecta.

    5. biceps tendinitis.

     

    Corrent answer: 2

     

    Large, displaced anterior inferior glenoid rim fractures predispose patients to recurrent anterior instability due to loss of the normal concavity compression effect of the glenoid. These defects require open reduction and internal fixation to reestablish shoulder stability. Although intra-articular fractures may lead to arthrosis, recurrent instability is more common.

     

     

     

    OrthoCash 2020

     

  119. A patient reports hyperesthesia over the base of the thenar eminence following volar locked plating of a distal radius fracture. A standard volar approach of Henry was used. What is the most likely cause of the hyperesthesia?

    1. Complex regional pain syndrome

    2. Wartenberg’s syndrome

    3. Carpal tunnel syndrome

    4. Palmar cutaneous nerve injury

    5. C7 radiculopathy

    Corrent answer: 4

     

    The palmar cutaneous branch of the median nerve separates from the median nerve approximately 4 to 6 cm proximal to the wrist crease and travels between the median nerve and the flexor carpi radialis tendon. It supplies the skin of the thenar region. This nerve is at risk for injury with retraction of the digital flexor tendons in plating the distal radius. Wartenberg’s syndrome is compression of the superficial radial nerve which innervates the dorsum of the thumb and the first dorsal web space. Carpal tunnel syndrome causes dysesthesias of the thumb, index, and/or middle fingers. C7 radiculopathy affects the index and middle fingers.

     

     

     

    OrthoCash 2020

     

  120. Which of the following surgical devices employed for stabilization of the sternoclavicular joint is associated with the highest incidence of life-threatening complications?

    1. Percutaneous pins

    2. Cannulated screws

    3. Cerclage wire

    4. Balser plate

    5. AO locking plate Corrent answer: 1

    Numerous reports have documented serious complications including death from migration of intact or broken Kirschner wires or Steinmann pins into hilar structures such as the heart, pulmonary artery, and the aorta.

     

     

     

    OrthoCash 2020

     

  121. Figure 11a shows the clinical photograph of a 46-year old woman who reports a 3-week history of pain and a “lump” at the base of her neck. She is otherwise in good health and denies any trauma. A 3-D reconstruction CT is shown in Figure 11b. What is the most likely diagnosis?

     

     

     

     

     

     

    1. Unreduced posterior sternoclavicular dislocation

    2. Congenital hypoplasia of the medial clavicle

    3. Postmenopausal arthritis of the sternoclavicular joint

    4. Sternoclavicular hyperostosis

    5. Spontaneous subluxation of the right sternoclavicular joint Corrent answer: 5

    Spontaneous subluxation of the sternoclavicular joint occurs without any significant trauma. It is usually accentuated by placing the extremity in an overhead position. Discomfort usually resolves within 4 to 6 weeks with nonsurgical management.

     

     

     

    OrthoCash 2020

     

  122. A 65-year-old woman fell onto her outstretched right arm and immediately had pain. She has a history of osteoporosis. Examination of the right arm reveals lateral arm swelling, ecchymosis, and she is unable to move the elbow due to pain. Her neurovascular status is

    intact. Radiographs are shown in Figures 14a and 14b. Appropriate treatment should include

     

     

     

     

    1. splint immobilization and early range-of-motion exercises.

    2. radial head excision.

    3. anatomic metallic radial head arthroplasty.

    4. radial head open reduction and internal fixation.

    5. anconeus interposition arthroplasty.

     

    Corrent answer: 3

     

    Comminuted, displaced radial head fractures (Hotchkiss type 3) require anatomic metallic radial head arthroplasty to regain function. Radial head excision has led to catastrophic sequelae including chronic wrist pain, elbow instability, and proximal radius migration. Immobilization, internal fixation, or anconeus arthroplasty are not recommended at this time because of the potentially poorer outcomes.

     

     

     

    OrthoCash 2020

     

  123. A patient sustains a transection of the posterior cord of the brachial plexus from a knife injury. This injury would affect all of the following muscles EXCEPT?

    1. Subscapularis

    2. Latissimus dorsi

    3. Supraspinatus

    4. Teres minor

    5. Brachioradialis

     

    Corrent answer: 3

     

    The posterior cord of the brachial plexus gives rise to the 1) upper subscapular nerve 2) lower subscapular nerve 3) thoracodorsal nerve 4) axillary nerves 5) radial nerve. The upper subscapular nerve innervates the subscapularis. The lower subscapular nerve innervates teres major and also subscapularis. The thoracodorsal nerve innervates latissimus dorsi. The axillary nerves innervates deltoid and teres minor. The radial nerve innervates the triceps, brachioradialis, wrist extensors, and finger extensors. The supraspinatus is innervated by the suprascapular nerve off the upper trunk and therefore would not be affected by an injury to the posterior cord. The anatomy of the brachial plexus is shown in Illustration A.

     

     

     

     

     

     

    OrthoCash 2020

     

  124. A 25-year-old left hand-dominant man has severe left shoulder pain after being involved in a high-speed motor vehicle accident.

    Examination reveals that he is unable to move the left shoulder. His neurovascular status is intact in the entire left upper extremity. A radiograph is shown in Figure 19. What is the most appropriate surgical management of this injury?

     

     

     

    1. Arthroscopic reduction and fixation

    2. Percutaneous pinning

    3. Open reduction and internal fixation

    4. Hemiarthroplasty with tuberosity reconstruction

    5. Reverse shoulder arthroplasty Corrent answer: 3

    In this young patient, every attempt must be made to retain the native proximal humerus; therefore, open reduction and internal fixation should be attempted of both the articular segment and tuberosities to the humeral shaft. This is best accomplished through an open approach. Shoulder arthroplasty should be reserved for the elderly and for failed internal fixation.

     

     

     

    OrthoCash 2020

     

  125. A 32-year-old woman sustained an elbow dislocation, and management consisted of early range of motion. Examination at the 3-month follow-up appointment reveals that she has regained elbow motion but has a weak pinch. A clinical photograph is shown in Figure

    21. What is the most likely diagnosis?

     

     

     

    1. Flexor pollicis longus rupture

    2. Median nerve palsy

    3. Ulnar nerve palsy

    4. Anterior interosseous nerve palsy

    5. Posterior interosseous nerve palsy Corrent answer: 4

    The photograph shows the characteristic attitude of the hand when an anterior interosseous nerve palsy is present. The patient is unable to flex the interphalangeal joint to the joint of the thumb. Anterior interosseous nerve palsies are often misdiagnosed as tendon ruptures.

     

     

     

    OrthoCash 2020

     

  126. A 22-year-old right hand-dominant man who fell off his motorcycle onto the tip of his right shoulder 2 weeks ago now reports pain and difficulty raising his right arm. Examination reveals tenderness and gross movement over the lateral scapular spine and severe weakness during resisted abduction. A radiograph and 3D-CT scan are shown in Figures 24a and 24b. What is the next most appropriate step in management?

     

     

     

     

     

     

    1. Open reduction and internal fixation

    2. External bone stimulator

    3. Ninety-degree abduction splint

    4. Arthroscopic acromioplasty

    5. Fragment excision

     

    Corrent answer: 1

     

    The patient has a displaced scapular spine fracture that has resulted in shoulder weakness from a poor deltoid lever arm. The downward tilt may lead to subacromial impingement and rotator cuff dysfunction. Open reduction and internal fixation would best allow normal deltoid and shoulder function. Bone

    stimulators and abduction bracing may lead to healing but in a malunited position. Arthroscopic acromioplasty and fragment excision should be avoided.

     

     

     

    OrthoCash 2020

     

  127. A 30-year-old man has pain in the left arm after a motor vehicle accident. His neurovascular examination is intact, and radiographs are shown in Figures 25a and 25b. What is the best course of management?

     

     

     

     

    1. Closed reduction and cast immobilization for 4 weeks, followed by therapy directed at regaining motion

    2. Open reduction and internal fixation of the olecranon fracture, functional bracing of the humeral fracture, and therapy directed at regaining motion initiated at 2 weeks after surgery

    3. Open reduction and internal fixation of the olecranon and humeral fractures, followed by therapy directed at regaining motion

    4. Open reduction and internal fixation of the olecranon and humeral fractures, and splint immobilization for 4 weeks followed by therapy directed at regaining motion

    5. Open reduction and internal fixation of the olecranon fracture, functional bracing of the humeral fracture, and therapy directed at regaining motion initiated at 4 weeks after surgery

    Corrent answer: 3

    The floating elbow is best managed with early open reduction and internal fixation of the humeral and forearm fractures, followed by early range of motion. These fractures predispose the elbow to stiffness, and early range of motion is recommended.

     

     

     

    OrthoCash 2020

     

  128. A patient who underwent open reduction and internal fixation of an olecranon fracture 2 months ago now reports painless limitation of motion. Examination reveals a well-healed incision and a flexion-extension arc from 40 degrees to 80 degrees. The patient has been performing home exercises. Radiographs are shown in Figures 26a and 26b. What is the most appropriate treatment?

     

     

     

     

     

     

    1. Continued observation and home therapy

    2. Radiation therapy, followed by aggressive range-of-motion exercises

    3. Formal physical therapy and static progressive splinting

    4. Revision open reduction and internal fixation and capsular release

    5. Manipulation under anesthesia Corrent answer: 3

    The radiographs do not show an articular malunion. Treatment is directed at the soft-tissue contracture and should begin with formal physical therapy and static progressive splinting. Radiation therapy is effective in the perioperative period and is indicated when ectopic bone formation is a concern.

     

     

    OrthoCash 2020

     

  129. A 40-year-old unrestrained passenger reports chest wall pain after a motor vehicle accident. Which of the following structures is most important in preventing the injury shown in Figure 33?

     

     

     

     

    1. First rib

    2. Intra-articular disk ligament

    3. Costoclavicular ligament

    4. Interclavicular ligament

    5. Posterior sternoclavicular joint capsule Corrent answer: 5

    Through cadaveric study, Spencer and associates measured anterior and posterior translation of the sternoclavicular joint. The study demonstrated that the posterior sternoclavicular joint capsule is the most important structure for preventing both anterior and posterior translation of the sternoclavicular joint.

     

     

     

    OrthoCash 2020

     

  130. A 45-year-old male falls onto his left shoulder while biking and an injury radiograph is shown in Figure A. He elects for nonoperative treatment. What is the most likely clinical outcome at one year after injury?

     

     

     

    1. Symmetric cosmesis of shoulders

    2. Decreased shoulder motion

    3. Symptomatic nonunion

    4. Shoulder instability

    5. Decreased shoulder strength and endurance Corrent answer: 5

    Patients who have nonoperative treatment of displaced midshaft clavicle fractures have significant decreases in both strength and endurance to approximately 80% of the contralateral side as described by the McKee article. There was a trend correlating shortening >2cm with poor outcome (p=0.06).

    Motion was found to be preserved.

     

    In the Canadian Orthopaedic Trauma Society's landmark randomized control trial of operative versus nonoperative treatment for displaced clavicle fractures, patients treated non-operatively had lower subjective outcomes scores, slower rates to union, more nonunions, more symptomatic malunions, and were less satisfied with the appearance of their shoulder. There were more hardware related complications in the operatively treated group.

     

    The second McKee article describes improvements in subjective outcome scores after midshaft clavicle malunion corrective osteotomy.

     

    Figure A shows a comminuted, displaced midshaft clavicle fracture.

     

     

     

    OrthoCash 2020

     

  131. A previously healthy 65-year-old woman has a closed fracture of the right clavicle after falling down the basement stairs. Examination reveals good capillary refill in the digits of her right hand. Radial and ulnar pulses are 1+ at the right wrist compared with 2+ on the

    opposite side. In the arteriogram shown in Figure 36, the arrow is pointing at which of the following arteries?

     

     

     

     

    1. Brachiocephalic

    2. Innominate

    3. Subclavian

    4. Axillary

    5. Circumflex scapular

     

    Corrent answer: 4

     

    The axillary artery commences at the first rib as a direct continuation of the subclavian artery and becomes the brachial artery at the lower border of the teres major. The arteriogram reveals a nonfilling defect in the third portion of the artery just distal to the subscapular artery. The complex arterial collateral circulation in this region often permits distal perfusion of the extremity despite injury.

     

     

     

    OrthoCash 2020

     

  132. Which of the following structures may help maintain radial length after a radial head fracture?

    1. Triangular fibrocartilage complex

    2. Medial ulnar collateral ligament

    3. Lateral ulnar collateral ligament

    4. Annular ligament

    5. Coronoid

     

    Corrent answer: 1

    Essex-Lopresti injuries affect axial stability of the forearm. Injury to the interosseous membrane or the triangular fibrocartilage complex can result in proximal migration of the radius.

     

     

     

    OrthoCash 2020

     

  133. An adult patient has a closed humeral fracture that was treated nonsurgically and a concomitant radial nerve injury. Six weeks after injury, electromyography shows no evidence of recovery. Management should now consist of

    1. exploration and neurolysis/repair.

    2. MRI of the arm.

    3. functional electrical stimulation.

    4. radial nerve tendon transfers.

    5. observation.

     

    Corrent answer: 5

     

    In patients with radial nerve injuries with closed humeral fractures, it has been reported that 85% to 95% spontaneously recover. Based on this premise, most surgeons favor expectant management of these injuries. Even if there is no evidence of recovery at 6 weeks, repeat electromyography at 12 weeks is advocated. If there is no clinical or electromyographic signs of recovery at 6 months, exploration is recommended. If the nerve is in continuity at the time of exploration, nerve action potentials are useful in helping determine the need for neurolysis, excision, and grafting, or if excision and repair is the best option.

     

     

     

    OrthoCash 2020

     

  134. Figure 37 shows a coronal T2-weighted MRI scan. What is the name of the labeled torn structure?

     

     

     

    1. Brachialis tendon

    2. Biceps tendon

    3. Flexor/pronator origin

    4. Medial collateral ligament (MCL)

    5. Lateral collateral ligament (LCL) Corrent answer: 5

    The labeled structure is the LCL, and it is avulsed from the lateral humeral epicondyle. This is the most common site of injury for the LCL. The biceps and brachialis tendon insertions are not well visualized in this section. The MCL and flexor/pronator origin are intact.

     

     

     

    OrthoCash 2020

     

  135. Which of the following findings is a contraindication to isolated percutaneous pinning of a distal radius fracture?

    1. Dorsal comminution

    2. Volar comminution

    3. Radial comminution

    4. Intra-articular fracture

    5. Physeal fracture

     

    Corrent answer: 2

     

    Intrafocal pinning allows the Kirschner wires to be placed through a site of comminution and then drilled through intact cortex. Generally Kapandji intrafocal pinning is done for dorsal comminuted extra-articular dorsal bending

    fractures, but it also may be used to elevate and buttress radial comminution. Simple intra-articular fractures can also be treated with pinning alone.

    Intrafocal pinning works best as a dorsal or radial buttress to prevent shortening. When there is volar comminution, the fracture is prone to shortening and supplemental external fixation or plating is recommended.

     

     

     

    OrthoCash 2020

     

  136. A 55-year-old man sustained an elbow dislocation in a fall. Postreduction radiographs are shown in Figures 40a and 40b. What is the best course of management?

     

     

     

     

     

     

    1. Closed reduction and casting for 4 weeks

    2. Closed reduction and bracing with immediate range of motion

    3. Open reduction, lateral collateral ligament repair, and open reduction and internal fixation or metallic replacement of the radial head

    4. Open reduction, radial head silastic arthroplasty, and lateral collateral ligament repair

    5. Open reduction, lateral collateral ligament repair, and radial head excision Corrent answer: 3

    The radiographs show an elbow dislocation associated with a comminuted radial head fracture. In the setting of comminution and instability, factures of the radial head are best managed with an arthroplasty rather than open reduction and internal fixation. Resection of the radial head will worsen the instability and is not recommended. Silastic radial head replacements are contraindicated.

     

     

     

    OrthoCash 2020

     

  137. In Figure A, the two red arrows point to which of the following two arteries?

     

     

     

     

    1. Superior gluteal and pudendal

    2. Internal iliac and medial circumflex

    3. External iliac and deep femoral

    4. Obturator and external iliac

    5. Medial circumflex and inferior gluteal Corrent answer: 4

    The external iliac and obturator artery anastomose to form the corona mortis. During the Stoppa or ilioinguinal approach to the pelvis, you need to be careful of the corona mortis because the vessels can cause significant bleeding especially if they retract into the pelvis.

    In the Tornetta et al article, fifty cadaver halves were dissected to determine the occurrence and location of the corona mortis. Anastomoses between the obturator and external iliac systems occurred in 84% of the specimens. Thirty-four percent had an arterial connection, 70% had a venous connection, and 20% had both. The distance from the symphysis laterally to the anastomotic vessels averaged 6.2 cm.

     

    The Okcu et al article showed similar results in 150 cadavers: they found vascular anastomoses between the obturator and external iliac systems in 61% of the sides, and anastomotic veins in 52% of the exposures. The mean distance between the anastomotic arteries and the symphysis pubis was 6.4 cm, and 5.6 cm for the communicating veins. There seemed to be no significant difference between genders in the incidence of corona mortis and the distance between communicating vessels and the symphysis pubis.

     

    Illustration A is a diagram of the involved vasculature.

     

     

     

     

     

     

    OrthoCash 2020

     

  138. A 38-year-old woman with diabetes mellitus reports a 6-week history of fever and pain localized to the right sternoclavicular joint. Local signs on examination include swelling about the joint, erythema, and increased warmth. Initial aspiration of the joint reveals Staphylococcus aureus. Radiographs reveal medial clavicular osteolysis. What is the most effective treatment at this time?

    1. Broad-spectrum parenteral antibiotics

    2. Repeat aspirations

    3. Irrigation and debridement

    4. Hyperbaric oxygen

    5. Resection of the sternoclavicular joint Corrent answer: 5

    Based on the findings, the treatment of choice is resection of the sternoclavicular joint. Antibiotic therapy, repeat aspirations, hyperbaric oxygen, and simple irrigation and debridement are generally ineffective and associated with a high rate of recurrence.

     

     

     

    OrthoCash 2020

     

  139. A patient has a humeral shaft fracture and is scheduled to undergo open reduction and internal fixation with a plate. What surgical approach will provide the greatest amount of exposure?

    1. Modified posterior approach with elevation of the medial and lateral heads of the triceps

    2. Posterior triceps-splitting approach

    3. Posterior triceps-splitting approach with radial nerve mobilization

    4. Posteromedial approach

    5. Lateral approach with radial nerve mobilization Corrent answer: 1

    The modified posterior approach with elevation of the medial and lateral heads of the triceps can provide exposure of 94% of the humeral shaft. The traditional posterior triceps-splitting approach exposes 55% of the humeral shaft.

     

     

     

    OrthoCash 2020

     

  140. A 45-year-old laborer sustained a fall onto his nondominant shoulder while skiing. His sensation is intact throughout the extremity but he is unable to flex the arm above 90 degrees. A radiograph of his shoulder obtained the next day in the emergency room is shown in Figure A. What is the best treatment option?

     

     

     

    1. Sling and swathe for 6 weeks then physical therapy

    2. Reverse total shoulder arthroplasty

    3. ORIF of proximal humerus

    4. Closed reduction and percutaneous pinning of the greater tuberosity

    5. Hemiarthroplasty

     

    Corrent answer: 3

     

    The radiograph shows a valgus impacted 4-part proximal humerus fracture. Due to the patient's young age, active lifestyle, and the displacement of the greater tuberosity operative management is necessary. Open reduction internal fixation has been shown in multiple studies to provide the best long term outcome in this patient population. ORIF allows for restoration of anatomical alignment, reducing the possibility of impingement and weakness from the greater tuberostiy malunion.

     

    The Robinson article reviewed series of 25 severely impacted proximal humerus fractures that all underwent ORIF and went on to union.

     

    Wijgman et al reviewed the results of 60 patients with 3 or 4 part fractures who underwent ORIF with a T-plate or cerclage wires and found 87% had good/excellent results based on the Constant score. Interestingly 22 pts (37%) had AVN, yet 17 of these 22 (77%) patients had excellent/good Constant scores.

     

     

     

     

     

    OrthoCash 2020

     

  141. A 35-year-old man sustained the closed injury shown in Figure 52 in his dominant extremity. Neurologic function is normal. Treatment should consist of

     

     

     

     

    1. functional bracing.

    2. a sling and swathe.

    3. intramedullary nail fixation.

    4. open reduction and internal fixation.

    5. iliac crest bone graft.

     

    Corrent answer: 1

     

    Functional bracing has been demonstrated to have a very high rate of healing without any functional limitations in a large series of patients. Surgery is reserved for “floating elbows,” open injuries, neurovascular injuries, and those fractures that go on to nonunion.

     

     

     

    OrthoCash 2020

     

  142. Figure 55 shows the radiograph of a 30-year-old man who sustained a closed comminuted fracture of the right clavicle.

    Examination reveals decreased sensation in the radial nerve distribution. Weakness is noted with shoulder abduction, internal rotation, and wrist extension. A displaced bone fragment is most likely pressing on what portion of the brachial plexus?

     

     

     

     

    1. C5 and C6 spinal roots

    2. Superior trunk

    3. Anterior division of the inferior trunk

    4. Posterior cord

    5. Lateral and posterior cords Corrent answer: 4

    Clavicular fractures are occasionally complicated by injury to the brachial plexus. A displaced bone fragment pressing on the posterior cord proximal to the upper subscapularis nerve would account for these findings.

     

     

    OrthoCash 2020

     

  143. A 7-year-old male is struck by a motor vehicle while crossing the street and suffers an open tibia fracture with a crush injury of the ipsilateral foot. After multiple attempts at limb salvage, the family and treating surgeon elect to proceed with a knee disarticulation. What complication of pediatric amputations is avoided with a knee disarticulation as opposed to a transtibial amputation?

    1. Neurogenic pain

    2. Bone overgrowth

    3. Hip flexion contracture

    4. Hip adduction contracture

    5. Leg length inequality Corrent answer: 2

    Bone overgrowth is a poorly understood phenomenon in which the bone end undergoes disorganized appositional growth following amputation in a skeletally immature patient. Overgrowth is the most common complication following transosseous amputation in pediatric patients.

     

    Krajbich reviews the management of pediatric patients with lower-limb deficiences and amputations. He advocates disarticulation as opposed to transosseous amputation when possible as bone overgrowth has not been observed in bone ends covered by articular cartilage.

     

    O'neal et al retrospectively reviewed their rates of surgical revision for bone overgrowth in pediatric and adolescent amputees. The highest rates of revision were seen with metaphyseal-level amputations (50%) and with traumatic amputations (43%).

     

    Benevenia et al reviewed their rates of overgrowth in skeletally immature transosseous amputees using an autogenous epiphyseal transplant from the amputated limb to cap the medullary canal. They found that only 1 of 10 patients undergoing amputation with this technique had complications due to bone overgrowth, compared with 6 of 7 patients undergoing traditional transosseous amputation.

     

    Illustration A is a clinical photo of bone overgrowth eroding through the soft tissue in a transhumeral amputee. Illustration B demonstrates the radiographic appearance of bone overgrowth in a transtibial amputation.

     

    Incorrect Answers:

    Answer 1. Neurogenic pain is a concern for adults, but rarely occurs in children.

    Answer 3. Flexion contracture is not a common complication following amputation in pediatric patients.

    Answer 4. Adduction contracture is not a common complication following amputation in this patient population.

    Answer 5. The proximal tibial physis, important for longitudinal growth of the limb, would be sacrificed with a knee disarticulation, meaning the residual limb will be shorter than if the patient had a transtibial amputation.

     

     

     

     

     

     

    OrthoCash 2020

     

  144. A 52-year-old woman reports mild pain localized to the left sternoclavicular joint. History is notable for chronic renal failure requiring dialysis for the last 5 years. A clinical photograph, chest radiograph, and bone scan are shown in Figures 58a through 58c. What is the most likely diagnosis?

     

     

     

     

     

     

     

     

     

    1. Pseudogout

    2. Spontaneous subluxation

    3. Postmenopausal arthritis

    4. Chronic osteomyelitis

    5. Friedreich’s disease

     

    Corrent answer: 4

    Spontaneous swelling with the appearance of joint subluxation may be associated with an acute, subacute, or chronic bacterial infection of the sternoclavicular joint. Common causes of infection include bacteremia, rheumatoid arthritis, alcoholism, intravenous drug use, and chronic debilitating diseases. Subclavian vein catheterization and renal dialysis can predispose patients to sepsis and osteomyelitis of the sternoclavicular joint.

     

     

     

    OrthoCash 2020

     

  145. A 19-year-old man was struck by a car and is seen in the emergency department with a grade IIIC open distal tibia and fibula fracture. Examination reveals that the toes are cool and dusky with a sluggish capillary refill. Angiography reveals a lesion in the posterior tibial artery amenable to repair. There is no sensation on the plantar aspect of the foot, and he is unable to flex his toes. A clinical photograph and radiograph are shown in Figures 2a and 2b. What is the next most appropriate step in management?

     

     

     

     

     

     

    1. Irrigation and debridement with immediate intramedullary fixation, vascular repair, and primary closure

    2. Irrigation and debridement with external fixation, vascular repair, and delayed closure

    3. Irrigation and debridement with external fixation, vascular repair, exploration of the tibial nerve, and delayed closure

    4. Guillotine amputation at the fracture site with delayed closure

    5. Immediate below-knee amputation Corrent answer: 2

    In the past, loss of plantar sensation in this grade IIIC tibial fracture would have been an indication for below-knee amputation regardless of the potential for vascular repair. However the 2002 LEAP study divided 55 patients with loss of plantar sensation into two groups, the insensate amputation group and the insensate limb salvage group, with 55% of patients in the insensate salvage group regaining normal sensation 2 years after injury. Furthermore, those in

    the salvage group who remained insensate after 2 years had equivalent outcomes to those in the amputation group. Because of these findings, limb salvage with vascular repair and external stabilization with delayed closure is deemed appropriate treatment. Immediate intramedullary fixation is not indicated. Because ischemia, contusion, and stretch can adversely affect the tibial nerve, the additional insult of exploration of the nerve is also not advisable given the soft-tissue compromise.

     

     

     

    OrthoCash 2020

     

  146. A 27-year-old man now reports dorsiflexion and inversion weakness after an automobile collision 6 months ago in which compartment syndrome developed isolated to the anterior and deep posterior compartments. Examination reveals the development of a progressive cavovarus deformity, but the ankle and hindfoot remain flexible. In addition to Achilles tendon lengthening, which of the following procedures is most likely to improve the motor balance of his foot and ankle?

    1. Anterior tibialis tendon transfer to the dorsolateral midfoot

    2. Posterior tibialis tendon transfer to the dorsolateral midfoot

    3. Peroneus longus tendon transfer to the dorsolateral midfoot

    4. Peroneus brevis tendon transfer to the dorsolateral midfoot

    5. Flexor hallucis longus tendon transfer to the peroneus brevis Corrent answer: 3

    Compartment syndrome of the anterior and deep posterior compartments results in anterior tibialis and posterior tibialis tendon weakness, respectively. Furthermore, the long flexors to the hallux and lesser toes will be weak as well. The intact peroneus longus overpowers the weak anterior tibialis tendon, resulting in plantar flexion of the first metatarsal, cavus, and hindfoot varus.

    Therefore, transferring the peroneus longus to the dorsolateral midfoot reduces the first metatarsal plantar flexion torque, and possibly augments ankle dorsiflexion torque.

     

     

     

    OrthoCash 2020

     

  147. Figures 4a through 4c show the radiographs of a 43-year-old woman who sustained a twisting injury to her right ankle. She has ankle pain and tenderness medially and laterally. To help determine the optimal treatment, an external rotation stress radiograph of the

    ankle is obtained. This test is designed to evaluate the integrity of what structure?

     

     

     

     

     

     

     

    1. Posterior talofibular ligament

    2. Distal tibiofibular syndesmosis

    3. Anterior talofibular ligament

    4. Deltoid ligament

    5. Calcaneofibular ligament

    Corrent answer: 4

     

    In the presence of a supination external rotation-type fracture of the distal fibula (Weber type B), stability of the ankle is best assessed by performing an external rotation stress AP view of the ankle. This test is used to assess the integrity of the deltoid ligament. The presence of a deltoid ligament rupture results in instability and generally is best managed surgically. The gravity stress test can also be used.

     

     

     

    OrthoCash 2020

     

  148. A 29-year-old patient sustains a closed, displaced joint depression intra-articular calcaneus fracture. In discussing potential complications of surgical intervention through an extensile lateral approach, which of the following is considered the most common complication following surgery?

    1. Nonunion

    2. Deep infection

    3. Delayed wound healing

    4. Peroneal tendinitis

    5. Posttraumatic arthritis

     

    Corrent answer: 3

     

    Delayed wound healing and wound dehiscence is the most common complication of surgical management of calcaneal fractures through an extensile lateral approach, occurring in up to 25% of patients. Most wounds ultimately heal with local treatment; the deep infection rate is approximately 1% to 4% in closed fractures. Posttraumatic arthritis may develop despite open reduction and internal fixation, but the percentages remain low. Peroneal tendinitis may occur from adhesions within the tendon sheath or from prominent hardware but is relatively uncommon. Nonunion of a calcaneal fracture is rare.

     

     

     

    OrthoCash 2020

     

  149. Twelve months after open reduction and internal fixation of a comminuted distal radius fracture as seen in Figure A and B, which of the following tendons is at greatest risk of rupture?

     

     

     

    1. Abductor Pollicis Longus

    2. Extensor Pollicis Brevis

    3. Extensor Indicis Proprius

    4. Flexor Pollicis Brevis

    5. Flexor Pollicis Longus Corrent answer: 5

    Figure A shows a distal radius fracture treated with a volar locked plate. Historically, distal radius fractures treated with dorsal plates were notorious for extensor tendon irritation with some cited rates as high as 50%.

     

    In the reference by Drobetz et al, they reviewed 50 fractures treated with a volar locked plate. They noted that in 6 (12%) of cases that there was rupture of the flexor pollicis longus (FPL) at a mean of 10 months post operatively.

     

    The second reference by Douthit is a retrospective review of 46 fractures treated with a volar locked plate. Excellent initial and maintenance of reduction was noted in 85% of cases, but FPL rupture occurred in 2 patients. Prominent plates and sharp screws were cited as reasons for tendon rupture.

     

     

     

    OrthoCash 2020

  150. Figures 5a and 5b show the radiographs of a 56-year-old man who was seen in the emergency department following a twisting injury to his left ankle. Examination in your office 3 days later reveals marked swelling and diffuse tenderness to palpation about the ankle and leg. What is the next most appropriate step in management?

     

     

     

     

    1. MRI of the ankle

    2. CT of the ankle

    3. Technetium bone scan

    4. Radiographs of the tibia and fibula

    5. Repeat radiographs of the ankle in 5 to 7 days Corrent answer: 4

    The radiographs show an isolated posterior malleolus fracture which, given the injury mechanism, is highly suspicious for a Maisonneuve injury. As with any suspected extremity injury, radiographs including the joints above and below the level of injury are acutely indicated. Although MRI may reveal a ligamentous injury to the ankle and CT may show asymmetry of the ankle mortise or syndesmosis, both studies are considerably more costly and are not indicated in the absence of a complete radiographic work-up. Technetium bone scan is nonspecific and would be of limited value in this instance, as would repeat radiographs of the ankle.

     

     

     

    OrthoCash 2020

  151. A 25-year-old woman with a healed proximal tibiofibular fracture treated with an intramedullary nail 2 years ago is currently wearing an ankle-foot orthosis (AFO) and reports a persistent foot drop. She is unhappy with the AFO and has not seen any functional improvement despite months of physical therapy. Serial electromyograms (EMG) show no recent change over the past year. Examination and EMG findings are consistent with a tibialis anterior 1/5, extensor hallucis longus 2/5, extensor digitorum longus 2/5, posterior tibial tendon (PTT) 5/5, peroneals 3/5, flexor hallucis longus 5/5, and gastrocsoleus 5/5. No discrete nerve lesion was identified. The patient has a flexible equinovarus contracture. What is the most appropriate management?

    1. Continued AFO bracing and therapy

    2. Ankle fusion

    3. Exploration and release of the common peroneal nerve

    4. Transfer of the PTT through the interosseous membrane with attachment to the tibialis anterior and peroneus tertius above the level of the ankle, debridement of the anterior compartment, and Achilles tendon lengthening

    5. Transfer of the peroneus longus to the dorsum of the foot and Achilles tendon lengthening

    Corrent answer: 4

     

    This pattern of injury is consistent with an unrecognized compartment syndrome of the anterior and lateral compartments. Transfer of the PTT through a long incision in the interosseous membrane corrects the foot drop deformity, and allows adequate dorsiflexion provided that the tendon to be transferred has a strength of 5/5. Muscles/tendons typically lose one grade of strength after transfer. Transfer into the tendons at the level of the ankle prevents overtensioning or pullout of a PTT tendon that is not long enough.

    Debridement of the scarred muscle in the anterior compartment decreases the risk of scarring down to the tendon transfer. Transfer of the peroneus longus is not preferred given its relative lack of strength and line of pull. Continued therapy and bracing are unlikely to lead to further improvement at 2 years after injury. An ankle fusion would correct the foot drop but would not address the tendon imbalances between the tibialis anterior and the peroneus longus, and the PTT and the peroneus brevis.

     

     

     

    OrthoCash 2020

     

  152. When using a two-incision approach for open reduction and internal fixation of a Hawkins III talar fracture-dislocation involving

    the talar neck and body, what anatomic structure must be preserved to optimize outcome?

    1. Deltoid branch of the artery of the tarsal canal

    2. Dorsalis pedis artery

    3. Tarsal sinus artery

    4. Perforating peroneal artery

    5. Navicular artery

     

    Corrent answer: 1

     

    A Hawkins III fracture-dislocation generally presents with posteromedial displacement with the deltoid ligament intact. Therefore, the only remaining blood supply is the deltoid branch of the artery of the tarsal canal originating from the posterior tibial artery. Often, the medial malleolus is fractured, assisting in reduction and visualization of fracture reduction. If the medial malleolus is intact, a medial malleolus osteotomy allows visualization of the reduction without compromising the last remaining blood supply to the talus.

     

     

     

    OrthoCash 2020

     

  153. A 68-year-old man fell off a 20-foot mountain cliff and was seen in the emergency department the following morning. A radiograph is shown in Figure 12. He is a nonsmoker with medical comorbidities of hypertension and hypercholesterolemia that is well controlled with medicine and diet. Capillary refill and sensation are intact distally and the patient is able to move his toes with mild discomfort.

    Serosanguinous fracture blisters are present laterally, and the foot is swollen and red. What is the most appropriate management?

     

     

     

    1. Short leg cast for 6 weeks

    2. Splinting with early range of motion at 3 weeks

    3. Immediate open reduction and internal fixation through a medial approach

    4. Delayed open reduction and internal fixation

    5. Fusion

     

    Corrent answer: 4

     

    Whereas a patient age of older than 50 years used to be a contraindication for open reduction and internal fixation of displaced intra-articular calcaneal fractures, new data suggest that the presence of associated medical comorbidities that affect wound healing such as smoking, diabetes mellitus, and peripheral vascular disease are more relevant to postoperative functional outcome. Surgical treatment of Sanders II and III displaced intra-articular calcaneal fractures with initial Bohler angles of > 15 degrees results in better outcomes as compared to nonsurgical management. Indications for primary fusion might include Sanders IV fractures in which articular congruity or Bohler angles cannot be restored. Given the condition of the soft tissues at presentation, delayed fixation is recommended.

     

     

     

    OrthoCash 2020

     

  154. A 45-year-old man is seen in the emergency department after returning from a 2-hour airplane flight. He is reporting severe pain in his right leg but has no trouble moving his ankle, leg, or knee. Venous doppler testing reveals no evidence of deep venous thrombosis. He is placed on IV cephazolin but continues to worsen. On the third day in the hospital he has increased pain, some respiratory distress, and trouble maintaining his blood pressure. His leg takes on the appearance seen in Figure 15. An urgent MRI scan shows thickening of the subcutaneous tissues and superficial swelling in the leg but no evidence of an abscess. What is the next most appropriate step in management?

     

     

     

    1. Triple antibiotic coverage

    2. Transfer to the ICU and a consult with infectious disease

    3. Urgent irrigation and debridement with gentle skin closure

    4. Urgent hyperbaric oxygen treatments and immunoglobulin

    5. Urgent aggressive debridement of skin, subcutaneous fat, and fascia Corrent answer: 5

    The patient has necrotizing fasciitis, a rare and sometimes fatal disease that has many different etiologies. Signs that this is not a normal infection are the worsening clinical symptoms despite IV antibiotics and the systemic symptoms. He needs urgent surgical care before he becomes completely septic and unstable. He needs very aggressive debridement of his tissues. Hyperbaric oxygen and immunoglobulins are only anecdotally helpful, and would only be used after surgery.

     

     

     

    OrthoCash 2020

     

  155. Figures 17a through 17c show the radiographs of a 38-year-old man following a motorcycle accident. The posterior portion of the talus extruded through a posterolateral wound. The extruded talar body is visible in the wound along with some road debris. Management should now consist of surgical irrigation, debridement, and

     

     

     

     

     

     

    1. removal of the extruded talus and placement of an external fixator.

    2. immediate tibiocalcaneal fusion.

    3. reimplantation of the talus, external fixation, and/or open reduction and internal fixation of the talar neck fracture.

    4. reimplantation followed by primary tibiotalar arthrodesis.

    5. Syme amputation.

     

    Corrent answer: 3

     

    The extruded talus should be placed in sterile bacitracin solution, irrigated thoroughly, gently debrided, and immediately replanted in the OR. Open reduction and internal fixation of the talar fracture may be attempted immediately depending on the soft-tissue envelope, or delayed after soft-tissue

    stabilization with an external fixator. A retrospective study of 19 patients with an extruded talus reported that 12 patients had no subsequent surgery after definitive fixation, 7 had subsequent procedures, and 2 patients developed infections that were treated successfully at an average of 42-month follow-up. Successful outcome in this series was attributed to multiple debridements, soft-tissue stabilization, and primary wound closure.

     

     

     

    OrthoCash 2020

     

  156. A 52-year-old woman slipped on ice in her driveway. Radiographs are shown in Figures 19a and 19b. The patient was treated in a short leg cast with weight bearing as tolerated for 6 weeks. Due to persistent tenderness at the fracture site, a CAM walker was used for an additional 8 weeks. Nine months after the injury, the patient still walks with a limp and reports pain with deep palpation at the fracture site. What is the next most appropriate step in management?

     

     

     

     

     

     

    1. CT scan

    2. Repeat period of immobilization

    3. Referral to pain management for sympathetic blocks

    4. Continued observation and physical therapy

    5. Acupuncture

     

    Corrent answer: 1

    Persistent pain at the fracture site in the absence of infection is most likely due to a nonunion, best detected by CT. Walsh and DiGiovanni reported on a series of closed rotational fibular fractures in which nonunions were detected by CT in the absence of standard ankle radiographic findings. Repeat immobilization would not be appropriate at this late date. Pain management/sympathetic blocks would be considered if the patient displayed pain with light touch and disproportionate pain consistent with a complex mediated pain syndrome.

    Acupuncture would be expected to be of limited benefit.

     

     

     

    OrthoCash 2020

     

  157. A 23-year-old woman has had a 14-month history of ankle pain after surgical treatment of multiple injuries resulting from a motor vehicle accident. Weight bearing began 4 months after surgery. The pain occurs with weight bearing and motion, but there is very little pain at rest. She has no pertinent medical history and does not smoke. Figures 23a and 23b show current radiographs. What is the most appropriate surgical option?

     

     

     

     

    1. Talectomy

    2. Revision open reduction and internal fixation (ORIF) with bone grafting

    3. Ankle arthrodesis

    4. Tibiotalocalcaneal arthrodesis

    5. Triple arthrodesis

     

    Corrent answer: 2

    The radiographs reveal nonunion of a talar neck fracture. There is no radiographic evidence of osteonecrosis or significant degenerative arthritis. The results of talectomy are suboptimal. Arthrodesis would be indicated for degenerative arthritis. Revision ORIF is feasible and preserves motion. A vascularized graft should be considered whenever osteonecrosis is present, but the talar body appears viable in this case.

     

     

     

    OrthoCash 2020

     

  158. A 27-year-old man was struck by a taxi cab and sustained comminuted right distal third tibia and fibula fractures; treatment consisted of placement of an intramedullary nail in the tibia the following morning. At his 6-month follow-up, he has clawing of all five toes. Examination reveals flexion deformities of the distal and proximal interphalangeal joints that are flexible with plantar flexion and rigid with dorsiflexion. Calluses are present on the dorsum and tip of the toes. Single heel rise is normal. He has a mild equinus contracture (relative to the left leg) that is not relieved with knee flexion. What is the most appropriate treatment option?

    1. Physical therapy and bracing

    2. Reassurance that the deformity will resolve with time

    3. Achilles tendon lengthening, and release or retromalleolar lengthening of the flexor digitorum longus (FDL) and flexor hallucis longus (FHL)

    4. FDL and FHL tenotomies at the individual digits with transfer of the posterior tibial tendon to the dorsum of the foot

    5. FDL and FHL tenotomies at the individual digits with midfoot capsular release and hallux interphalangeal fusion

    Corrent answer: 3

     

    This is an example of tethering of the flexor hallucis longus/flexor digitorum longus (FHL/FDL) to the fracture site. Additional time and/or physical therapy and bracing would not be expected to be of benefit. Release of the FHL and FDL from the fracture site or retromalleolar lengthening will address the posttraumatic claw toe deformity and Achilles tendon lengthening will address the mild equinus. Posterior tibial tendon transfer is not appropriate as the patient demonstrates a normal heel rise. Midfoot releases and hallux fusion are also not indicated.

     

     

     

    OrthoCash 2020

  159. A 24-year-old man reports the development of a foot drop following a knee dislocation 1 year ago. The common peroneal nerve was found to be in continuity at the time of surgical reconstruction of the posterolateral corner of the knee joint. He would like to eliminate the need for an ankle-foot orthosis. What is the best option to achieve elimination of the orthosis?

    1. Repeat neurolysis of the common peroneal nerve at the knee level

    2. Repeat neurolysis of the common peroneal nerve with cable grafting

    3. Extensor hallucis longus transfer to the distal first metatarsal

    4. Anterior transfer of the tibialis posterior tendon through the interosseous membrane

    5. Ankle fusion

     

    Corrent answer: 4

     

    The ankle dorsiflexor muscles have been denervated for too long a period to expect reinnervation to be successful. Even if the extensor hallucis longus tendon was functional, it is unlikely to have sufficient strength to achieve dynamic ankle dorsiflexion. The tibialis posterior tendon transfer has been shown to predictably achieve these goals in a high percentage of patients.

    Successful ankle fusion is likely to fail with time due to the development of forefoot equinus.

     

     

     

    OrthoCash 2020

     

  160. A 21-year-old male construction worker fell from a roof and sustained an injury to his left foot. Radiographs and CT scans are shown in Figures 24a through 24e. Compared to nonsurgical management, surgical treatment offers which of the following advantages?

     

     

     

     

     

     

     

     

     

     

     

    1. Quicker return to activities

    2. Quicker return to work

    3. Increased subtalar joint range of motion

    4. Decreased risk of nonunion

    5. Decreased risk of posttraumatic arthritis Corrent answer: 5

    The radiographs and CT scans show a displaced intra-articular calcaneal fracture, with loss of calcaneal height and length. Recent multicenter, randomized, prospective studies suggest that surgical treatment of displaced intra-articular calcaneal fractures is associated with an almost six-fold decrease

    in the risk of posttraumatic subtalar arthritis (necessitating subtalar arthrodesis) compared to nonsurgical treatment. Despite ongoing controversy, surgical treatment has not been shown to be advantageous with respect to activity, time to return to work, or subtalar joint range of motion. A nonunion of a calcaneal fracture is exceedingly rare regardless of the treatment method.

     

     

     

    OrthoCash 2020

     

  161. A 35-year-old man is seen in the emergency department with a bullet wound to the foot that occurred 2 hours ago. Examination reveals a 0.5-cm entrance wound on the dorsum of the foot and a 1.5-cm exit wound on the plantar aspect. Exploration of the plantar wound in the emergency department reveals bone and metal fragments.

    Radiographs reveal a comminuted, unstable fracture of the base of the first metatarsal and cuneiform. Management should consist of tetanus toxoid, and

    1. surface irrigation, sterile dressing, and a short leg cast.

    2. surface irrigation, sterile dressing, a short leg cast, and oral antibiotics.

    3. surface irrigation, sterile dressing, a short leg cast, and IV antibiotics.

    4. surgical debridement, a short leg cast, and IV antibiotics.

    5. surgical debridement, external or internal fixation, and IV antibiotics.

     

    Corrent answer: 5

     

    The patient sustained a type I unstable fracture that requires debridement of superficial fragments from the sole and surgical stabilization. Low-velocity wounds less than 8 hours old are considered type I open fractures. In contrast, gunshot wounds with associated fractures more than 8 hours old are considered type II open fractures using the Gustilo and Anderson classification. Gustilo type I stable fractures due to gunshot wounds and seen within 8 hours can be treated with tetanus toxoid (if no history of immunization or booster within 5 years), surface irrigation, and casting or a hard sole shoe. Antibiotics are not required unless gross contamination is present. However, if the extent of contamination is unclear, or if a joint is penetrated, then routine antibiotic prophylaxis is recommended. Indications for surgery include: articular involvement, unstable fractures, presentation 8 or more hours after injury, tendon involvement, and superficial fragments in the palm or sole. Type I unstable fractures may be stabilized with internal or external fixation. Type II unstable fractures should be treated with external fixation and repeat debridements until clean.

     

     

     

    OrthoCash 2020

  162. A 32-year-old laborer reports left ankle pain and deformity. History reveals that he sustained a left ankle fracture 2 years ago and was treated with closed reduction and casting. Radiographs are shown in Figures 25a through 25c. What is the most appropriate management?

     

     

     

     

     

     

     

     

     

    1. Bracing and physical therapy

    2. Intra-articular injection of steroids into the ankle joint, bracing, and physical therapy

    3. Intra-articular injection of hyaluronic acid product into the ankle joint, bracing, and physical therapy

    4. Ankle fusion

    5. Corrective osteotomy of the fibula and medial malleolus with reconstruction of the syndesmosis if unstable

    Corrent answer: 5

     

    Corrective osteotomy of fibular malunions, with appropriate lengthening, even in the presence of early arthritis, has been shown to decrease ankle pain and increase stability. Reduction and bone grafting of the medial malleolar nonunion is also needed. There is no evidence supporting the use of intraarticular steroids or hyaluronic acid in the ankle joint. Lateral talar displacement of even 1 mm has been reproducibly shown to decrease tibiotalar contact by 40% to 42%, causing a predisposition to arthritis.

     

     

     

    OrthoCash 2020

     

  163. Figures 29a and 29b show a clinical photograph and radiographs of a patient who sustained an open calcaneus fracture in a motor vehicle accident. The patient received immediate IV antibiotics and an emergent irrigation and debridement. The swelling has subsided by 3 weeks and the medial wound is clean. What do you tell the patient

    about the likelihood of infection if a formal open reduction and internal fixation via a lateral approach is performed?

     

     

     

     

     

     

     

    1. There is no significant difference between the infection rate for this fracture and a similar closed fracture.

    2. Due to the risk of infection, open reduction and internal fixation is not recommended for this fracture.

    3. The infection rate is three to five times more likely with this fracture.

    4. Due to the risk of infection from a lateral approach, treatment is confined to limited internal fixation or an external fixator.

    5. The patient will need to undergo 3 weeks of IV antibiotics at home.

     

    Corrent answer: 1

     

    Multiple authors have shown similar infection rates for grade 1 and 2 open medial fractures and closed fractures that have been treated with an extensile lateral approach and open reduction and internal fixation. Patients only need IV antibiotics for 2 to 3 days after surgery. Formal open reduction and internal fixation is not recommended for grade 3 medial wounds and most lateral wounds.

     

     

     

    OrthoCash 2020

  164. A 24-year-old woman was struck by a mini van in a parking lot and sustained a closed segmental tibia fracture that was treated with an intramedullary nail the following morning. Follow-up examinations reveal a slowly progressive clawing of all five toes, a progressive equinocavovarus contracture, and the patient is unable to perform a single heel rise on the affected limb. At 1 year after surgery, the patient now has a 10-degree equinus contracture that is not relieved with knee flexion. Treatment should now consist of

    1. physical therapy and bracing.

    2. reassurance that these problems will resolve with time.

    3. posterior capsule release, Achilles tendon lengthening, and excision of the scarred muscle and tendon in the leg and foot.

    4. Achilles tendon lengthening, and flexor digitorum longus and flexor hallucis longus tenotomies at the individual digits with transfer of the posterior tibial tendon to the dorsum of the foot.

    5. flexor digitorum longus and flexor hallucis longus tenotomies at the individual digits with midfoot capsular release and hallux interphalangeal fusion.

    Corrent answer: 3

     

    This is an example of a missed deep posterior compartment syndrome that typically presents 6 months after the injury with progressive clawing due to necrosis, scarring, and contracture of the posterior tibial tendon, flexor digitorum longus, and flexor hallucis longus. Treatment consists of debridement of necrotic muscle and scar tissue with corresponding tendon excision. After debridement and posterior capsule release, if the equinus is relieved with knee flexion, a gastrocnemius slide may be performed.

    Otherwise, the lengthening should be at the level of the Achilles tendon. Bracing will not address the claw toes.

     

     

     

    OrthoCash 2020

     

  165. Optimal management of the injury shown in Figure 31 should include which of the following?

     

     

     

    1. Cast immobilization in equinus

    2. Open reduction and internal fixation once the acute soft-tissue swelling has resolved

    3. Urgent reduction and fixation

    4. Arthroscopic-assisted percutaneous fixation

    5. Open reduction and internal fixation with primary subtalar arthrodesis Corrent answer: 3

    The radiograph shows a displaced calcaneal beak fracture, a tongue-type fracture variant. The fracture fragment typically includes the insertion point of the Achilles tendon, which places marked tension on the thin overlying soft-tissue envelope and can lead to full-thickness necrosis if not acutely addressed. Cast immobilization does not adequately address the increased soft-tissue tension, as the fragment will be difficult to control. Arthroscopic-assisted techniques or primary arthrodesis are not indicated because calcaneal beak fractures are typically extra-articular.

     

     

     

    OrthoCash 2020

     

  166. A 23-year-old man who was the restrained driver in a car involved in a high-speed motor vehicle accident sustained the closed injury shown in Figures 32a through 32c. Which of the following factors has the greatest impact on the risk of osteonecrosis?

     

     

     

     

     

     

    1. Surgical stabilization within 6 to 8 hours of injury

    2. Extent of initial fracture displacement

    3. Nicotine use

    4. Posterior-to-anterior screw fixation

    5. Anatomic fracture reduction Corrent answer: 2

    The incidence of osteonecrosis following displaced talar neck fractures is most related to the extent of initial fracture displacement. With increasing fracture displacement, the tenuous vascular supply to the talar body is more at risk for damage, thereby increasing the risk of osteonecrosis. Although displaced talar neck fractures have historically been considered a surgical emergency, recent studies have shown that the timing of surgical intervention bears no impact on the development of osteonecrosis. While nicotine use has an influence on fracture healing, it has never been shown to be a factor in osteonecrosis, nor has posterior-to-anterior screw fixation or the quality of fracture reduction.

     

     

     

    OrthoCash 2020

     

  167. A 32-year-old woman sustained a closed calcaneus fracture 2 years ago and was treated nonsurgically. She now reports a 6-month history of progressively worsening pain over the anterior ankle and lateral hindfoot. Climbing stairs and ascending slopes is particularly difficult for her. Bracing and intra-articular corticosteroid injections have not provided sufficient relief. Figure 36 shows a weight-bearing lateral radiograph. What is the most appropriate surgical option?

     

     

     

     

    1. Subtalar arthrodesis in situ with plantar flexion osteotomy of the talar neck

    2. Distraction subtalar arthrodesis with a corticocancellous bone block autograft

    3. Subtalar arthrodesis in situ

    4. Triple arthrodesis

    5. Subtalar arthrodesis in situ with anterior ankle exostectomy Corrent answer: 2

    Intra-articular fractures of the calcaneus often include depression of the posterior facet of the subtalar joint. This can lead to dorsiflexion of the talus because of diminished height posteriorly. In a weight-bearing position, the dorsal surface of the talar neck can impinge against the distal tibia, causing anterior ankle pain. In addition, posttraumatic arthritis of the subtalar joint typically occurs after a calcaneus fracture. The patient’s symptoms are consistent with both anterior ankle impingement and subtalar degenerative arthritis. The Bohler angle, approximately 15 degrees, confirms depression of the posterior facet. Distraction subtalar arthrodesis with a corticocancellous bone block autograft will improve talar declination, decrease anterior impingement, and address the subtalar degenerative arthritis simultaneously.

     

     

     

    OrthoCash 2020

     

  168. A 38-year-old man underwent a transtibial amputation for chronic posttraumatic foot and ankle pain and chronic calcaneal osteomyelitis. Postoperative radiographs are seen in Figures 41a and 41b. What is the proposed purpose of the surgical modification seen in the radiographs?

     

     

     

     

    1. Reduces shrinkage of the residual limb

    2. Creates a more stable platform for load transfer

    3. Reduces wound healing complications by avoiding the soft-tissue dissection necessary to transect the fibula at a level proximal to the tibia

    4. Connecting bone strut provides an attachment point for more effective

      myodesis

    5. Allows a more proximal resection level to decrease tension on the wound Corrent answer: 2

    The Ertl modification of a below-knee amputation has been proposed to create a more stable “platform” to aid in transferring the load of weight bearing between the residual limb and the prosthetic socket. It is felt that a stable platform allows total contact loading over an enlarged stable surface area.

    Early studies have suggested that this modification may enhance the patient’s perceived functional outcome.

     

     

     

    OrthoCash 2020

     

  169. A 51-year-old man sustained an open fracture of his tibia in Korea 42 years ago. An infection developed and it was resolved with surgical treatment. For the past 6 months, an ulcer with mild drainage has developed over the medial tibia. The ulcer is small and there is minimal erythema at the ulcer site. A radiograph and MRI scan are shown in Figures 43a and Figure 43b. Initial cultures show Staphylococcus aureus susceptible to the most appropriate antibiotics. Laboratory studies show an erythrocyte sedimentation rate of 70 mm/h. What is the most appropriate surgical treatment at this time?

     

     

     

     

    1. Irrigation and debridement of the cystic lesion and 6 weeks of IV antibiotics

    2. Curettage, debridement of nonviable bone, and placement of absorbable

      antibiotic beads, followed by a course of IV antibiotics from 1 to 4 weeks and a 6-week course of oral antibiotics

    3. Complete resection of the infected portion of bone, placement of an external fixator to stabilize the tibia, and 6 weeks of IV antibiotics

    4. Amputation

    5. Local debridement of bone and the overlying skin and soft tissues, 6 weeks of IV antibiotics, and free-flap wound coverage

    Corrent answer: 2

     

    The patient has chronic tibial osteomyelitis that is due to low virulent bacteria. The history and studies do not suggest the need for an amputation or a free-flap procedure. This is a localized tibial infection that is in a healed bone; there is no need to resect the entire area of the tibia bone around the infection. The most appropriate treatment is curettage, debridement of nonviable bone, and placement of absorbable antibiotic beads, followed by a course of IV antibiotics from 1 to 4 weeks and a 6-week course of oral antibiotics. Studies have shown that in cases of localized osteomyelitis that are of low virulence, as little as 1 week of IV antibiotics followed by 6 weeks of oral antibiotics is successful.

     

     

     

    OrthoCash 2020

     

  170. A 59-year-old woman underwent open reduction and internal fixation (ORIF) of her ankle 6 months ago, with subsequent hardware removal 3 months later. She now reports persistent, diffuse ankle pain, swelling, and limited range of motion. Figure 48 shows an oblique radiograph of the ankle. What is the next most appropriate step in management?

     

     

     

    1. Physical therapy

    2. Hardware removal

    3. Repeat placement of a syndesmotic screw

    4. Deltoid ligament reconstruction

    5. Revision ORIF with exploration of the syndesmosis and medial ankle Corrent answer: 5

    The radiographs demonstrate persistent widening of the medial clear space with an ossicle. This represents soft-tissue interposition-scar tissue, the deltoid ligament, or the posterior tibialis tendon. Physical therapy will not improve the symptomatic malalignment. Hardware removal would be indicated for pain localized to the lateral fibula. Repeat syndesmotic screw fixation alone will not reduce the malalignment. Deltoid ligament repair may be necessary but will need to be combined with debridement of the medial ankle and syndesmosis, as well as repeat placement of one or more syndesmotic screws to maintain the reduction.

     

     

     

    OrthoCash 2020

     

  171. A farmer is seen in the emergency department after falling out of a hay loft onto the barn floor below. He is unable to bear weight.

    Exploration of a 0.5 cm laceration over the anterior tibia reveals bone. Radiographs reveal oblique displaced midshaft tibial and fibular fractures. Based on these findings, what is the most appropriate antibiotic prophylaxis?

    1. Cephalosporin

    2. Cephalosporin and aminoglycoside

    3. Cephalosporin and penicillin

    4. Cephalosporin and vancomyacin

    5. Cephalosporin, aminoglycoside, and penicillin Corrent answer: 5

    A farm injury is automatically considered a grade III (Gustillo classification) injury regardless of size, energy, or additional soft-tissue injury due to the likelihood of substantial contamination. Antibiotic recommendations for grade III injuries include a first- or second-generation cephalosporin with an aminoglycoside or fluoroquinolone within 3 hours of injury, with penicillin added for farm injuries.

     

     

     

    OrthoCash 2020

     

  172. Which of the following have been found to affect the rate of perioperative infections or wound complication rates in foot and ankle surgery?

    1. Methotrexate

    2. Gold

    3. Hydroxychloroquine

    4. TNF-a inhibitors

    5. Smoking

     

    Corrent answer: 5

     

    Clinical studies have shown that smoking cessation for 4 weeks reduces the risk of infection to the level of nonsmokers. Adverse effects on wound healing caused by chemotherapy used to treat rheumatoid arthritis has not been borne out in the literature.

     

     

     

    OrthoCash 2020

     

  173. Your 25-year-old patient complains of anterior knee pain after retrograde femoral nailing for a diaphyseal fracture and asks you why you didn’t perform antegrade nailing as he has seen on the internet.

    You tell him that retrograde nailing is your preferred technique over antergrade nailing for diaphyseal femoral fractures because it has been shown to have which of the following?

    1. Increased rate of union

    2. Decreased rate of infection

    3. Shorter operative time

    4. Lower rates of hip pain

    5. Lower rates of knee pain Corrent answer: 4

    Patients with retrograde femoral nails commonly have knee pain, while antegrade nails commonly have hip pain, abductor weakness and heterotopic ossification of the abductors.

     

    Ostrum’s randomized prospective study of 100 patients with reamed femoral nails found 22% of antegrade nail patients had proximal hip pain, weak hip abductors or trendelenburg gait. No significant difference was found in set-up time, operative time, knee motion or pain, or infection rates.

     

    Ricci performed a retrospective study of 293 fractures and found that the antegrade femoral nail group had more hip pain (10% vs 4%) and the retrograde nail group had more knee pain (36% vs 9%). There was no difference in healing, malunion, non-union or other complications.

     

    Tornetta performed a randomized controlled comparison of 69 femur fractures and found more problems of length and rotation using a retrograde nailing.

    There was no difference in time to union, operating time, blood loss, complications, size of nail or reamer, or transfusion requirements.

     

     

     

    OrthoCash 2020

     

  174. Sacral fractures are most likely to be associated with neurologic deficits when they involve what portion of the sacrum?

    1. Zone 1 (the ala)

    2. Zone 2 (the foramina)

    3. Zone 3 (the central canal)

    4. Zones 1 and 2

    5. The sacral laminae Corrent answer: 3

    Denis divided the sacrum into three zones: zone 1 represents the lateral ala, zone 2 represents the foramina, and zone 3 represents the central canal. A fracture is classified according to its most medial extension. Those in zone 3

    are typically bursting-type fractures or fracture-dislocations and are most prone to neurologic sequelae.

     

     

     

    OrthoCash 2020

     

  175. A 31-year-old male sustains the injury shown in Figure A. As compared to treatment with a simple sling, what is the primary advantage of treatment with a figure-of-eight brace?

     

     

     

     

    1. Decreased sleep disturbance

    2. Decreased personal care and hygiene impairment

    3. Decreased rates of malunion

    4. Improved long-term clinical outcomes

    5. No advantage, equivalent result between a simple sling and figure-of-eight brace

    Corrent answer: 5

     

    Figure of eight braces have been shown to have no differences as compared to simple slings in regard to healing times, healing rates, and alignment at final follow-up.

     

    The referenced study by Andersen et al is a Level 1 randomized controlled study showing equivalent cosmetic and clinical outcomes with sling versus figure of eight bracing despite increased sleep disturbances and increased rate of personal care impairment in those treated with a figure of eight brace.

     

    The second referenced study by Nordqvist et al is a Level 4 case series designed to analyze the long-term outcome of mid-clavicle fractures in adults and to evaluate the clinical importance of displacement and fracture

    comminution. They found a 39/225 rate of moderate shoulder pain with figure of eight bracing. Overall they concluded that few patients with fractures of the mid-part of the clavicle require operative treatment.

     

     

     

    OrthoCash 2020

     

  176. A 26-year-old woman sustained a nondisplaced femoral neck fracture and treatment consisted of use of percutaneous cannulated screws. At her 3-month follow-up visit, she reports hip pain and is unable to ambulate. A radiograph is shown in Figure 1. What is the next most appropriate treatment?

     

     

     

     

    1. Bone grafting and revision open reduction and internal fixation

    2. Hemiarthroplasty

    3. Dynamic hip screw without angular correction

    4. Valgus intertrochanteric osteotomy

    5. Core decompression

     

    Corrent answer: 4

     

    Femoral neck fracture nonunion is a challenging problem for orthopaedic surgeons. Vertical fractures are more prone to nonunion due to shear stress rather than compressive forces across the fracture site. Several authors have suggested these fractures are more common in young adults due to injury type and bone composition. It is widely regarded that an effort should be made to salvage the femoral head if vascularity remains. The most common method to treat this complication is valgus intertrochanteric osteotomy of the femur. This functionally makes a vertical fracture more horizontal, converting shear into

    compressive forces. It also helps correct the varus position of the fracture nonunion.

     

     

     

    OrthoCash 2020

     

  177. Which of the following choices best describes the fracture pattern shown in Figures 2a through 2c?

     

     

     

     

     

     

     

    1. Anterior column

    2. Anterior wall

    3. Posterior column

    4. Both column

    5. Transverse

     

    Corrent answer: 3

     

    The fracture pattern shown in the radiographs is a fracture of the posterior column. The only line interrupted on the AP pelvis is the ilioischial line. The obturator oblique view shows that the iliopectineal line is intact as is the outline of the posterior wall. The iliac oblique view shows an interruption of the ilioischial line and an intact anterior wall. Therefore, this fracture is a fracture of the posterior column.

     

     

     

    OrthoCash 2020

     

  178. The correct starting point for an external fixation half pin placed into the anterior inferior iliac spine (AIIS) is labeled by what letter in Figure 3?

     

     

     

     

    1. A

    2. B

    3. C

    4. D

    5. E

     

    Corrent answer: 1

    Half pins placed in the AIIS are an alternative to pins placed in the iliac crest. A strong pillar of bone runs from the AIIS to the posterior iliac crest and less soft tissue is typically present in this area. The starting point is best seen on an obturator outlet view. The obturator outlet view is a combination of the pelvic outlet view and the obturator view of Judet and Letournel. The beam is rotated “over the top” of the patient since the iliac wing is externally rotated as well as cephalad to best visualize this column of bone running from the AIIS to the posterior iliac spine. This corridor of bone will appear as a teardrop. Once the correct view is obtained, the pin should be started at least 2 cm proximal to the hip joint to avoid placing a pin within the hip capsule. Blunt dissection and a guide sleeve should be used to prevent damage to the lateral femoral cutaneous nerve. An iliac oblique view is used after the pin has been partially inserted to make sure the pin is passing superior to the superior gluteal notch, and an obturator inlet view can be used at the completion of the procedure to make sure the pin is contained within the bone for its entire length.

     

     

     

    OrthoCash 2020

     

  179. Figures 4a and 4b show the radiographs of a 53-year-old woman who was injured in a fall. After initial closed reduction, what is the preferred treatment for this fracture?

     

     

     

     

     

     

    1. Open reduction and internal fixation of the radial head and immobilization

    2. Medial collateral ligament repair

    3. Radial head replacement, ulnar nerve transposition, and external fixation

    4. Coronoid repair, radial head replacement, and lateral ligamentous repair

    5. Nonsurgical management in a hinged elbow brace Corrent answer: 4

    This elbow fracture-dislocation involves a radial head fracture, coronoid fracture, and ulnohumeral dislocation (terrible triad). Several algorithms exist for treatment; surgical treatment is indicated. The treatment should address the radial head. Studies have shown replacement to be superior to repair in comminuted fractures. The coronoid may be addressed in unstable cases at the time of radial head excision and replacement. Lateral ligamentous repair is carried out during closure of the lateral elbow capsule. Medial ligamentous repair also may be undertaken but usually in concert with bony repair. Hinged

    external fixation remains an option when instability exists following bony and soft-tissue repair. Acute ulnar nerve transposition is rarely indicated.

     

     

     

    OrthoCash 2020

     

  180. A 29-year-old woman was injured in a high-speed motor vehicle accident 3 hours ago. Radiographs are shown in Figures 7a through 7e. Her right foot injury is open and contaminated. Her associated injuries include a closed head injury and a ruptured spleen requiring resection. She has had 6 units of packed red blood cells and the trauma surgeon has turned her care over to you. Her current base deficit is 10 and her urinary output has averaged 0.4 mL/kg for the last 2 hours. What is the best treatment at this time?

     

     

     

     

     

     

     

     

     

    1. Irrigation and debridement, external fixation of the ankle and foot, traction and pinning of the femur, open reduction and internal fixation of the forearm

    2. Irrigation and debridement, external fixation of the ankle, foot, and femur, splinting of the forearm

    3. Irrigation and debridement and open reduction and internal fixation of the ankle and foot, intramedullary nailing of the femur, open reduction and internal fixation of the forearm

    4. Irrigation and debridement and open reduction and internal fixation of the ankle and foot, intramedullary nailing of the femur, splinting of the forearm

    5. Irrigation and debridement, external fixation of the foot and ankle, intramedullary nailing of the femur, open reduction and internal fixation of the forearm

    Corrent answer: 2

     

    The patient appears to be a borderline or unstable surgical patient following her initial trauma and spleenectomy (high base excess and low urine output). She needs continued resuscitation and minimal additional blood loss. This is best accomplished with irrigation and debridement of the ankle, external fixation of the ankle, foot, and femur, and splinting of the forearm. A traction pin for the femoral fracture will not control bleeding as well as an external fixator. Intramedullary nailing of the femur and open reduction and internal fixation of the forearm would be appropriate in patients that are euvolemic and stable.

     

     

    OrthoCash 2020

     

  181. Which of the following is most commonly associated with an open clavicular fracture?

    1. Scapulothoracic dissociation

    2. Closed head injury

    3. Calcaneus fracture

    4. Pelvic ring injury

    5. Open tibial fracture Corrent answer: 2

    Open clavicular fractures are rare and result from high-energy trauma. In a series of 20 patients with open clavicular fractures, 13 (65%) sustained a closed head injury. Fifteen (75%) had associated pulmonary injuries and 35% had a cervical or thoracic spine fracture. Only one demonstrated scapulothoracic dissociation. Screening for pulmonary and closed head injuries should be considered in the setting of traumatic open clavicular fractures.

     

     

     

    OrthoCash 2020

     

  182. A 26-year-old man falls off a motorcycle and injures his left wrist. There are no open wounds and the neurovascular examination is normal. Radiographs are shown in Figures 10a and 10b. Definitive management should consist of

     

     

     

    1. closed reduction and casting.

    2. external fixation and percutaneous pinning of the distal radius.

    3. open reduction and internal fixation of the distal radius.

    4. open reduction and internal fixation of the distal radius and open repair of the ulnar styloid.

    5. nonbridging external fixation of the distal radius.

     

    Corrent answer: 3

     

    The patient has a high-energy injury with resultant comminution of the distal radius metaphysis. Cast immobilization is likely to lead to radial shortening and angulation due to the comminution. Similarly, while external fixation and pinning has been successful in the past, some loss of radial length and volar angulation is typically noted. Present plate fixation devices for the distal radius employing locking screw technology have a superior ability to resist radial shortening and dorsal angulation. Fixation of the ulnar styloid is warranted when there is distal radioulnar joint instability or significant displacement of the styloid. This is more likely to occur with a fracture at the base of the styloid. In this instance, the distal radioulnar joint does not appear to be disrupted.

     

     

     

    OrthoCash 2020

     

  183. Following fixation of a displaced intra-articular fracture of the distal humerus through a posterior approach, what is the expected outcome?

    1. Development of arthritic changes at 1 year

    2. Restoration of full elbow range of motion

    3. Loss of approximately 25% of elbow flexion strength

    4. Posterolateral rotatory instability

    5. Olecranon nonunion

     

    Corrent answer: 3

     

    Following repair of a displaced intra-articular distal humerus fracture, the ability to regain full elbow range of motion is rare. Recent reports of olecranon osteotomy have yielded healing rates of between 95% to 100%. According to McKee and associates, patients can be expected to have residual loss of elbow flexion strength of 25%.

     

     

     

    OrthoCash 2020

  184. A 28-year-old cowgirl was injured while herding cattle 1 week ago. A radiograph and CT scans are shown in Figures 13a through 13c. What is the most appropriate management for this injury?

     

     

     

     

     

     

     

     

     

    1. Nonsurgical management and gradual weight bearing as tolerated

    2. Nonsurgical management and restricted weight bearing

    3. Placement of a pelvic binder

    4. Open reduction and internal fixation of the symphysis

    5. Open reduction and internal fixation of the symphysis and iliosacral screws Corrent answer: 1

    The patient has an AP I pelvic ring disruption with minimal symphyseal widening. The best treatment is nonsurgical management and weight bearing as tolerated. This will help close the anterior pelvic ring during the healing process. Pelvic binders are excellent for acute treatment of widely displaced pelvic fractures but are not recommended for long-term use. Open reduction and internal fixation is not indicated for this injury and furthermore, the posterior ring is not injured.

     

     

     

    OrthoCash 2020

     

  185. As reflected by the SF-36 scores, patients with which of the following conditions demonstrate the most disability in physical function?

    1. AIDS

    2. Polytrauma

    3. Pelvic fracture

    4. Pilon fracture

    5. Acute myocardial infarction (AMI)

    Corrent answer: 4

     

    Pollak and associates found that the average SF-36 score for patients who sustained a pilon fracture was significantly lower than patients with diabetes mellitus, AIDS, hypertension, asthma, migraines, pelvic fracture, polytrauma, and AMI. Moreover, patients having undergone pilon fixation scored lower on all but three of the SF-36 scales (vitality, mental health, and emotional health).

     

     

     

    OrthoCash 2020

     

  186. A 25-year-old man is involved in a motor vehicle accident and brought to the emergency department at 4 am on Sunday morning. He has a closed distal third femoral shaft fracture. His leg is initially pulseless but after applying inline traction, a distal pulse can be palpated and the limb appears to be viable. The pulse in the injured limb “feels” different than the pulse in the uninjured limb. What is the next step in assessing the vascular status of this limb?

    1. Serial physical examinations

    2. Angiography

    3. Duplex ultrasound examination

    4. Ankle-brachial index (ABI)

    5. Measurement of compartment pressures Corrent answer: 4

    The patient initially has a distal third femoral fracture and a pulseless limb. The first step is to reduce the fracture and reassess the vascular status. Although the pulse returns, it feels different than the quality of the pulse in the contralateral uninjured extremity. There is a risk of a vascular injury with this fracture pattern due to tethering of the femoral vessels at the adductor hiatus; therefore, the vascular status needs further assessment since the pulses are not symmetrical. A physical examination is not very accurate in assessing whether a vascular injury is present; therefore, serial examinations are not appropriate. Angiography is very sensitive and specific but is time consuming and can cause complications secondary to the dye and the arterial puncture required to perform it. Duplex ultrasound is effective but is very operator-dependent and may not be available 24 hours a day. The ABI is easily performed and has been shown to be sensitive and specific. If the value is greater than 0.9, the negative predictive value is 99% and when the value is less than 0.9, it is 95% sensitive and 97% specific for a major arterial injury. It has been shown to be useful for blunt lower extremity injuries as well as knee dislocations.

     

     

    OrthoCash 2020

     

  187. What is the most appropriate treatment for a 50-year-old woman who sustains the injury shown in Figures 14a and 14b?

     

     

     

     

     

     

     

    1. Total elbow arthroplasty

    2. Functional hinge bracing

    3. Long arm casting

    4. Crossed Kirschner wires

    5. Dual column plates

    Corrent answer: 5

     

    This intra-articular distal humerus fracture with displacement at the joint surface is best treated with surgical fixation. The most biomechanically sound construct is two plates applied to either column 180 degrees from one another. Elbow arthroplasty is most appropriate for low demand elderly patients.

     

     

     

    OrthoCash 2020

     

  188. A 20-year-old man is brought to the emergency department after a high-speed motor vehicle accident. His initial blood pressure is 70/40 mm Hg. He is currently receiving intravenous fluids as well as blood. His Focused Assessment with Sonography for Trauma examination did not show any free fluid in his abdomen and his chest radiograph is unremarkable. An AP pelvis radiograph is shown in Figure 15. What is the next most appropriate step in the management of his pelvic injury?

     

     

     

     

    1. Inlet and outlet views of the pelvis to better delineate the injury

    2. Angiography

    3. Laparotomy

    4. Open reduction and internal fixation of the pelvis

    5. Placement of a pelvic binder around the patient Corrent answer: 5

    This hypotensive patient has an obvious open book injury of the pelvic ring on the AP pelvis radiograph and further radiographs are not needed prior to the

    initiation of treatment. Although angiography may be indicated if he does not respond to stabilization of his pelvis and fluid/blood administration, temporary stabilization of the pelvis with a sheet or binder should be performed first because it is simple, quick, and has been shown to be effective. This patient does not need a laparotomy at this point since the FAST examination did not show any free intra-abdominal fluid and his chest radiograph was unremarkable, leaving the most likely source of bleeding the pelvic fracture.

    Open reduction with internal fixation of a pelvic injury is not indicated in an acutely ill patient.

     

     

     

    OrthoCash 2020

     

  189. A 220-lb 20-year-old man was involved in a motor vehicle accident. His work-up reveals that he has multiple long bone fractures as well as a splenic injury that is currently being managed nonsurgically. His initial blood pressure in the trauma bay was 70/30 mm Hg. After receiving 4 liters of fluid and 3 units of packed red blood cells, his blood pressure is currently 110/70, his heart rate is 100, his urine output is 90 mL/h (normal 0.5 to 1 mL/kg/h), and his core temperature is 97.9 degrees F (36.5 degrees C). At this point, the patient’s resuscitation can be described as which of the following?

    1. Complete based on the normalization of his blood pressure, urine output, and heart rate

    2. Cannot be determined based on the data presented

    3. Incomplete based on his fluid requirements calculated using his initial blood pressure as a measure of blood volume loss

    4. Incomplete since he will need surgery on the long bone fractures and should be “tanked up” prior to losing blood in the operating room

    5. Incomplete based on his heart rate Corrent answer: 2

    Although the end points of resuscitation are still unclear, what is known is that normalization of the standard hemodynamic parameters (blood pressure, heart rate, and urine output) is not adequate. Up to 85% of patients with normal hemodynamic parameters can still have inadequate tissue oxygenation or uncompensated shock. The initial base deficit, lactate level, or gastric pHi can be used to stratify patients for resuscitation needs, risks of death, and multiple organ failure (level 1 evidence). The time it takes to normalize the base deficit, the lactate level, or gastric pHi, can predict survival (level 2 evidence). Patients who have been in uncompensated shock (abnormal vital signs) should have their resuscitation monitored using data other than vital signs.

     

     

    OrthoCash 2020

     

  190. A 30-year-old man who sustained a work-related injury 6 weeks ago reports persistent back and left-sided buttock pain that has been attributed to lumbar transverse process fractures. A pelvic radiograph and CT scans obtained 2 days ago are seen in Figures 17a through 17c. What is the best treatment for his injury?

     

     

     

     

     

     

     

     

     

    1. Continued nonsurgical management

    2. Posterior open reduction and internal fixation with tension band plating

    3. Posterior iliosacral screws

    4. Anterior open reduction and internal fixation

    5. Anterior open reduction and internal fixation and posterior fixation Corrent answer: 5

    Fortunately, surgical treatment of sub-acute pelvic ring injuries is relatively uncommon as acute management has become more common. Delayed reconstruction of pelvic ring malunion and impending malunion is rare.

    Nonsurgical management may have a role as long as the hemipelvis does not flex, shorten, and/or externally rotate. The AP pelvic radiograph suggests that all three motions are happening in this patient. These are just a few of the indications to repair the pelvic ring and this is best done with anterior and posterior fixation. Anterior symphyseal plating will help correct most of the deformity. Posterior fixation can and should be added to lessen the forces on the anterior ring reconstruction when repair is performed in a sub-acute or delayed fashion. Posterior fixation can help obtain a more anatomic reduction and helps decrease the risk of anterior hardware failure.

     

     

     

    OrthoCash 2020

     

  191. To avoid an injury to the L5 nerve root when placing an S1 sacroiliac screw, what area of the sacrum should be avoided on the lateral C arm image shown in Figure 21?

     

     

     

    1. A

    2. B

    3. C

    4. D

    5. E

     

    Corrent answer: 1

     

    Safe placement of a sacroiliac screw depends on excellent imaging of and understanding of pelvic anatomy. There are variations in the anatomy of the upper sacrum. Patients with dysplasia of the sacrum can have “in-out-in” screws placed that exit the ilium, pass anterior to the sacral ala, and injure the L5 nerve root. To make sure that this does not occur, a lateral image of the sacrum is used to ensure that the starting point is in the “safe zone.” The starting point needs to be below the iliac cortical density (ICD) which parallels the sacral alar slope. This will prevent placing screws into the recessed ala of patients with a dysplastic sacrum. The triangular area anterior to the ICD is labeled A in the figure, B represents the sacral canal, C is S2, D is the anterior border of the sacrum, and E represents the greater sciatic notches.

     

    Illustration A (Routt et al) shows the important landmarks and safe zone of the lateral sacral view for screw placement.

     

     

     

     

     

     

    OrthoCash 2020

     

  192. An otherwise healthy 37-year-old man fell off the flatbed of a delivery truck and landed directly on his dominant left hand. Surgical stabilization of a distal radius fracture is performed. An intraoperative radiograph is shown in Figure 22. What is the next most appropriate step in management?

     

     

     

    1. Immobilization of the wrist in ulnar deviation for 4 weeks before starting range-of-motion exercises

    2. In situ Kirschner wire fixation of the carpal bones for 6 weeks

    3. Extending the volar incision used for fracture fixation and repairing the injured structures in addition to percutaneous Kirschner wire fixation

    4. Performing a separate dorsal incision and repairing the injured structures in addition to percutaneous Kirschner wire fixation

    5. Arthroscopic repair of the injured structures and percutaneous Kirschner wire fixation

    Corrent answer: 4

     

    The intraoperative radiograph reveals a scapholunate ligament disruption. Repair of the stout dorsal scapholunate interosseous ligaments is required. Interestingly, the results of scapholunate ligament injuries associated with distal radius fractures appear to be superior to those of isolated ligament injuries.

     

     

     

    OrthoCash 2020

     

  193. A 36-year-old woman is placed in a short arm cast for a nondisplaced extra-articular distal radius fracture. Seven weeks later she notes the sudden inability to extend her thumb. What is the most likely cause of her condition?

    1. Posterior interosseous nerve palsy

    2. Cervical disk herniation

    3. Entrapment of the flexor pollicis longus tendon

    4. Rupture of the extensor pollicis longus tendon

    5. Metacarpophalangeal joint dislocation Corrent answer: 4

    A recent review of 200 consecutive distal radius fractures noted that the overall incidence of extensor pollicis longus rupture was 3%. The causes are believed to be mechanical irritation, attrition, and vascular impairment. The fracture is usually nondisplaced and the patient notes weeks to months after injury the sudden, painless inability to extend the thumb. Treatment involves extensor indicis proprius tendon transfer or free palmaris longus tendon grafting.

     

     

     

    OrthoCash 2020

     

  194. In Gustilo type III open tibial diaphyseal fractures, which of the following factors is associated with an increased risk of a poor functional outcome?

    1. Soft-tissue coverage within 3 days of injury

    2. Bone grafting 3 months after injury

    3. Wound debridement within 6 to 24 hours from injury

    4. Definitive treatment with external fixation

    5. Free tissue transfer for soft-tissue coverage Corrent answer: 4

    According to the published outcomes analyses from the Lower Extremity Assessment Project (LEAP) study group of patients prospectively followed for 2 to 7 years, definitive fixation with an intramedullary nail has shown improved outcomes when compared to definitive external fixation. The findings showed that the timing of wound debridement (within 6 hours from injury as compared to within 6 to 24 hours), the timing of soft-tissue coverage (3 days or less from injury as compared to more than 3 days), and the timing of bone grafting after injury (within or after 3 months) did not impact the infection or union rates and had no effect on functional outcome. The LEAP study has shown at 7-year follow-up that patients who are definitively treated with external fixation have a significantly longer time to union, poorer functional outcomes, longer time to achieve full weight bearing, and more time in the hospital.

     

     

     

    OrthoCash 2020

  195. Figures 23a and 23b show the radiographs of a 75-year-old woman who sustained an injury to her nondominant hand. Initial treatment should consist of

     

     

     

     

     

     

     

    1. closed reduction and splinting.

    2. open reduction and internal fixation through a volar approach.

    3. external fixation and Kirschner wire fixation.

    4. intrafocal pinning and casting.

    5. acceptance of alignment and bracing.

     

    Corrent answer: 1

    Definitive treatment decisions for displaced distal radius fractures in the elderly are based on a number of factors related to the fracture pattern and patient demographics. The first step in any treatment algorithm is a closed reduction and splinting with reassessment of alignment parameters. This is an extra-articular fracture with dorsal angulation. Low-demand elderly patients can be treated well with accepted minor malreduction.

     

     

     

    OrthoCash 2020

     

  196. A 43-year-old man sustained a closed, intra-articular pilon fracture. It has now been 1 year since he underwent open reduction and internal fixation. Which of the following statements most accurately describes his perceived outcome?

    1. His clinical outcome will correlate closely with his initial reduction.

    2. His outcome will correlate with his radiographic score on the Ankle Osteoarthritis Score.

    3. He will likely require a late ankle arthrodesis.

    4. He will demonstrate marked limitations with regard to recreational activities.

    5. He will perceive improvements for a period of over 2 years.

     

    Corrent answer: 5

     

    Marsh and associates retrospectively reviewed 56 tibial plafond fractures and found that the patients perceived improvement in their function and pain for an average of 2.4 years. They demonstrated some limitations in recreational activities but not marked limitations. Patients were unlikely to need a late arthrodesis (13%), and their outcomes did not correlate well with assessments of reduction or arthritis scores.

     

     

     

    OrthoCash 2020

     

  197. The injury shown in Figure 24 was most likely caused by what mechanism of injury?

     

     

     

    1. Anterior posterior compression

    2. Lateral compression

    3. Vertical shear

    4. Combined mechanism

    5. Flexion-rotation

     

    Corrent answer: 2

     

    The CT cut shows a fracture through the posterior portion of the iliac wing or a crescent fracture. This occurs after a laterally directed force is applied to the anterior part of the involved iliac wing.

     

     

     

    OrthoCash 2020

     

  198. A 32-year-old man has an open comminuted humeral shaft fracture. Examination reveals absence of sensation in the first web space and he is unable to fully extend the thumb, fingers, and wrist. What is the recommended treatment following irrigation and debridement of the fracture?

    1. Functional bracing

    2. Hanging long arm cast immobilization

    3. Intramedullary nailing

    4. Open reduction and internal fixation, radial nerve exploration

    5. External bone stimulator

    Corrent answer: 4

     

    There is a high incidence of partial or complete laceration of the radial nerve with high-energy open fractures of the humeral shaft. The recommended treatment is irrigation and debridement of the fracture followed by open reduction and internal fixation and exploration of the radial nerve. If the nerve is completely lacerated, primary repair may be performed but poor outcomes have been reported. If a large zone of nerve injury is identified, delayed nerve grafting is advocated.

     

     

     

    OrthoCash 2020

     

  199. Which of the following is most predictive of a medial side ankle injury in the presence of a fibula fracture above the level of the joint?

    1. Severe medial ankle tenderness

    2. Severe medial ankle ecchymosis

    3. Stress radiographs showing the medial clear space measuring 6 mm and the superior joint space measuring 3 mm

    4. Inability to ambulate

    5. Medial ankle swelling Corrent answer: 3

    Isolated Lauge-Hansen supination-external rotation-type ankle fractures comprise 20% to 40% of ankle fractures and nonsurgical management is effective for managing SER-2 ankle fractures. Tornetta and associates recently showed that medial ankle tenderness, ecchymosis, and swelling are not reliable findings when trying to determine deltoid competence. Stress radiographs showing a medial clear space of greater than 4 mm or one that is also 1 mm greater than the superior joint space, or any lateral talar subluxation are indicative of deltoid incompetence and indicative of a SER-4 ankle fracture.

     

     

     

    OrthoCash 2020

     

  200. What letter in Figure 33 marks the correct starting point for a transiliac pelvic screw?

     

     

     

    1. A

    2. B

    3. C

    4. D

    5. E

     

    Corrent answer: 1

     

    Iliosacral screws cannot always be placed safely due to variations in pelvic anatomy. Transiliac bars are an alternative method for fixation but are placed using an open technique. A screw can be placed percutaneously through both iliac wings posterior to the posterior border of the sacrum. The starting point is visualized using a lateral C arm shot and is located on the posterior iliac crest at about the level of the S1 body where the crest has its largest area posterior to the sacrum. This area is labeled A in the figure, B represents the sacral canal, C is S1, D is the area cephalad to the iliocortical density, and E is the anterior border of the sacrum. The radiograph demonstrates a well-placed sacroiliac screw.

     

     

     

    OrthoCash 2020

     

  201. A 57-year-old man involved in a motor vehicle accident sustains an injury to his right shoulder. A spot AP radiograph is shown in Figure

    34. What is the next most appropriate step in the orthopaedic management of this patient?

     

     

     

    1. Axillary view

    2. CT of the shoulder

    3. Closed reduction

    4. Sling and close follow-up

    5. Functional brace

     

    Corrent answer: 1

     

    The next step in the management of this injury is completion of the shoulder trauma series. An axillary radiograph, which can be quickly performed in the emergency department, must be obtained to accurately assess the humeral head relationship to the glenoid. If difficulty is encountered, a “Velpeau” axillary may be substituted. If that fails to elucidate the status of the glenohumeral joint, a CT scan should be obtained.

     

     

     

    OrthoCash 2020

     

  202. Which of the following findings best describes the acetabular fracture shown in Figure 38?

     

     

     

    1. Posterior column with articular impaction and a free fragment

    2. Anterior column with articular impaction

    3. Posterior wall with an intra-articular fragment

    4. Posterior wall with articular impaction and a free intra-articular fragment

    5. Posterior wall with articular impaction Corrent answer: 4

    The CT scan shows a posterior wall fracture with impaction of the articular surface and a free fragment within the joint. Proper treatment of this injury requires not only reduction and fixation of the posterior wall fragment but also removal of the free fragment and elevation of the depressed articular segment.

     

     

     

    OrthoCash 2020

     

  203. A 28-year-old female firefighter fell from the top of a three-story building in the line of duty. She sustained a displaced pelvic fracture with more than 5 mm displacement. Compared to normal healthy controls, these patients have a higher incidence of

    1. normal sexual function and normal vaginal childbirth.

    2. sexual dysfunction (dyspareunia) and normal vaginal childbirth.

    3. normal sexual function and caesarean section childbirth.

    4. sexual dysfunction (dyspareunia) and caesarean section childbirth.

    5. normal sexual function and caesarean section childbirth until hardware removal.

    Corrent answer: 4

     

    Pelvic trauma in women has been shown to increase the risk of sexual dysfunction and dyspareunia. Additionally, caesarean section childbirth is

    almost universal following pelvic trauma regardless of whether anterior pelvic hardware is present or not.

     

     

     

    OrthoCash 2020

     

  204. A 30-year-old man falls off a 7-foot ladder and sustains the injury seen in the radiograph and the CT scan shown in Figures 39a and 39b. Medical history is negative. Management of this injury should include which of the following?

     

     

     

     

     

     

     

    1. Closed treatment and casting

    2. Open reduction and internal fixation

    3. Primary subtalar arthrodesis

    4. Percutaneous fixation

    5. External fixation

    Corrent answer: 2

     

    A Sanders type 2 intra-articular calcaneus fracture in a young healthy nonsmoker is best treated with open reduction and internal fixation. Whereas nonsurgical management is an option, Buckley and associates have shown that these fractures have a better outcome with surgical care. Percutaneous fixation is reserved for tongue-type fractures and subtalar arthrodesis is used in some type 4 fractures. External fixation has not been shown to be advantageous in closed fractures.

     

     

     

    OrthoCash 2020

     

  205. A 24-year-old woman fell from a horse and landed on her outstretched right arm. Radiographs reveal an elbow dislocation with a type II coronoid fracture and a nonreconstructable comminuted radial head fracture. What is the most appropriate management?

    1. Radial head resection, open reduction and internal fixation of the coronoid, and medial collateral ligament repair

    2. Radial head resection and lateral collateral ligament repair

    3. Radial head arthroplasty alone

    4. Radial head arthroplasty and lateral collateral ligament repair

    5. Radial head arthroplasty, open reduction and internal fixation of the coronoid, and lateral collateral ligament repair

    Corrent answer: 5

     

    The combination of an elbow dislocation and a fracture of the radial head and coronoid is known as a terrible triad injury. To restore elbow stability, each injury must be addressed. The nonreconstructable radial head fracture requires implant arthroplasty. Open reduction and internal fixation of the coronoid is also necessary as is repair of the lateral collateral ligament complex which is usually avulsed from the lateral epicondyle region.

     

     

     

    OrthoCash 2020

     

  206. A 30-year-old man is brought to the emergency department after a motor vehicle accident. He has a closed midshaft femoral fracture and an intra-abdominal injury. He is currently in the operating room and the exploration of his abdomen has been completed. His initial blood pressure was 70/30 mm Hg and is now 90/50 mm Hg after 4 liters of fluid and 2 units of blood. His initial serum lactate was 3.0 mmol/L (normal < 2.5), 1 hour postinjury it was 3.5 mmol/L, and it is now 5 mmol/L. His core temperature is 93 degrees F (34 degrees C).

    What is the most appropriate management for the femoral shaft fracture at this point?

    1. Reamed intramedullary nailing

    2. Traction

    3. External fixation

    4. Open plating

    5. Mast suit

     

    Corrent answer: 3

     

    The patient has several indications that he is not ready for definitive fixation of the femoral shaft fracture at this point. He is cold with a core temperature of 93 degrees F, and hypothermia of less than 95 degrees F (35 degrees C) has been shown to be associated with an increased mortality rate in trauma patients. The patient has also not been resuscitated based on his increasing lactate levels and although controversial, it has been shown that temporary external fixation leads to a lower incidence of multiple organ failure and acute respiratory distress syndrome.

     

     

     

    OrthoCash 2020

     

  207. A 45-year-old male karate instructor sustained the injury shown in Figures 40a through 40c while practicing karate. The decision to proceed with surgery depends on which of the following factors?

     

     

     

     

     

     

    1. MRI scan

    2. Physical examination

    3. Workers’ compensation status

    4. Surgeon availability

    5. Patient age

     

    Corrent answer: 2

    The most important criteria in determining the need for surgery following a nondisplaced or minimally displaced tibial plateau fracture is knee stability to varus/valgus stress. Soft-tissue injury noted on MRI may be addressed at a later time following fracture healing. This fracture pattern is amenable to nonsurgical management. Decisions regarding surgical intervention may be made up to 2 weeks after injury.

     

     

     

    OrthoCash 2020

     

  208. A 32-year-old man has a Glasgow Coma Scale score of 8 and an open pelvic fracture. The patient’s family reports that he is a Jehovah’s Witness. Initial hemodynamic instability has resolved. In the operating room during a washout, the patient’s blood pressure becomes unstable. What is the most appropriate action?

    1. Consult the ethics committee before giving blood.

    2. Use cell saver blood.

    3. Ask the patient’s family for consent to give blood.

    4. Use plasma expanders.

    5. Give the patient blood.

     

    Corrent answer: 4

     

    Certain medical procedures involving blood are specifically prohibited in the belief system of a Jehovah’s Witness whereas others are not doctrinally prohibited. For procedures where there is no specific doctrinal prohibition, a Jehovah’s Witness should obtain the details from medical personnel and make his or her own decision. Transfusions of allogeneic whole blood or its constituents or preoperative donated autologous blood are prohibited. Other procedures, while not doctrinally prohibited, are not promoted such as hemodilution, intraoperative cell salvage, use of a heart-lung machine, dialysis, epidural blood patch, plasmapheresis, white blood cell scans (labeling or tagging of removed blood returned to the patient), platelet gel, erythropoietin, or blood substitutes. The patient should not be given blood. Plasma expanders should be used first to restore hemodynamic stability. Cell saver blood from an open wound is not recommended nor would there likely be enough from an open pelvic fracture to salvage. The patient’s family may be expressing their own beliefs rather than the patient’s beliefs and it would be better to ask the patient when he or she is more alert to determine what procedures they would allow. A consult with the ethics committee will unnecessarily delay an intervention that should restore hemodynamic stability.

     

     

     

    OrthoCash 2020

  209. Figure 50 shows the radiograph of a 26-year-old man who sustained an isolated open injury to his foot. Examination reveals no gross contamination in the wound. There is a palpable dorsalis pedis pulse and sensation is present on the dorsal and plantar aspects of the foot. Initial treatment should consist of wound debridement, antibiotics, and

     

     

     

     

    1. talectomy.

    2. reimplantation of the talus.

    3. reimplantation of the talus with acute triple arthrodesis.

    4. Syme amputation.

    5. transtibial amputation.

     

    Corrent answer: 2

     

    The radiograph shows a complete extrusion of the talus. Reimplantation of the talus after wound debridement has been reported to be safe and successful, and provides for flexibility with any future reconstructive procedures.

     

     

     

    OrthoCash 2020

     

  210. Which of the following long bone fracture patterns occurs after a pure bending force is exerted to the bone?

    1. Spiral

    2. Oblique

    3. Transverse

    4. Segmental

    5. Comminuted

     

    Corrent answer: 3

     

    A pure bending force produces a transverse fracture pattern. Spiral fractures are mainly rotational, oblique are uneven bending, segmental are four-point bending, and comminuted are either a high-speed torsion or crush mechanism.

     

     

     

    OrthoCash 2020

     

  211. A 38-year-old woman fell from a ladder onto her right hip. The radiographs and CT scan are shown in Figures 52a through 52d. What is the best surgical approach for this fracture?

     

     

     

     

     

     

     

     

     

     

     

    1. Kocher-Langenbeck

    2. Iliofemoral

    3. Ilioinguinal

    4. Extended iliofemoral

    5. Triradiate approach

     

    Corrent answer: 3

     

    The fracture is an associated both column fracture. The best approach for this fracture is the ilioinguinal. The Kocher-Langenbeck is best for posterior injuries to the acetabulum and some transverse fractures. The iliofemoral alone is limited to high anterior column injuries. The extended iliofemoral and triradiate

    approaches although useful for this fracture, have a higher rate of complications.

     

     

     

    OrthoCash 2020

     

  212. An otherwise healthy 26-year-old woman is involved in a high speed motor vehicle accident and sustains the injury shown in Figure 54 to her dominant right arm. Appropriate treatment of this injury complex includes

     

     

     

     

    1. plating of the radial shaft fracture then open repair of the triangular fibrocartilage complex.

    2. open reduction and internal fixation of the radius and ulna.

    3. plating of the radius then closed reduction and evaluation of the distal radioulnar joint (DRUJ).

    4. closed reduction of the radius and DRUJ.

    5. plating of the radius then pinning of the DRUJ in pronation.

     

    Corrent answer: 3

     

    This Galeazzi fracture is an injury that requires surgical treatment in an adult. The algorithm includes anatomic reduction of the radial shaft and closed reduction of the DRUJ with assessment of stability. If the DRUJ remains unstable, supination of the wrist may reduce the DRUJ. If not, either open or closed reduction with pinning is undertaken. The closer the radius fracture is to the DRUJ, the more likely it is to be unstable.

     

     

    OrthoCash 2020

     

  213. A 40-year-old laborer sustains the injury shown in the radiograph and CT scan in Figures 56a and 56b. What is the most common complication associated with surgical intervention?

     

     

     

     

     

     

     

    1. Chronic osteomyelitis

    2. Planovalgus hindfoot

    3. Plantar nerve entrapment

    4. Wound dehiscence

    5. Painful hardware

     

    Corrent answer: 4

    The patient has a severe Sanders type 4 calcaneus fracture. By far the most common complication associated with surgical treatment of calcaneus fractures is wound dehiscence.

     

     

     

    OrthoCash 2020

     

  214. Patients in compensated shock (normal vital signs) are thought to be at risk for which of the following?

    1. A primed immune system with an increased risk of a systemic inflammatory response

    2. Nothing since they are no longer in uncompensated shock and their vital signs have normalized

    3. Higher nonunion rates after fracture fixation

    4. Higher infection rates after definitive fracture fixation

    5. Higher complication rates after temporizing external fixation of long bone fractures

    Corrent answer: 1

     

    Patients who are in compensated shock have normal vital signs but still have hypoperfusion of organ beds such as the splanchnic circulation due to preferential perfusion of the heart and brain. The response to this continued hypoperfusion may be the development of a systemic inflammatory response that may lead to multiple organ failure. The patients are thought to be at risk for a “primed” immune system due to the ongoing stimulation of the immune system and may have an exaggerated response to a second stimulus such as surgery or infection. Other markers of resuscitation should be used besides vital signs to determine when resuscitation has been completed. The use of temporizing fixation has been shown to lower systemic complication rates, and the infection and union rate after staged fixation is not altered.

     

     

     

    OrthoCash 2020

     

  215. A 14-year-old boy sustains a right leg injury after being thrown from his motorcycle while racing. He reports diffuse right leg pain starting at his knee and proceeding distally to include his foot. After the injury the patient’s mother reports the tibia moving posteriorly then anteriorly while she was supporting the leg. In the emergency department 4 hours after injury, examination reveals a large knee effusion, firm compartments of the leg, a palpable posterior tibialis pulse with a warm, pink foot, and capillary refill of 2 seconds at the toes. His blood pressure is 100/50 mm Hg. Motor examination is

    intact, but there is decreased sensation in the dorsal first interspace and plantar aspect of the foot. Compartment pressure measurement reveals all four compartments with pressures of 33, 36, 33, and 38 mm Hg respectively. Radiographs are shown in Figure 59a and 59b. The remainder of the skeletal examination is normal. What is the optimal management for this injury?

     

     

     

     

    1. Emergent four compartment fasciotomies

    2. Emergent four compartment fasciotomies and open reduction and internal fixation of the fracture

    3. Elevation of the limb overnight and four compartment fasciotomies in the morning

    4. Elevation of the limb overnight and a recheck of compartment pressures in the morning

    5. Emergent MRI of the knee and leg Corrent answer: 2

    The patient has a compartment syndrome based on the firm compartments of the leg and the elevated compartment pressures measured at the diastolic pressure reading. Muscle ischemia occurs quickly when compartment pressures are elevated, and within 6 hours irreversible damage can occur. Emergent fasciotomies permit decompression of all four compartments and reestablishment of vascular supply to the muscles. Stabilization of the fracture prevents further soft-tissue injury.

     

     

    OrthoCash 2020

     

  216. Resuscitation of a trauma patient who has been in hypovolemic shock is complete when which of the following has occurred?

    1. The mean arterial blood pressure is above 90 mm Hg.

    2. The pulse pressure has normalized.

    3. Urine output is greater than 0.5 to 1 mL/kg/h.

    4. Oxygen delivery has been maximized.

    5. Aerobic metabolism has been restored in all tissue beds.

     

    Corrent answer: 5

     

    Shock can be defined as inadequate tissue perfusion. Resuscitation or the resolution of shock is defined as when oxygen debt has been repaid, tissue acidosis is eliminated, and aerobic metabolism has been restored in all tissue beds. The end points for resuscitation are not clearly defined, but occult shock can still be present in the setting of normal vital signs and normal urine output due to selective perfusion of organ systems.

     

     

     

    OrthoCash 2020

     

  217. A 12-year-old girl falls in gymnastics and sustains comminuted midshaft radius and ulna fractures. Closed reduction and cast immobilization are attempted but fracture redisplacement with 20 degrees of angulation occurs. Surgical treatment includes closed reduction and intramedullary fixation of both bones. What is the most common long-term complication for this fracture?

    1. Infection

    2. Malunion

    3. Loss of forearm rotation

    4. Refracture

    5. Delayed union/nonunion

     

    Corrent answer: 3

     

    Healing of forearm fractures in skeletally immature patients is the usual outcome. The use of intramedullary fixation has been reported to result in a lower frequency of refractures when compared to plate osteosynthesis due to the absence of diaphyseal holes after plate removal, which are considered stress risers. Regardless of implant technique, malunion and infection are infrequent. Loss of forearm pronation and supination is a common occurrence in surgically treated fractures due to the higher degree of soft-tissue injury, and periosteal stripping leads to fracture site instability and fracture comminution.

     

     

    OrthoCash 2020

     

  218. The teardrop shape marked with an asterisk in Figure 61 represents what anatomic structure?

     

     

     

     

    1. Anterior superior iliac spine

    2. Sciatic buttress

    3. A column of bone running from the anterior inferior iliac spine (AIIS) to the posterior superior iliac spine (PSIS)

    4. The most superior portion of the roof of the acetabulum

    5. Iliopectineal line

     

    Corrent answer: 3

     

    The teardrop can be visualized on the obturator outlet view of the pelvis and represents a thick column of bone that runs from the AIIS to the PSIS. Half pins for eternal fixation frames or screws can be inserted into this column for fixation of fractures.

     

     

     

    OrthoCash 2020

     

  219. A patient was treated with a revision reamed intramedullary nail for a nonunion 6 months ago. A current radiograph is shown in Figure

    62. Based on these findings, what is the most appropriate treatment?

     

     

     

    1. Electrical stimulation

    2. Bone grafting

    3. No weight bearing

    4. Bone grafting and compression plating

    5. Free vascularized bone transport Corrent answer: 4

    Nonunions after intramedullary nails are often treated with exchange reamed nailing. In a recent study, this resulted in a union rate of 53%. After failed exchange nailing, bone grafting and compression plating should be used. The other options resulted in less satisfactory results as compared to bone grafting and compression plating.

     

     

     

    OrthoCash 2020

     

  220. Figure 63 shows the radiographs of a 23-year-old man who sustained a twisting injury at work. Swelling, tenderness, and ecchymosis are noted about the entire midfoot. What associated injury is most likely to be problematic?

     

     

     

    1. Peroneal tendon tear

    2. Lateral process talus fracture

    3. Talar neck fracture

    4. Lisfranc injury

    5. Deltoid ligament tear Corrent answer: 4

    This cuboid compression fracture (“nutcracker” injury) is associated with subtle injury to the Lisfranc complex. This diagnosis must be made to ensure proper treatment.

     

     

     

    OrthoCash 2020

     

  221. A 24-year-old man is ejected from his motorcycle and sustains a significant hip injury. The fracture shown in Figures 64a through 64e is best described as what type of fracture?

     

     

     

     

     

     

     

     

     

     

     

     

     

     

    1. Posterior column/posterior wall acetabular

    2. Associated both column acetabular

    3. Transverse plus posterior wall acetabular

    4. Anterior column posterior hemitransverse acetabular

    5. Anterior column acetabular Corrent answer: 5

    The radiographs and CT scans reveal an anterior column acetabular fracture. The fracture has quadrilateral plate extension but does not exit out the posterior column. The CT scans confirm an intact posterior column and no wall fracture. A transverse fracture is best seen on the CT scan and runs in the sagittal plane, not the coronal plane.

     

     

     

    OrthoCash 2020

  222. A 71-year-old woman who reports long-term use of oral steroids for asthma is referred for treatment of a distal humerus fracture.

    Radiographs reveal diffuse osteopenia and a severely comminuted intra-articular fracture. What is the most appropriate treatment?

    1. Long arm cast immobilization

    2. Total elbow arthroplasty

    3. Open reduction and internal fixation

    4. Osteoarticular allograft

    5. Resection arthroplasty

     

    Corrent answer: 2

     

    Several studies have documented the satisfactory outcomes of total elbow arthroplasty when osteosynthesis is not feasible for fixation of a distal humerus fracture, particularly in the physiologically older patient with low functional demands. Total elbow arthroplasty should be considered when a comminuted intra-articular distal humerus fracture occurs in a woman older than age 65 years, particularly with such associated comorbidities as systemic steroid use, osteoporosis, or rheumatoid arthritis.

     

     

     

    OrthoCash 2020

     

  223. A 14-year-old boy sustained a 100% displaced distal radius Salter-Harris type II fracture. Neurologic examination demonstrates normal motor examination and two-point discrimination. He undergoes fracture reduction to the anatomic position with the application of a long arm cast. Postreduction he reports increasing hand and wrist pain with diminution of two-point discrimination to 10 mm over the index and middle fingers over the next several hours after surgery. The cast is bivalved and the padding released relieving all external pressure over the arm. Reevaluation reveals increasing sensory deficit over the affected area. What is the next most appropriate management intervention?

    1. Cast removal and measurement of carpal canal pressure

    2. Immediate carpal tunnel release and pinning of the fracture

    3. Continued observation

    4. Surgical reduction and pinning of the fracture

    5. Electromyography/nerve conduction velocity studies Corrent answer: 2

    The patient has an evolving acute carpal tunnel syndrome. Initial management for this injury is to relieve all external pressure that may elevate the neural

    compression. Surgical decompression of the median nerve at the carpal tunnel is the optimal intervention. Further nonsurgical interventions (cast removal or further bivalving) are insufficient to alleviate the neural compression.

     

     

     

    OrthoCash 2020

     

  224. A 25-year-old male polytrauma patient undergoes initial temporary external fixation for a femoral shaft fracture. He is converted to a femoral nail at 7 days. This management can be expected to result in

    1. higher infection rates.

    2. higher nonunion rates.

    3. equal union and infection rates.

    4. higher rate of ARDS.

    5. higher mortality rate.

     

    Corrent answer: 3

     

    Recently Harwood and associates investigated the principles of damage control orthopaedics (DCO) as they apply to patients with femoral shaft fractures.

    When they compared those who underwent initial external fixation of femoral shaft fractures with conversion to an intramedullary nail to those who underwent intramedullary nailing as their initial treatment, they found the following: overall infection rates were comparable in patients receiving DCO versus primary intramedullary fixation; open fracture was an independent risk factor for infection regardless of the treatment method; contamination rates in external fixator pin sites rose considerably when left in place more than 2 weeks and logistic regression analysis suggests that infection rates may increase when conversion to an intramedullary nail occurs after 2 weeks following external fixation; and there was no significant difference in time to union among treatment groups.

     

     

     

    OrthoCash 2020

     

  225. Which of the following is the most stable construct for fixation of an unstable transforaminal sacral fractures?

    1. External fixation

    2. Iliosacral osteosynthesis

    3. Combined iliosacral and lumbopelvic fixation (triangular osteosynthesis)

    4. Anterior pelvic ring plating with bilateral sacroilliac percutaenous screw fixation

    5. Transiliac bars with anterior pelvic ring plating

    Corrent answer: 3

     

    Combined iliosacral and lumbopelvic fixation (triangular osteosynthesis) for sacral fractures has the greatest stiffness when used for an unstable sacral fracture.

     

    The referenced article by Schildhauer et al is a cadaveric study that examined the biomechanical properties of different fixation constructs under cyclic loading and demonstrates that triangular osteosynthesis for unstable transforaminal sacral fractures provides significantly greater stability than iliosacral screw fixation under in-vitro cyclical loading.

     

    Illustration below shows the radiographic appearance of lumbopelvic fixation. The addition of iliosacral fixation would complete triangular osteosynthesis.

     

     

     

     

     

     

    OrthoCash 2020

     

  226. The Cotton test evaluates which of the following structures?

     

    1. Calcaneofibular ligament

    2. Lateral ulnar collateral ligament of the elbow

    3. Ligamentum flavum

    4. Anterior talofibular ligament

    5. Ankle syndesmosis

    Corrent answer: 5

     

    The inferior tibiofibular syndesmosis is a fibrous articulation consisting of four ligaments; the elasticity of these ligaments permits axial, vertical, anterior, posterior, and mediolateral motion at the ankle syndesmosis during weight bearing.

     

    Of note, the Cotton test was originally described around 1910 by Frederic J. Cotton as the "talar glide test" evaluating the medial/lateral translation of the talus in the mortise. A positive result indicates disruption of the ankle syndesmosis in the face of an ankle injury.

     

    Nielson et al reported that the level of the fibular fracture does not correlate reliably with the integrity or extent of the interosseous membrane (IOM) tears identified on MRI in operative ankle fractures. Therefore, one cannot consistently estimate the integrity of the IOM and subsequent need for transsyndesmotic fixation based solely on the level of the fibular fracture. This supports the need for intraoperative stress testing (ie, external rotation stress or Cotton test) of the ankle syndesmosis in all operative ankle fractures.

     

    The study by Leeds et al noted a correlation between syndesmosis reduction (initial and final) and outcomes (radiographic and clinical).

     

    The attached video shows the Cotton test during an ankle fixation procedure.

     

     

     

    OrthoCash 2020

     

  227. A comminuted femoral shaft fracture is treated with an intramedullary nail locked with a single distal screw. What is the most likely mode of failure of the screw?

    1. Screw pulls out of the cortical shaft

    2. Screw head breaks off due to bending stresses

    3. Shaft of the screw fractures in the region that is inside the nail

    4. Screw threads are damaged by fretting against the edges of the holes in the nail

    5. Screw bends excessively Corrent answer: 3

    The screw is being loaded and pushed distally at the two points where it contacts the walls of the nail, and it is being pushed proximally at the two points where it contacts the cortex, ie, near the head and tip of the screw. This places the screw in four-point bending, producing tensile stresses on the

    inferior side of the screw and compressive stresses on the superior side. The tensile stresses, combined with stress risers at the screw threads, eventually could lead to fatigue fracture of the screw. Because the cortices in the metaphysis are far apart, the bending moment is large and, therefore, stresses near the midshaft of the screw produced by bending are much larger than shear stresses in this case. Pullout of the screw is unlikely because the loads are not directed along the axis of the screw. There are no bending stresses at the ends of the screw. A bent screw may be difficult to remove, but this would not likely cause failure of the fixation.

     

     

     

    OrthoCash 2020

     

  228. A 30-year-old female presents with the injury shown in Figure A after falling on her outstretched arm. During operative treatment of the fracture, anatomic reduction of the radius is achieved. However, the surgeon is unable to reduce the distal radioulnar joint. What structure is most likely impeding the reduction?

     

     

     

     

    1. Median nerve

    2. Flexor carpi radialis

    3. Pronator quadratus

    4. Extensor carpi ulnaris

    5. Flexor carpi ulnaris

    Corrent answer: 4

     

    Figure A shows a Galeazzi fracture (distal 1/3 radial shaft fracture with associated distal radioulnar joint dislocation). In this injury, an inability to reduce the distal radioulnar joint in a closed fashion is most commonly secondary to interposition of the extensor carpi ulnaris tendon. Early recognition of the dislocation of the ulna and ECU into the DRUJ and their significance may avoid poor results.

     

    The referenced study by Biyani et al reports a case in which both the extensor carpi ulnaris and extensor digiti minimi tendons were displaced on either side of the ulnar head.

     

    The referenced study by Budgen et al presents a case of a Galeazzi fracture dislocation with an irreducible distal radioulnar joint.

     

    The referenced study by Paley et al reports two cases of distal radioulnar joint (DRUJ) disruption and diastasis secondary to distal radial fractures that were associated with displacement of the ulnar styloid and extensor carpi ulnaris (ECU) into the DRUJ. Both cases had a palpable empty ECU tendon sulcus.

     

     

     

    OrthoCash 2020

     

  229. What is the antibiotic of choice for gonococcal septic arthritis of the knee?

    1. Erythromycin

    2. Penicillin

    3. Tetracycline

    4. Ceftriaxone

    5. Vancomycin

     

    Corrent answer: 4

     

    Gonococcal septic arthritis, caused by the gram-negative diplococcus Neisseria gonorrhoeae, typically affects two age groups: newborns and adolescents. The level of penicillin and tetracycline resistance in Neisseria gonorrhoeae is so high that it is completely ineffective in most parts of the world. A third-generation cephalosporin such as ceftriaxone is recommended in most areas. Fluoroquinolones may be an alternative treatment option if antimicrobial susceptibility can be documented by culture. In areas where co-infection with chlamydia is common, doxycycline may be used with ceftriaxone.

     

     

    OrthoCash 2020

     

  230. An above-the-knee amputation is performed 12 cm above the joint line. What is the best management of the adductor muscle group?

    1. Resection of the adductors to prevent adductor contracture

    2. Shortening and reattachment to the mid-femur to improve biomechanics

    3. Myodesis to the distal end of the bone

    4. Transfer to the quadriceps to improve hip flexion

    5. Attachment to the hamstrings Corrent answer: 3

    The best socket fit requires resection of the wide flair of the condyles and amputation approximately 12 cm above the joint line. The adductor magnus is a very important muscle that participates in achieving a more efficient gait.

    Myodesis of the bone through drill holes near the cut end of the bone has been shown to improve biomechanics.

     

     

     

    OrthoCash 2020

     

  231. In determining the FRAX score (fracture risk assessment tool), the World Health Organization determined that which of the following risk factors is not contributory to the clinical risk of fracture in its population cohorts?

    1. BMI (body mass index)

    2. Spine T-score from DEXA scan (dual-energy absorptiometry)

    3. Current smoking activity

    4. Parental history of hip fracture

    5. Prior history of fracture before age 50 Corrent answer: 2

    The FRAX score calculates the clinical risk of fracture using bone mineral density of the femoral neck, BMI, current smoking activity, history of parental hip fracture, and prior personal history of fracture before age 50. The World Health Organization has developed this new fracture risk assessment tool to identify individuals at high risk of osteoporotic fracture. The current standard, which bases treatment decisions largely on bone mineral density measurement, has proven to be specific, but not sensitive, for the identification of patients at high risk of fracture. Because nearly 50% of postmenopausal women in the community older than age 50 years who suffer an osteoporotic fracture do not have osteoporosis defined by a BMD test and because of the limited availability of BMD in many countries, clinical risk factors were added to BMD to identify patients at high risk for osteoporotic fractures. The site and

    reference technology is DEXA at the femoral neck. T-scores are based on the National Health and Nutrition Examination Survey reference values for women aged 20 to 29 years. The same absolute values are used in men. Although the model is constructed for BMD at the femoral neck, the total hip site is thought to predict fracture equivalently in women.

     

     

     

    OrthoCash 2020

     

  232. Glenohumeral disarticulation often leads to which of the following changes?

    1. Hiking of the shoulder girdle

    2. Hypertrophy of the amputated shoulder girdle

    3. Improvement in thoracic spinal deformity

    4. Protraction of the shoulder

    5. Winging of the scapula Corrent answer: 1

    Postural abnormalities are common after high upper extremity amputation. Normally the weight of the upper extremity and the shoulder girdle muscles keep the shoulder balanced. When the arm is amputated and the scapula remains, the shoulder girdle muscles are unopposed, resulting in upward movement often called "hiking" of the shoulder girdle. In a growing child, removal of the entire upper limb can result in scoliosis of the spine due to muscle imbalance. Abnormal shoulder elevation can often be minimized by corrective exercises and wearing a shoulder prosthesis.

     

     

     

    OrthoCash 2020

     

  233. A 10-day-old girl has decreased active motion of the left upper extremity. The mother reports a difficult vaginal delivery with presumed shoulder dystocia. Examination shows full passive range of motion of the shoulder, elbow, and wrist but only active flexion of the fingers and wrist. Factors predictive of a good outcome include which of the following?

    1. Breech delivery

    2. Absence of an ipsilateral clavicle fracture

    3. Horner's sign and an APGAR score of 10 at 1 minute

    4. Return of active biceps before 3 months and preservation of full passive shoulder range of motion

    5. Absent Moro and Babinski reflexes

    Corrent answer: 4

     

    Return of active biceps before 3 months and preservation of full passive shoulder range of motion are predictors of a good outcome. Breech delivery is usually associated with preganglionic injury. Preganglionic injury can result in a Horner's sign, which includes ptosis, myosis, and anhydrosis. Preganglionic injuries are unlikely to recover. The Moro reflex is elicited by dropping a baby's head a short distance and observing active elbow extension and fanning of the fingers, followed by elbow flexion and crying. Absence of the Moro reflex suggests a poor prognosis.

     

     

     

    OrthoCash 2020

     

  234. An 18-month-old child was involved in a motor vehicle accident and sustained an isolated injury to the left upper extremity. A radiograph is shown in Figure 33. What is the most appropriate management for this injury?

     

     

     

     

    1. Hanging arm cast

    2. Closed reduction with flexible intramedullary nail fixation

    3. Coaptation splinting and bandaging the arm to the thorax

    4. Closed reduction and external fixation

    5. Locking plate fixation Corrent answer: 3

    Humeral shaft fractures in infants and young children heal rapidly and have excellent remodeling potential. Appropriate treatment in this age group is immobilization with a coaptation splint and bandaging the arm to the thorax for comfort. Internal fixation is appropriate in multiple trauma, and external fixation may be useful when soft-tissue injury is extensive.

     

     

     

    OrthoCash 2020

  235. If a percutaneous iliosacral screw is placed too anteriorly, and the screw exits anterior to the sacral ala before re-entering the sacral body, what will be the most likely finding postoperatively?

    1. Lack of ankle dorsiflexion

    2. Lack of ankle plantarflexion

    3. Lack of knee extension

    4. Loss of bowel and/or bladder control

    5. Lack of great toe extension Corrent answer: 5

    This question is a simple review of anatomy and nerve innervation. The L5 root is at risk with an "in-out-in" screw, as described in the question, as the nerve root is just anterior to the sacral ala as it enters the true pelvis. L5 is primarily evaluated by extensor hallucis longus function. L4 is tested with tibialis anterior function and S1 by gastroc-soleus function (ankle plantarflexion).

     

     

     

    OrthoCash 2020

     

  236. A 10-year-old boy is struck by a car and sustains open left tibia and fibula fractures with bone protruding through a 7-cm laceration, multiple deep and superficial abrasions over the anterior leg, and road gravel is present in the wounds. His foot is warm and well-perfused with normal sensation and he has no pain with passive range of motion of the toes. Optimal treatment should consist of

    1. irrigation and debridement of the fractures and application of an external fixator.

    2. irrigation and debridement of the fractures and a reamed intramedullary nail.

    3. irrigation and debridement of the fracture and percutaneous Kirschner wire fixation.

    4. submuscular plating.

    5. reduction and a short leg cast.

     

    Corrent answer: 1

     

    The patient has a grade 2 open fracture and therefore needs wound debridement as a first step, followed by fracture stabilization preferably with an external fixator. A reamed intramedullary nail is not indicated in a 10-year-old child with open growth plates. Submuscular plating is not needed in an open fracture and there is no mention of fracture debridement. Percutaneous Kirschner wires will not provide adequate fracture stabilization, nor will a short leg cast. Flexible nailing should be considered as another form of fixation.

     

     

    OrthoCash 2020

     

  237. Figures 5a and 5b show the radiographs of a 21-year-old wrestler who reports that his leg was rolled over while wrestling. The patient has decreased sensation and function in the distribution of the peroneal nerve, and he has absent pulses. What is the most appropriate initial management at this time?

     

     

     

     

    1. Acute reconstruction of all ligamentous structures

    2. Emergency MRI and reconstruction of all ligamentous structures

    3. Emergency arteriogram followed by MRI

    4. Emergency surgery with open reduction and repair of all torn structures with vascular surgery available

    5. Closed reduction in the emergency room and reevaluation of the vascular status

    Corrent answer: 5

     

    The patient has an acute traumatic anteromedial dislocation of the knee with occlusion of the popliteal artery with a possible tear. Treatment should include reduction and reevaluation of the vascular status. At this time, if pulses are symmetric, observation may be appropriate without surgical intervention of the artery, but documentation with studies would be appropriate. Delayed reconstruction of injured structures is appropriate.

     

     

    OrthoCash 2020

     

  238. A 26-year-old male presents after a motor vehicle accident. Work-up reveals a closed left femoral shaft fracture, and an ipsilateral posterior wall fracture. He undergoes intramedullary nailing of the femur, and open reduction internal fixation of the posterior wall. He is treated with 25 mg of indomethacin three times daily for 6 weeks following an initial dose on the evening of surgery for heterotopic ossification prophylaxis. Which of the following is true regarding this post-operative treatment protocol?

  1. It is associated with an increased rate of femoral shaft nonunion

  2. It has no effect on the healing time of the posterior wall fracture

  3. It is associated with a faster time to union

  4. Indomethacin is superior to radiation treatment in the prevention of heterotopic ossification

  5. There is a decreased rate of revision surgery needed when indomethacin is administered post-operatively

Corrent answer: 1

 

Heterotopic ossification (HO) prophylaxis with indomethacin has been shown to increase the risk of long-bone nonunion.

 

Indomethacin therapy has been shown to be an effective means of preventing HO formation, however literature has shown that it increases the risk of long bone and acetabular nonunion. Indomethacin works primarily by inhibiting IGF-1, which is a different mechanism from other NSAID's which typically inhibit the COX enzymes. IGF-1 is important for bone healing, and its inhibition may be a risk factor for delayed bone healing.

 

Burd et al performed a study to determine if patients with an acetabular fracture, who received indomethacin for prophylaxis against HO, were at risk of delayed healing or nonunion of any associated fractures of long bones. The study group consisted of 112 patients who had sustained at least one concomitant fracture of a long bone; of which 36 needed no prophylaxis, 38 received focal radiation and 38 received indomethacin. When comparing patients who received indomethacin with those who did not, a significant difference was noted in the rate of long bone nonunion (26% vs 7%).

 

Jordan et al performed a study to document the efficacy of variable treatment durations with indomethacin prophylaxis for HO and its effect on union of the posterior wall (PW) in operatively treated acetabular fractures. Patients were randomly assigned to one of four treatment groups: (1) placebo for 6 weeks,

(2) 3 days of indomethacin followed by placebo for a total of 6 weeks, (3) 1